Download as pdf or txt
Download as pdf or txt
You are on page 1of 74

JEE (MAIN & ADV.

), MEDICAL + BOARD, NDA, X & IX


Enjoys unparalleled reputation for best results
in terms of percentage selection
www.newtonclasses.net

TRIGONOMETRIC FUNCTION ( )
Only one option is correct.
1. The numerical value of tan 9° − tan 27° − tan 63° + tan 81° is
(a) 2 (b) 3 (c) 4 (d) 5
2. The numerical value of tan 3° + tan 42° + tan 3° tan 42° is
1 3
(a) 0 (b) (c) 1 (d)
2 2
1
3. 3 cos 23° − sin 23°  =
4
(a) cos 43° (b) cos 7° (c) cos 53° (d) None of these
4. The value of sin 47° + sin 61° − sin11° − sin 25° =
(a) sin 36° (b) cos 36° (c) sin 7° (d) cos 7°
5. The value of cos 52° + cos 68° + cos172° is
3
(a) 0 (b) 1 (c) 2 (d)
2
cos 9° + sin 9°
6.
cos 9° − sin 9°
=

(a) tan 54° (b) tan 36° (c) tan18° (d) none of these
7. If in any ∆ ABC , the value of ∠A is obtained from the equation 3cos A + 2 = 0 , then the equation
whose roots are sin A and tan A , is :
(a) 6 x 2 + 5 x − 5 = 0 (b) 6 x 2 + 5 x − 5 = 0 (c) x 2 + 5 x − 5 = 0 (d) none of these
1 − sin θ 1 + sin θ
8. If θ lies in the second quadrant, then the value of is equal to :
1 + sin θ 1 − sin θ
+

(a) 2sec θ (b) −2sec θ (c) 2 cosec θ (d) none of these


1 + sin 2 A + 1 − sin 2 A
9. If tan A < 1 and A is acute, then is equal to
1 + sin 2 A − 1 − sin 2 A
(a) tan A (b) − tan A (c) cot A (d) − cot A
10. The expression cos 2 ( A − B ) + cos 2 B − 2 cos ( A − B ) cos A cos B is :
(a) dependent on B (b) dependent on A and B
(c) dependent on A (d) independent of A and B
 cos A + cos B   sin A + sin B 
11. The value of   (where n is an even) is :
 sin A − sin B   cos A − cos B 
n n

 +

(a) 2 tan n  (b) 2 cot n  (c) 0 (d) none of these


 2   2 
 A− B   A− B 
 

12. If sin θ , cos θ are the roots of ax 2 − bx + c = 0 , then :


(a) a 2 + b 2 = 2ac (b) a 2 − b 2 = 2ac (c) a 2 + b 2 = c 2 (d) b 2 − a 2 = 2ac

13. The value of 2 cos x − cos 3 x − cos 5 x is equal to :

Office.: 606 , 6 Floor, Hariom Tower, Circular Road, Ranchi-1, Ph.: 0651-2562523, 9835508812, 8507613968
th
2 BY R. K. MALIK’S NEWTON CLASSES
( )
(a) 16 cos x sin x
3
(b) 16 sin 3 x cos 2 x
2
(c) 4 cos3 x sin 2 x (d) 4sin 3 x cos 2 x
14. The value of sin12° sin 48° sin 54° is equal to :
1 1 1 1
(a) (b) (c) (d)
16 32 8 4
15. If sec α and cosec α are the roots of the equation x 2 − px + q = 0 , then :
(a) p 2 = q ( q − 2 ) (b) p 2 = q ( q + 2 ) (c) p 2 + q 2 = 2q (d) none of these

16. The value of cos 2 A ( 3 − 4 cos 2 A ) + sin 2 A ( 3 − 4sin 2 A ) is equal to :


2 2

(a) cos 4A (b) sin 4 A (c) 1 (d) none of these

17. If α , β , γ , δ are four solution of the equation tan  θ +  = 3 tan 3θ , then tan α tan β tan γ tan δ
4
 π

equals :

1
(a) 3 (b) 1 / 3 (c) − (d) none of these
3
18. If tan α / 2 and tan β / 2 are the roots of the equation 8 x 2 − 26 x + 15 = 0 , then cos (α + β ) is equal to :
627 627
(a) − (b) (c) −1 (d) none of these
725 725

19. The numerical value of 2 tan + 3sec − 4 cos is equal to


10 10 10
π π π

(a) 5 (b) 1 (c) 0 (d) none of these


tan 77° + cot 47°
20. If = tan 77° cot α , then α equals
tan13° + cot 43°
(a) 77° (b) 47° (c) 13° (d) 43°
21. The value of 2 ( sin 6 θ + cos 6 θ ) − 3 ( sin 4 θ + cos 4 θ ) + 1 is :
(a) 2 (b) 0 (c) 4 (d) 6
sin ( x + y ) tan x
22. If , then is equal to :
sin ( x − y ) tan y
a+b
=
a −b

(a) (b) (c) ab (d) none of these


b a

23. If tan θ + sec θ = e x , then cos θ equals :


a b

2
(a) (b) x − x (c) (d) x − x
2 2
e x + e− x e x − e− x e x − e− x

24. The expression tan 2 α + cot 2 α is :


e +e e +e

(a) ≥ 2 (b) ≤ 2 (c) ≥ −2 (d) none of these


3π 4π
25. The value of cos 2 + cos 2 is equal to :
5 5
(a) 3 / 4 (b) 5 / 4 (c) 5 / 2 (d) 4 / 5
26. cos θ − sin θ is equal to :
4 4

(a) 1 + 2 sin 2   (b) 2 cos 2 θ − 1 (c) 1 − 2sin 2   (d) 1 + 2 cos 2 θ


2 2
θ  θ 

1 − tan 2 15°
27. The value of is :
1 + tan 2 15°

Office.: 606 , 6 Floor, Hariom Tower, Circular Road, Ranchi-1, Ph.: 0651-2562523, 9835508812, 8507613968
th
( ) BY R. K. MALIK’S NEWTON CLASSES 3

(a) 1 (b) 3 (c) 3/2 (d) 2


28. If sin θ + cosec θ = 2 , then sin 2 θ + cosec 2 θ is equal to :
(a) 1 (b) 4 (c) 2 (d) none of these
3π 5π 7π 9π
29. tan tan tan tan tan
20 20 20 20 20
π
=

(a) −1 (b) 1 (c) 1 / 2 (d) ∞


30. The least value of 2sin 2 θ + 3cos 2 θ is :
(a) 1 (b) 2 (c) 3 (d) 5
31. The greatest value of sin 4 θ + cos 4 θ is :
(a) 1 / 2 (b) 1 (c) 2 (d) 3
32. The value of sin (π /10 ) sin (13π / 10 ) is equal to :
(a) 1 / 2 (b) −1/ 2 (c) −1/ 4 (d) 1
33. Given π / 2 < α < π , then the expression (1 − sin α ) / (1 + sin α ) + (1 + sin α ) / (1 − sin α ) =
(a) 1 / cos α (b) −2 / cos α (c) 2 / cos α (d) none of these
7π 4π
34. sin 2 + sin 2 + sin 2 + sin 2
18 9 18 9
π π
=

(a) 1 (b) 4 (c) 2 (d) 0


35. If θ and φ are angles in the first quadrant such that tan θ = 1/ 7 and sin φ = 1/ 10 , then :
(a) θ + 2φ = 90° (b) θ + 2φ = 30° (c) θ + 2φ = 75° (d) θ + 2φ = 45°

36. If tan x = then (a + b) / ( a − b) + ( a − b) / ( a + b) where a > b > 0 , is equal to :


b

(a) 2 sin x / sin 2 x (b) 2 cos x / cos 2 x (c) 2 cos x / sin 2 x (d) 2 sin x / cos 2 x
a

3 + cot 76° cot16°


37.
cot 76° + cot16°
=

(a) tan16° (b) cot 76° (c) tan 46° (d) cot16°
1
38. If sin (α + β ) = 1 , sin (α − β ) = , α , β ∈  0,  , tan (α + 2 β ) tan ( 2α + β ) =
2  2
 π

(a) 1 (b) 2 (c) –1 (d) none of these


2sin θ 1 − cos θ + sin θ
39. If x = , then :
1 + cos θ + sin θ 1 + sin θ
1
(a) 1 + x (b) 1 − x (c) x (d)
x

40. If a ≤ 3cos x + 5sin  x −  ≤ b for all x , then ( a, b ) =


6
 π

(a) − 19, 19 (b) ( −17, 17 ) (c) − 21, 21 (d) none of these



( ) ( )
41. If sin ( x + 3α ) = 3sin (α − x ) , then :
(a) tan x = tan α (b) tan x = tan 2 α (c) tan x = tan 3 α (d) tan x = 3 tan α
42. If a sec α − c tan α = d and b sec α + d tan α = c , then :
(a) a 2 + c 2 = b 2 + d 2 (b) a 2 + d 2 = b 2 + c 2 (c) a 2 + b 2 = c 2 + d 2 (d) ab = cd

Office.: 606 , 6 Floor, Hariom Tower, Circular Road, Ranchi-1, Ph.: 0651-2562523, 9835508812, 8507613968
th
4 ( ) BY R. K. MALIK’S NEWTON CLASSES

43. Given < α < π , then the expression


(1 − sin α ) + (1 + sin α ) =
2 (1 + sin α ) (1 − sin α )
π

1 2 2
(a) (b) − (c) (d) none of these
cos α cos α cos α
44. If the angle A of a triangle ABC is given by the equation 5cos A + 3 = 0 , then sin A and tan A are
the roots of the equation :
(a) 15 x 2 − 8 x − 16 = 0 (b) 15 x 2 − 8 2 x + 16 = 0 (c) 15 x 2 − 8 x + 16 = 0 (d) 15 x 2 + 8 x − 16 = 0
45. If sin θ1 + sin θ 2 + sin θ3 = 3 , then cos θ1 + cos θ 2 + cos θ3 is equal to :
(a) 3 (b) 2 (c) 1 (d) 0

46. The value of ∑ cos ( 5r ) ° , where x° denotes the x degree, is equal to :


18
2

r =1

(a) 0 (b) 7 / 2 (c) 17 / 2 (d) 25 / 2


47. tan 75° − cot 75° =
(a) 2 3 (b) 2 + 3 (c) 2 − 3 (d) none of these
48. sin163° cos 347° + sin 73° sin167° =
(a) 0 (b) 1 / 2 (b) 1 (d) none of these
49. If 2 sec 2α = tan β + cos β , then one of the values of α + β is :

(a) (b) (c) π (d) 2π


4 2
π π

50. The value of 6 ( sin 6 θ + cos 6 θ ) − 9 ( sin 4 θ + cos 4 θ ) + 4 is


(a) −3 (b) 0 (c) 1 (d) 3

51. If cos x + cos y + cos α = 0 and sin x + sin y + sin α = 0 , then cot 
 2 
 x+ y
=

(a) sin α (b) cos α (c) cot α (d) sin 


 2 
 x+ y

52. If tan θ + sec θ = e x , then cos θ equals :


2
(a) (b) (c) (d)
(e + e− x ) (e − e− x ) (e − e− x )
2 2
x x x

( e + e− x )
x
(e x
+ e− x )
53. If tan A + cot A = 4 , then tan 4 A + cot 4 A is equal to :
(a) 110 (b) 191 (c) 80 (d) 194
tan 70° − tan 20°
54. The value of
tan 50°
=

(a) 1 (b) 2 (c) 3 (d) 0


55. sin θ + cos θ + 3sin θ cos θ =
6 6 2 2

(a) 0 (b) –1 (c) 1 (d) none of these


56. The numerical value of sin 9° − cos 9° is :
5− 5 5− 5 5− 5 5− 5
(a) ± (b) − (c) (d)
2 2 4 2
57. If cos2 x + cos4 x = 1, then tan 2 x + tan 4 x is equal to
(a) 1 (b) 0 (c) 2 (d) none of these

Office.: 606 , 6 Floor, Hariom Tower, Circular Road, Ranchi-1, Ph.: 0651-2562523, 9835508812, 8507613968
th
( BY R. K. MALIK’S NEWTON CLASSES
) 5
58. If tan θ = 2 tan φ + 1, then the numerical value of cos 2θ + sin 2 φ is
2 2

1 1 1
(a) 0 (b) (c) (d)
2 3 4
sin α cos α
59. If tan β = then tan (α − β ) = k tan α , where the value of k is
2 + cos2 α
1 2 3
(a) (b) (c) (d) 2
2 3 2
60. The value of log cos1° + log cos 2° + ..... + log cos 44° + log cosec 46° + log cosec 47° +
..... + log cosec 89° is :
(a) log ( cos 45° + cosec 45° ) (b) log cos 45°
(c) log cosec 45° (d) log ( cos 45°.cosec 45° )

61. The numerical value of sin 2 24° − sin 2 6°


( ) ( sin 2
42° − sin 2 12° is
)
1 1 1
(a) 1 (b) (c) (d)
8 12 16
cos8° + sin 8°
62. If = cot α , then the complement of α is
cos 8° − sin 8°
(a) 37° (b) 53° (c) 47° (d) 43°
63. The numerical value of sin 54° + cos 72° is equal to
2 2

1 2 3
(a) 1 (b) (c) (d)
2 3 4
3 1
64. The numerical value of is equal to
sin 20° cos 20°

(a) 1 (b) 2 (c) 3 (d) 4


cot θ tan θ
65. The numerical value of is equal to
cot θ − cot 3θ tan 3θ − tan θ

1 3
(a) 0 (b) (c) 1 (d)
2 2
1
66. The numerical value of − 2 sin 70° is equal to
2 sin10°
1 1
(a) 0 (b) (c) (d) 1
2 3
67. The numerical value of cos 306° + cos 234° + cos162° + cos18° is equal to
(a) 3 (b) 2 (c) 0 (d) −2
 2π  4π
68. If 4 cos θ cos  + θ  cos  + θ  = cos kθ , then the value of k is equal to
 3  3
 

(a) 2 (b) 3 (c) 4 (d) 5


 

69. The numerical value of sin10° sin 50° + sin 50° sin 250° + sin 250° sin10° is k , where the negative of k
is
1 3 1 3
(a) − (b) − (c) (d)
2 4 2 4
70. The numerical value of sin163° cos 347° + sin 73° sin167° is equal to

Office.: 606 , 6 Floor, Hariom Tower, Circular Road, Ranchi-1, Ph.: 0651-2562523, 9835508812, 8507613968
th
6 ( )BY R. K. MALIK’S NEWTON CLASSES
1 1
(a) 0 (b) (c) (d) 1
3 2
96sin 65° sin 35° sin 80°
71. The numerical value of is equal to
sin 20° + 2sin 80° cos 30°
(a) 96 (b) 48 (c) 24 (d) 12
3π 4π
72. The numerical value of cos2 + cos2 is equal to
5 5
3
(a) 0 (b) 1 (c) (d)
2 4
1

2π 8π
73. The numerical value of cos cos cos is equal to
15 15 15
π

1 1 1 1
(a) (b) (c) − (d) −
15 16 15 16
74. The numerical value of sin 20° sin 40° sin 80° is equal to
1 1 2 3
(a) (b) (c) (d)
8 4 5 8
75. Two parallel chords are drawn on the same side of the centre of a circle of radius 12 units. If is found
that they subtend 72° and 144° angles at the centre of the circle. The perpendicular distance in units
between the chords is
(a) 6 (b) 4 (c) 3 (d) 2
76. If in ∆ΑΒC , tan A + tan B + tan C > 0, then
(a) ∆ is always obtuse angled triangle (b) ∆ is always equilateral triangle
(c) ∆ is always acute angled triangle (d) nothing can be said about the type of triangle

77. If α , β , γ , δ are four solutions of the equation tan  θ +  = 3 tan 3θ , then tan α tan β tan γ tan δ
4
 π

equals :

(a) 3 (b) 1 / 3 (c) −1 / 3 (d) none of these


78. The least value of cosec2 x + 25sec2 x is
(a) 0 (b) 26 (c) 28 (d) 36
79. the maximum value of sin ( x + π / 6 ) + cos ( x + π / 6 ) in the interval ( 0, π / 2 ) is attained at x equal
(a) π / 12 (b) π / 6 (c) π / 3 (d) π / 2
80. If sin α = sin β and cos α = cos β , then

(a) sin  =0 (b) cos  =0 (c) sin  =0 (d) cos  =0
 2   2   2   2 
α + β  α + β  α − β  α − β 

2π 3π 4π 5π 6π 7π
81. The value of cos cos cos cos cos cos cos is equal to
15 15 15 15 15 15 15
π

(a) 1 / 2 6 (b) 1 / 2 7 (c) 1 / 28 (d) none of these


82. If 2 cos θ + sin θ = 1, then 7 cos θ + 6sin θ equals
(a) 1 or 2 (b) 2 or 3 (c) 2 or 4 (d) 2 or 6
a cos φ + b
83. If cos θ = , where θ and φ be in first quadrant. Then tan θ / 2 is equal to
a + b cos φ

Office.: 606 , 6 Floor, Hariom Tower, Circular Road, Ranchi-1, Ph.: 0651-2562523, 9835508812, 8507613968
th
( ) BY R. K. MALIK’S NEWTON CLASSES 7

(a)  tan φ / 2 (b)  cos φ / 2 (c)  sin φ / 2 (d) none of these


a−b a+b a−b
  

84. If α , β , γ , δ are the smallest positive angles in ascending order of magnitude which have their sines
a+b a−b a+b

equal to the positive quantity k , then the value of 4 sin + 3sin + 2sin + sin is equal to
2 2 2 2
α β γ δ

(a) 2 1 − k (b) 2 1 + k (c) 2 k (d) 2 k + 2


85. If α = 22°30 ', then (1 + cos α )(1 + cos 3α )(1 + cos 5α ) (1 + cos 7α ) equals

1 1 1+ 2 2 −1
(a) (b) (c) (d)
8 4 2 2 2 +1
1 1
86. A positive acute angle is divided into two parts whose tangents are and . Then the angle is
2 3

(a) (b) (c) (d)


4 5 3 6
π π π π

87. ( sec A + tan A − 1)( sec A − tan A + 1) −2 tan A =


(a) sec A (b) 2 sec A (c) 0 (d) 1
88. If 3sin A + 5cos A = 5, then the value of ( 3cos A − 5sin A) is
2

(a) 4 (b) 5 (c) 2 (d) 9

89. If y = (1 + tan A)(1 − tan B ) where A − B = , then ( y + 1) is equal to


y +1

4
π

(a) 9 (b) 4 (c) 27 (d) 81


cot A cot B
90. If A + B = 225°, then .
1 + cot A 1 + cot B
=

1
(a) 1 (b) −1 (c) 0 (d)
2
91. If cos ( A + B ) = α cos A cos B + β sin A sin B, then (α , β ) =
(a) ( −1, −1) (b) ( −1,1) (c) (1, −1) (d) (1,1)
92. The expression cos2 ( A − B ) + cos2 B − 2 cos ( A − B ) cos A cos B is
(a) Dependent on B (b) Dependent on A and B
(c) Dependent on A (d) Independent of A and B
93. The value of cos12° + cos84° + cos156° + cos132° is
1 1 1
(a) (b) 1 (c) − (d)
2 2 8
3
94. If cos ( A − B ) = and tan A tan B = 2, then
5
1 2 1 1
(a) cos A cos B = (b) sin A sin B = − (c) cos A cos B = − (d) sin A sin B = −
5 5 5 5

95. If cos x + cos y + cos α = 0 and sin x + sin y + sin α = 0, then cot 
 2 
x+ y
=

(a) sin α (b) cos α (c) cot α (d) sin 


 2 
x+ y

Office.: 606 , 6 Floor, Hariom Tower, Circular Road, Ranchi-1, Ph.: 0651-2562523, 9835508812, 8507613968
th
8 (BY R. K. MALIK’S NEWTON CLASSES
)
96. If ABCD is a cyclic quadrilateral such that 12 tan A − 5 = 0 and 5cos B + 3 = 0 , then the quadratic
equation whose roots are cos C and tan D is :
(a) 39 x 2 + 88 x + 48 = 0 (b) 39 x 2 − 16 x − 48 = 0 (c) 39 x 2 − 88 x + 48 = 0 (d) none of these
97. Given that tan A and tan B are the roots of x 2 − px + q = 0 , then the value of sin 2 ( A + B ) is :

(a) (b) (c) (d)


p2 p2 q2 p2
+ (1 − q ) { p − (1 − q ) }
2 2 2
{p 2
} ( p2 + q2 ) ( p + q)

98. If a sin 2 x + b cos 2 x = c , b sin 2 y + a cos 2 y = d and a tan x = b tan y , then is equal to :
a2
b2

(a) (b) (c) (d)


( b − c )( d − b ) ( a − d )( c − a ) ( d − a )( c − a ) ( b − c )( b − d )
( a − d )( d − b ) ( b − c )( d − b ) ( b − c )( d − b ) ( a − c )( a − d )
1 3
99. and cos 2θ + cos 2φ = , then cos 2 (θ − φ ) =
sin 2θ + sin 2φ =
2 2
(a) 3 / 8 (b) 5 / 8 (c) 3 / 4 (d) 5 / 4
1
100. If tan α = and tan β = , then α + β =
m +1 2m + 1
m

(a) (b) (c) (d) none of these


3 4 6
π π π

101. Given that cos   = 2 cos   , then tan tan is equal to :


 2   2  2 2
α − β  α + β  α β

1 1 1 1
(a) (b) (c) (d)
2 3 4 8
102. If A + B + C = π and cos A = cos B cos C , then tan B tan C is equal to :
1 1
(a) (b) 2 (c) 1 (d) −
2 2
103. The value of k , for which ( cos x + sin x ) + k sin x cos x − 1 = 0 is an identity, is :
2

(a) –1 (b) –2 (c) 0 (d) 1


1 13
104. If cos P = and cos Q = , where P and Q both are acute angles. Then the value of P − Q is :
7 14
(a) 30° (b) 60° (c) 45° (d) 75°
105. If cos A = cos B cos C and A + B + C = π , then the value of cot B cot C is :
1 1
(a) 1 (b) 2 (c) (d)
3 2
1
106. If x + = 2 cos θ , then x 6 + x −6 =

(a) 2 cos 6θ (b) 2 cos12θ (c) 2 cos 3θ (d) 2sin 2θ


x

tan 2 α
107. If α ∈  0,  , then 2
(where x is a variable) is always greater than or equal to :
 2
 π
(x + x) +
2
(x + x)

(a) 2 tan α (b) 1 (c) 2 (d) sec2 α



108. If < α < π , then cosec2 α + 2 cot α is equal to :
4
(a) 1 + cot α (b) 1 − cot α (c) −1 − cot α (d) −1 + cot α

Office.: 606 , 6 Floor, Hariom Tower, Circular Road, Ranchi-1, Ph.: 0651-2562523, 9835508812, 8507613968
th
( BY R. K. MALIK’S NEWTON CLASSES
) 9
1 1
109. If x + = 2 cos θ , then x3 + 3 =

1 1
(a) cos 3θ (b) 2 cos 3θ (c) cos 3θ (d) cos 3θ
x x

2 3
110. cos 2 (θ + φ ) + 4 cos (θ + φ ) sin θ sin φ + 2sin 2 φ =
(a) cos 2θ (b) cos 3θ (c) sin 2θ (d) sin 3θ
21 27
111. Let α , β be such that π < (α − β ) < 3π . If sin α + sin β = − and cos α + cos β = − , then the
65 65
value of cos is :
2
α −β

6 3 6 3
(a) − (b) (c) (d) −
65 130 65 130
x −1
112. If θ is an acute angle and sin , then tan θ is equal to :
2 2x
θ
=

(a) x 2 − 1 (b) x2 − 1 (c) x2 + 1 (d) x 2 + 1


113. If α and β are the roots of both cos2 x + p cos x + q = 0 and sin 2 x + r sin x + s = 0, then p, q, r and s
are related by
(a) p 2 + q 2 = 2 ( q + s + 1) (b) p 2 + r 2 = q 2 + p 2
(c) p 2 + r 2 = 2 ( q + s + 1) (d) p 2 + q 2 − 2 = 2 ( r + s )
114. If for any real value of θ , a = cos 2 θ + sin 4 θ , then :
1 1 3
(a) −1 ≤ a ≤ 1 (b) − ≤ a ≤ (c) ≤ a ≤1 (d) none of these
8 8 4

115. For θ ∈  0,  , the expression ( x 2 + 4 xy + y 2 ) is reduced into the form ax ' 2 + by '2 due to the
 2
 π

transformation x = x 'cos θ + y 'sin θ , y = x 'sin θ − y 'cos θ . Then

(a) a = 3, b = 1, θ = (b) a = 1, b = 1, θ = (c) a = 3, b = −1, θ = (d) a = 3, b = −1, θ =


4 2 4 3
π π π π

116. In a triangle ABC , tan tan + tan + tan + tan tan is equal to
2 2 2 2 2 2
A B B C C A

1 1
(a) −1 (b) (c) (d) none of these
2 3
tan (α − β )
117. If sin 2α = 4 sin 2 β then = k , where k is equal to
tan (α + β )
3 5 1 3
(a) (b) (c) (d)
5 3 2 2
1 3π 1 5π 9π 12π
118. The numerical value of cos + cos − cos cos = k . then the reciprocal of k is
2 13 2 13 13 13
1
(a) 0 (b) 1 (c) (d) none of these
2
119. If cot x cos 2 x − tan x sin 2 x = k cot 2 x, then k is equal to
1 1
(a) (b) 2 (c) (d) 3
2 3

Office.: 606 , 6 Floor, Hariom Tower, Circular Road, Ranchi-1, Ph.: 0651-2562523, 9835508812, 8507613968
th
10 ( )BY R. K. MALIK’S NEWTON CLASSES
120. If 3 + 4 cos 2θ + cos 4θ = k cos2 θ , then the numerical value of k is equal to
(a) 4 (b) 6 (c) 8 (d) 10
3π 5π 7π
121. The numerical value of sin 4 + sin 4 + sin 4 + sin 4 is equal to
8 8 8 8
π

1 3
(a) 0 (b) 1 (c) (d)
2 2
3π 3π
122. If cos α + cos β = cos and sin α + sin β = sin , then the value of cos2 is given by
7 7 2
α −β

1 1 1
(a) (b) (c) (d) 1
8 4 2

123. If m = k cos3 θ + 3k cos θ .sin 2 θ and n = k sin 3 θ + 3k cos 2 θ sin θ , then ( m + n ) 3 + ( m − n ) 3 is equal to
2 2

(a) k (b) k (c) 2k (d) 2k


2 3 2 3
3 2 3 2

124. If x = tan15°, then


(a) x 2 + 2 3 x + 1 = 0 (b) x 2 + 2 3 x − 1 = 0 (c) x 2 − 2 3 x + 1 = 0 (d) x 2 − 2 3 x − 1 = 0
99π
125. If x = sin θ sin θ , y = cos θ cos θ , where ≤ θ ≤ 50π , then
2
(a) x − y = 1 (b) x + y = −1 (c) x + y = 1 (d) y − x = 1
cos x cos ( x + θ ) cos ( x + 2θ ) cos ( x + 3θ )
126. If , then is equal to
a+c
= = =

(a) a / d (b) c / d (c) b / c (d) d / a


a b c d b+d

2π 4π
127. If x = y cos = z cos , then xy + yz + zx is equal to
3 3
(a) −1 (b) 0 (c) 1 (d) 2
128. If 0 < α < π / 6 and sin α + cos α = 7 / 2, then tan α / 2 is equal to
7 −2 7 +2 7
(a) (b) (c) (d) none of these
3 3 3

129. If , then x + y + z is equal to


cos α 2π  2π 
cos  α −  cos  α +
x y z
= =

3  3 
 

(a) 1 (b) 0 (c) −1 (d) none of these


 

130. If cos α + cos β = sin α + sin β , then cos 2α + cos 2 β is equal to


(a) −2sin (α + β ) (b) −2 cos (α + β ) (c) 2sin (α + β ) (d) 2 cos (α + β )
131. If 4nα = π , then the numerical value of tan α tan 2α tan 3α .. tan ( 2n − 1) α is equal to
(a) −1 (b) 0 (c) 1 (d) 2

132. If cos x + sin x = a  − < x < −  , then cos 2x is equal to


 2 4
 π π

(a) a 2 (b) a (2 − a) (c) a (2 + a) (d) a (2 − a )


2

133. If tan x tan y = a and x + y = , then tan x and tan y satisfy the equation
6
π

(a) x 2 − 3 (1 − a ) x + a = 0 (b) 3 x 2 − (1 − a ) x + a 3 = 0

Office.: 606 , 6 Floor, Hariom Tower, Circular Road, Ranchi-1, Ph.: 0651-2562523, 9835508812, 8507613968
th
( ) BY R. K. MALIK’S NEWTON CLASSES 11

(c) x 2 + 3 (1 + a ) x − a = 0 (d) 3 x 2 + (1 + a ) x − a 3 = 0

134. If tan θ − cot θ = a and sin θ + cos θ = b, then ( b 2 − 1) ( a 2 + 4 ) is equal to


2

(a) 2 (b) −4 (c) ±4 (d) 4


135. If sin A + sin B = C , cos A + cos B = D, then the value of sin ( A + B ) =
2CD
(a) CD (b) (c) (d) 2
C 2 + D2
2CD
CD
2 2
C + D2
136. If sin θ + sin 2θ + sin 3θ = sin α and cos θ + cos 2θ + cos 3θ = cos α , then θ is equal to
C +D

(a) (b) α (c) 2α (d)


2 6
α α

137. If sin θ + cos θ = p , then sin θ + cos θ is equal to:


4 4

1 1 1 1
2

(a) − (2 p 2 − 1) (b) 1 − ( p 2 − 1) 2 (c) 1 −  p 2 − 1 (d) − ( p 2 − 1)2 .


2 2 2 2

138. If sin θ + cosec θ = 2, then sin n θ + cosec n θ is equal to:


(a) 2 (b) n 2 (c) 2n −1 (d) 2n .

139. The expression tan  +  is equal to: where A ∈  0, 


4 2  2
π A  π

1 − cos A 1 − sin A 1 + cos A 1 + sin A


(a) (b) (c) (d) .
1 + cos A 1 + sin A 1 − cos A 1 − sin A
140. The value of expression ( 3 sin 75° − cos 75°) is:
1
(a) (b) 1 (c) 2 (d) 2 2 .
2

141. If α + β = , then 1 + tan β is equal to:


4
π

1 2 1 1 − tan α
(a) (b) (c) (d) .
1 − tan α 2 + tan α 2 + tan α 1 + tan α

142. If tan  + θ  + tan  − θ  = p sec 2θ , then the value of p is equal to:


4 4
π  π 

(a) 1 (b) 2 (c) 3 (d) 4.


 

143. If 270° < θ < 360°, then the sign of sin is: + cos
2 2
θ θ

(a) positive (b) negative


(c) sometimes positive and sometimes negative (d) cannot be determined.
144. If (1 + sin α ) (1 + sin β ) (1 + sin γ ) = (1 − sin α ) (1 − sin β ) (1 − sin γ )
(sec γ − tan γ ) is equal to:
(a) − sin α sin β sin γ (b) ± cos α cos β cos γ (c) ±2sin α sin β sin γ (d) 2 cos α cos β cos γ .
145. cos α sin( β − γ ) + cos β sin(γ − α ) + cos γ sin(α − β ) is equal to:
1
(a) 0 (b) (c) 1 (d) 4 cos α cos β cos γ .
2
146. If tan θ + sin θ = α and tan θ − sin θ = β , then α 2 − β 2 is equal to:

Office.: 606 , 6 Floor, Hariom Tower, Circular Road, Ranchi-1, Ph.: 0651-2562523, 9835508812, 8507613968
th
12 ( ) BY R. K. MALIK’S NEWTON CLASSES

(a) ± α 2 + β 2 (b) ±4 αβ (c) ± (d) ± .


α2 + β2
2
αβ

147. If sin θ , cos θ and tan θ are in geometric progression, then cot 6 θ − cot 2 θ is equal to:
αβ

1
(a) (b) 1 (c) 2 (d) 3.
2
148. If cos(θ + φ ) = m cos(θ − φ ), the tan θ is equal to:
(1 − m) (1 − m) (1 + m) (1 + m)
(a) tan φ (b) cot φ (c) sec φ (d) tan φ .
(1 + m) (1 + m) (1 − m) (1 − m)

149. If ( A + B ) = and cos A + cos B = 1 , then cos( A − B ) is equal to:


3
π

1 3 2 1
(a) − (b) − (c) (d) .
3 2 3 2 3
150. If cot(α + β ) = 0, then sin(α + 2 β ) is equal to:
(a) sin α (b) sin β (c) sin 2α (d) cos 2 β .
151. The maximum value of 12 sin θ − 9sin 2 θ is equal to:
(a) 3 (b) 4 (c) 5 (d) None of these.
 A 1 B 2
152. Let A, B and C be the angles of triangle and tan   = , tan = . Then, tan   is equal to:
2 3 2 3 2
C 

2 1 7 2
(a) (b) (c) (d) .
9 3 9 3
153. tan 5 x − tan 3 x − tan 2 x is equal to:
sin 5 x − sin 3 x − sin 2 x sin 5 x − sin 3 x − sin 2 x
(a) 0 (b) (c) tan 2 x tan 3 x tan 5 x (d) .
cos x cot 5 x
cos 2 θ cos θ ⋅ sin θ − sin θ
154. f (θ ) = cos θ .sin θ sin θ
2
cos θ , then for all θ :
sin θ − cos θ 0
(a) f (θ ) = 0 (b) f (θ ) = 1 (c) f (θ ) = 2 (d) None of these.
155. If α lies in the third quadrant, then the value of the expression
 π − 2α 
4 cos 2   + 4 sin α + sin 2α is:
4 2

4
3 3
 

(a) 2 (b) (c) 2 + 4sin α (d) + 4sin α .


4 2
156. Evaluate cos 2(θ + φ ) + 4 cos(θ + φ ) sin θ sin φ + 2sin 2 φ :
(a) sin 2θ (b) cos 2θ (c) sin 3θ (d) cos 3θ .
cos u − e
157. If cos θ = , then tan is equal to:
1 − e cos u 2
θ

e −1 e +1 1+ e e −1
(a) ± tan (b) ± tan u (c) ± tan (d) ± tan u .
e +1 2 e −1 1− e 2 e +1
u u

2sin x 1 + sin x − cos x


158. If θ = , then equals:
1 + sin x + cos x 1 + sin x

Office.: 606 , 6 Floor, Hariom Tower, Circular Road, Ranchi-1, Ph.: 0651-2562523, 9835508812, 8507613968
th
( ) BY R. K. MALIK’S NEWTON CLASSES 13

(a) − θ (b) − (c)0 (d) θ .


2
θ

159. If tn = sin n θ + cos n θ and tn − tn − 2 = ktn − 4 , then k is equal to:


(a) − sin θ cos θ (b) sin 2 θ (c) − sin 2 θ cos 2 θ (d) cot 2 θ .
160. The expression 4(cos8 θ − sin 8 θ ) is equal to:
(a) cos 2θ .cos 4θ − 4 (b) cos 4θ (cos 2θ − 2) (c) cos 2θ (cos 4θ + 3) (d) 2 cos 4θ (cos 2θ + 1) .
161. If (1 + cos α )(1 + cos β )(1 + cos γ ) = λ = (1 − cos α )(1 − cos β ) (1 − cos γ ), then | λ | is equal to:
(a) | sin(α + β + γ ) | (b) | cos 2α cos 2 β cos 2γ | (c) | sin α sin β sin γ | (d)| cos 2α + cos 2 β + cos 2γ | .
162. The expression (1 − tan 2 θ ) (1 − 3 tan 2 θ ) (1 + tan 2θ tan 3θ ) is equal to:
(a) cosec 2 θ cot 2 θ (b) sec 2 θ tan 2 θ (c) sec4 θ (d) 6 tan 4 θ − 1 .

163. If sin θ + sin φ = a and cos θ + cos φ = b, then tan   is:


 2 
 α −φ 

4 − a2 − b2 4 + a2 + b2
(a) (b) (c) (d) .
a 2 + b2 a2 + b2
4 + a2 + b2 a2 + b2 a 2 + b2 4 − a2 − b2
164. If sin α + sin β = a, cos α + cos β = b, then sin(α + β ) is equal to:
2ab 2ab
(a) (b) (c) (d) .
2(a 2 + b 2 )
ab a+b
a + b2
2 2
a + b2
2

3cos 2 β − 1
a − b2

165. If cos 2α = , then tan α is equal to:


3 − cos 2 β
tan β tan 2 β
(a) ± (b) ± tan β (c) ± (d) ± 2 tan β .
2 2
166. cos 20° + cos100° + cos140° is equal to:
1 1
(a) – 1 (b) 0 (c) (d) .
2 8
 1 
167. tan  7 °  is equal to:
 2 
1+ 3 1 2 2 − (1 + 3)
(a) (b) + 3 (c) 2 2 + 3 (d) .
1− 3 3 3 −1
2π 3π 4π
168. Evaluate cos cos cos cos :
9 9 9 9
π

1 1 1 1
(a) − (b) (c) (d) − .
8 4 16 2
169. The value of 6(sin 6 θ + cos 6 θ ) − 9(sin 4 θ + cos 4 θ ) + 4 is:
(a) – 3 (b) 0 (c) 1 (d) 3.
170. If Pn = cos α + sin θ , then 2 P6 − 3P4 + 1 is equal to:
(a) 0 (b) 1 (c) 2 (d) 3.
n n

sin( A − B ) sin( B − C ) sin(C − A)


171. In a triangle ABC , is:
sin A sin B sin B sin C sin C sin A
+ +

(a) – 3 (b) – 1 (c) 0 (d) 1.

Office.: 606 , 6 Floor, Hariom Tower, Circular Road, Ranchi-1, Ph.: 0651-2562523, 9835508812, 8507613968
th
14 ( ) BY R. K. MALIK’S NEWTON CLASSES
1 3
172. If sin a + sin b = and cos a + cos b = for any two real numbers a and b, then sin(a + b) is equal
2 2
to:
1 1 3 1
(a) (b) (c) (d) .
2 3 2 2 3
173. If sin A + sin B = l , cos A − cos B = m, then the value of cos( A − B ) is:
2lm 2lm
(a) (b) (c) (d) .
l 2 − m2 l 2 + m2
l 2 + m2 l 2 − m2 l 2 + m2 l 2 − m2
174. If sin θ = sin15° + sin 45°. where 0° < θ < 90° , then θ is equal to:
(a) 75° (b) 60° (c) 30° (d) none of these
 3 + 2 cos A   1 + 2sin A 
−3 −3

175.  is equal to:


 1 − 2 sin A   3 − 2 cos A 
 +
 

1 2
(a) − (b) − (c) 0 (d) 3.
2 3
176. If x cos θ − y sin θ = A, x sin θ + y cos θ = B, then x 2 + y 2 is equal to:
(a) A2 − B 2 (b) 0 (c) 1 (d) A2 + B 2 .
2π 3π 4π 5π 6π 7π
177. cos .cos .cos .cos .cos .cos .cos is equal to:
15 15 15 15 15 15 15
π

1 1 7π
(a) 5 .sin (b) 7 (c) 5 .cos (d) .
2 15 2 2 15 2 .32
π π π
3

178. If α , β , γ are the angles of a triangle, then sin 2 α + sin 2 β + sin 2 γ − 2 cos α cos β cos γ is equal to:
(a) – 2 (b) – 1 (c) 0 (d) 2.
179. If sin θ and cos θ are the roots of the equation ax 2 − bx + c = 0, then:
(a) a 2 − b 2 − 2ac = 0 (b) a 2 − b 2 + 2ac = 0 (c) a 2 + b 2 − 2ac = 0 (d) a 2 + b 2 + 2ac = 0 .
180. For all values of α + β + γ the expression
cos α + cos β + cos γ + cos(α + β + γ ) is equal to:

(a) 4 cos   cos   cos  (b) 4 sin   cos   sin 


 2   2   2   2   2   2 
α − β   β −γ   γ −α  α − β   β − γ   γ −α 
 

(c) 4 cos   cos   cos  (d) 4 sin   sin   sin  .


 2  2  2   2   2   2 
α + β   β +γ   γ +α  α + β   β +γ   γ +α 

181. cosec10° − 3 sec10° is equal to:


 

(a) 0 (b) 2 (c) 4 (d) 8.


182. If A + B = C , then cos A + cos B + cos C − 2 cos A cos B cos C is:
2 2 2

(a) 0 (b) 1 (c) 2 (d) 3.


183. If x = tan 6° tan 42° and y = cot 66° cot 78°, then x : y is equal to:
(a) 1 : 1 (b) 1 : 2 (c) 2 : 3 (d) 3 : 2.
184. If, in ∆ ABC , a = 13 cm, b = 12 cm, c = 5 cm, then distance from A to side BC is
(a) 144 /13 (b) 65 /12 (c) 60 /13 (d) 25 /13
m − n tan θ
185. If m sin θ + n cos θ = p, then is equal to:
n + m tan θ

Office.: 606 , 6 Floor, Hariom Tower, Circular Road, Ranchi-1, Ph.: 0651-2562523, 9835508812, 8507613968
th
( ) BY R. K. MALIK’S NEWTON CLASSES 15

p 2 − ( m − n) 2
(a) ± (b) ± (c) ± (d) ± .
m2 + n2 + p 2 m2 + n2 + p2 m2 + n2 − p 2
p ( m + n) mn p

186. If a = radians, then cos a + cos 2a + ............... + cos18a is equal to:


18
π

(a) – 3 (b) – 1 (c) 0 (d) 1.

187. In a triangle PQR, ∠R = . If tan   and tan   are the roots of the equation
2 2 2
π P Q

ax 2 + bx + c = 0 (a ≠ 0), then:
(a) b + c = a (b) a + c = b (c) a + b = c (d) b = c .
188. If α + β − γ = π , then sin α + sin β − sin γ is equal to:
2 2 2

(a) 2 cos α cos β sin γ (b) 2 cos(α + β − γ ) (c) 2 sin  (d) 2sin α sin β cos γ .
2
α + β −γ 

3
 

189. If cos( A − B ) = and tan A tan B = 2, then which of the following is true?
5
1 1 1 1
(a) sin( A − B ) = (b) sin( A + B ) = − (c) cos( A + B ) = − (d) sin A − sin B = − .
5 5 5 5
190. If tan 2 A = 2 tan 2 B + 1. then cos 2 A + sin 2 B equals:
(a) – 1 (b) 0 (c) 1 (d) 2.
191. The value of sin16° + cos16° is:
1 1
(a) ( 2 cos1° + sin1°) (b) (cos1° + 3 sin1°)
3 2
1 1
(c) (cos1° + 2 sin1°) (d) ( 3 cos1° + sin1°) .
3 2

192. If tan x tan y = a and x + y = , then tan x and tan y satisfy the equation:
6
π

(a) 3 x 2 + (1 + a ) x − a 3 = 0 (b) x 2 + 3(1 − a ) x − a = 0


(c) 3 x 2 − (1 − a ) x + a 3 = 0 (d) x 2 − 3 (1 − a ) x + a = 0 .
193. For all values of α , β , γ , the expression sin α + sin β + sin γ − sin(α + β + γ ) is equal to:

(a) −4sin   sin   sin  (b) −4 cos   cos   cos 


 2   2   2   2  2  2 
α − β   β −γ   γ −α  α − β   β −γ   γ −α 
 
 

(c) 4 sin   sin   sin  (d) 4 cos   cos   cos  .


 2   2   2   2   2   2 
 α + β   β + γ   γ +α  α + β   β +γ   γ +α 

194. If α = sin 2 θ + cos 4 θ , then for all real values of θ :


3 3 4 4
(a) ≤ α ≤1 (b) (c) 1 ≤ α ≤ 2 (d) ≤α ≤ 2.
4 4 3 3
≤α ≤

cos α − cos β
195. If cos θ = , then tan   is equal to:
1 − cos α cos β 2
θ 

(a) ± tan   tan   (b) ± tan   cot   (c) ± cot   tan   (d) ± cot   cot   .
2 2 2 2 2 2 2 2
α   β  α   β  α   β  α   β 

 2π  4π  8π   14π 
196. The value of cos   cos   cos   cos   is:
 15  15  15   15 
 
 

Office.: 606 , 6 Floor, Hariom Tower, Circular Road, Ranchi-1, Ph.: 0651-2562523, 9835508812, 8507613968
th
16 ( ) BY R. K. MALIK’S NEWTON CLASSES
1 1 1 1
(a) (b) (c) (d) .
4 8 12 16
197. If α cos 2 3θ + β cos 4 θ = 16 cos 6 θ + 9 cos 2 θ is an identity, then:
(a) α = 4, β = 2 (b) α = 1, β = 18 (c) α = 1, β = 24 (d) α = 3, β = 24 .
1
198. Evaluate tan 82 ° :
2
(a) ( 3 − 2) ( 2 − 1) (b) ( 3 − 2) ( 2 + 1) (c) ( 3 + 2) ( 2 − 1) (d) ( 3 + 2) ( 2 + 1) .
199. cosec 15° + sec15° is equal to:
(a) 6 (b) 2 6 (c) 2 2 (d) 6+ 2.
cos θ sin θ
200. If , then is:
sec 2θ cosec 2θ
α β
= +

(a) α (b) β + 2α (c) αβ (d) α ( β + 2) .


α β

−21 −27
211. Let α , β such that π < α − β < 3π . If sin α + sin β = and cos α + cos β = , then the value of
65 65
cos   is:
 2 
α − β 

−6 −3 3 6
(a) (b) (c) (d) .
65 130 65 130
212. The maximum value of the expression
(4 cos x + 3sin x).(3cos x − 4sin x) is:
7 25
(a) (b) (c) 50 (d) 24.
2 2
3π 5π 7π 9π
213. Evaluate cos + cos + cos + cos + cos :
11 11 11 11 11
π

3 1 1 1
(a) − (b) − (c) (d) .
2 2 2 2
15π 3π
214. The value of the expression sin .sin .sin is:
32 16 8

1 1 1
(a) (b) cosec (c) (d) cosec .
8 32 4 16
1 π π
8 2 sin 4 2 sin
32 16
π π

215. cot( A + 15°) − tan( A − 15°) is equal to:


cos 2 A 4 cos A 4 cos 2 A 2 cos 2 A
(a) (b) (c) (d) .
1 − sin 2 A 2sin 2 A + 1 1 − sin 2 A sin A − 1
 π   5π   7π 
216. The value of the expression sin   sin   sin   is:
 18   18   18 
1 1 1 1
(a) (b) (c) (d) .
16 8 4 2

217. Let 0 ≤ θ ≤ and x = X cos θ + Y sin θ , y = X sin θ − Y cos θ such that x 2 + 4 xy + y 2 = aX 2 + bY 2 ,


2
π

where a,b are constants, then:

(a) θ = (b) θ = (c) a = 3, b = −1 (d) a = −1, b = 3 .


3 6
π π

Office.: 606 , 6 Floor, Hariom Tower, Circular Road, Ranchi-1, Ph.: 0651-2562523, 9835508812, 8507613968
th
( BY R. K. MALIK’S NEWTON CLASSES
) 17
5π 7π
218. The value of sin .sin .sin is:
18 18 18
π

1 3 1 3
(a) (b) (c) (d) .
4 8 8 4
1 − 4sin10° sin 70°
219. The value of the expression is:
2sin10°
1
(a) (b) 1 (c) 2 (d) None of these.
2
sin 2 3θ cos 2 3θ
220. The expression is equal to:
sin 2 θ cos 2 θ

(a) 2sin θ (b) 8 cos 2θ (c) 4 sin 3θ (d) 3cos 4θ .


221. sin 47° − sin 25° + sin 61° − sin11° is equal to:
(a) sin 7° (b) cos 7° (c) 2sin 7° (d) 2 cos 7° .
222. Let tr denote the rth term of an A.P. and let the sum of the first 10 terms be equal to 11 and also
t55 = 11 . If t5 = tan A, t10 = tan B and t15 = tan C , then A + B + C is equal to:
2π 3π
(a) (b) (c) π (d) .
2 3 2
π

5 1 1
223. If cos(α + β ) = sin(α − β ) = and α , β lie between 0 and , then tan 2α is equal to:
52 5 13 4
π

16 60 52 56
(a) (b) (c) (d)
65 61 42 33
2π 4π
224. cos cos cos is equal to:
7 7 7
π

1 1 1
(a) − (b) 0 (c) (d) .
8 4 2
225. If sin 6θ = 32 cos5 θ sin θ − 32 cos3 θ sin θ + 3 x, then x is equal to:
(a) sin θ (b) cos θ (c) sin 2θ (d) cos 2θ .
226. If cos α + cos β = 0 = sin α + sin β , then cos 2α + cos 2 β is equal to:
(a) −2sin(α + β ) (b) −2 cos(α + β ) (c) 2sin(α + β ) (d) 2 cos(α + β ) .
sec 2 θ − tan θ
227. the value of the expression always lies in the interval:
sec 2 θ + tan θ
1  1 1 1   1 
(a)  −3, −  (b)  − ,  (c)  3 , 3 (d)  − ,1 .
3  3 3  3 

228. If x = tan15°, y = cosec 75° and z = 4 sin18°, then:


(a) x < y < z (b) x < z < y (c) z < x < y (d) y < z < x .
(a) – 1 (b) 0 (c) 1 (d) 2.
2π 4π 8π
230. The value of cos cos cos cos is:
15 15 15 15
π

1 1
(a) − (b) 0 (c) (d) 1.
16 16
sin A cos A
231. If = m and = n, then tan A is equal to:
sin B cos B

Office.: 606 , 6 Floor, Hariom Tower, Circular Road, Ranchi-1, Ph.: 0651-2562523, 9835508812, 8507613968
th
18 ( ) BY R. K. MALIK’S NEWTON CLASSES
m −1 m 1 − n2 n 1 − m2 n2 −1
(a) ± (b) ± (c) ± (d) ± .
2

n2 −1 n m2 − 1 m n2 − 1 1 − m2
 1− x  cos θ − x
232. If tan  tan , then the value of is:
2  1+ x  2 1 − x cos θ
θ φ
= 

(a) sin φ (b) cos φ (c) sin φ (d) cos x .


233. cos 2 76° + cos 2 16° − cos 76° cos16° is equal to:
1 1 3
(a) − (b) 0 (c) (d) .
4 2 4
2π 3π
234. Evaluate sin + sin + sin :
7 7 7
π

1 4π 1 1 2π 1
(a) cosec (b) tan (c) cos (d) cot .
2 7 2 14 2 7 2 14
π π

235. cos12° + cos84° + cos132° + cos156° is equal to:


1 1 3
(a) − (b) − (c) 0 (d) .
4 2 4
236. If the roots of the quadratic equation x 2 + px + q = 0 are tan 30° and tan15°, respectively then the
value of 2 + q − p is:
(a) 0 (b) 1 (c) 2 (d) 3.
237. If an angle α is divided into two parts A and B such that A − B = x and tan A : tan B = k :1 , then the
value of sin x is:
k −1 k +1
(a) sin α (b) sin α (c) sin α (d) None of these.
k +1 k +1 k −1
k

tan 3 A sin 3 A
238. If = α , then is equal to:
tan A sin A
2α 2α
(a) (b) (c) (d) .
α +1 α −1 α +1 α −1
α α

7
239. If sin α + cos α = , 0 < α < , then tan is:
2 6 2
π α

2 2 1 1
(a) (2 − 7) (b) ( 7 − 2) (c) ( 7 − 2) (d) (2 − 7) .
3 3 3 3
2b
240. If tan x = , y = a cos 2 x + 2b sin x cos x + c sin 2 x and z = a sin 2 x − 2b sin x cos x + c cos 2 x, then
which option is correct?

a c

(a) y − z = a + c (b) y + z = a + c (c) y = z (d) y − z = (a − c) 2 + 4b 2 .


3 + cot 76° cot16°
241. The expression is equal to:
cot 76° + cot16°
(a) tan 44° (b) cot 44° (c) tan 32° (d) cot 32° .
242. If 5cos x + 12 cos y = 13, then the maximum value of
5sin x + 12sin y is:
(a) 65 (b) 120 (c) 156 (d) 14.
243. If α sin θ + 3α cos θ sin θ = x and α cos θ + 3α cos θ sin θ = y, then ( x + y ) 2/3 + ( x − y ) 2/3 is equal to:
3 2 3 2

Office.: 606 , 6 Floor, Hariom Tower, Circular Road, Ranchi-1, Ph.: 0651-2562523, 9835508812, 8507613968
th
( ) BY R. K. MALIK’S NEWTON CLASSES 19

(a) 1 (b) 2α (c) (d) 2α 2/3 .


α2
3
244. If α + β − γ = π , then sin α + sin β − sin γ is equal to:
2 2 2

(a) 2 sin α sin β sin γ (b) 2sin α sin β cos γ (c) 2 cos α cos β sin γ (d) 2 cos α cos β cos γ .
245. If tan A and tan B are the roos of the quadratic equation x 2 + mx + n = 0, m ≠ 0, then the incorrect
statement is:
n −1
(a) tan( A + B ) = (b) cos( A + B ) = ±
(n − 1)
m
m2 + n 2
(c) sin 2 ( A + B ) + m sin( A + B ) cos( A + B ) + n cos 2 ( A + B ) = n
(d) None of these.
246. If sin A = 3sin( A + 2 B ), then tan( A + B ) is equal to:
(a) −2sin 2 B (b) −2 cos B (c) −2 tan B (d) −2 cosec2 B .
247. If α and β are the solutions of the equation m sin θ + n cos θ = p , then sin α + sin β is equal to:
2p 2(m 2 + n 2 ) 2mp 2(m 2 + n 2 )
(a) (b) (c) (d) .
m + n2
2
p2 m2 + n2 p 2 − m2 − n 2
248. If sin A + cos B = a and sin B + cos A = b, then sin( A + B ) is equal to:
a2 − b2 + 2 a2 + b2 − 2
(a) (b) (c) (d) None of these.
a2 + b2
2 2 2
249. If α and β are the solutions of the equation m sin θ + n cos θ = p, then cos α cos β is equal to:
2p (m 2 + n 2 ) 2m 2 n 2
(a) (b) (c) (d) .
p 2 − m2
m + n2
2
p2 m2 + n2 p 2 − m2 − n 2
2π  4π  1 1 1
250. If x cos θ = y cos  θ +  = z cos  θ +  then the value of + + is equal to:
3  3 
 

(a) 0 (b) 1 (c) 2 (d) 3cos θ .


  x y z

251. If cos θ = cos α cos β , then tan   .tan   is equal to:


 2   2 
θ +α   α −θ 

(a) tan tan (b) tan 2 (c) tan 2 (d) cot tan .
2 2 2 2 2 2
α β α β α β

252. The expression tan θ + tan  + θ  − tan  − θ  is equal to:


3 3
π  π 

1 3 1
 

(a) cot 3θ (b) cot θ (c) 3 tan 3θ (d) tan 3θ .


3 2 4
253. If sin θ + cos θ = h then the quadratic equation having sin θ and cos θ as its roots is:
(a) x 2 − hx + (h 2 − 1) = 0 (b) x 2 − hx + 2(h 2 − 1) = 0
(c) x 2 − 2hx + (h 2 − 1) = 0 (d) 2 x 2 − 2hx + (h 2 − 1) = 0 .
α cos u + β
254. If cos θ = , then tan   is equal to:
α + β cos u 2
θ 

(a) ± α 2 − β 2 tan (b) tan (c) ± tan (d) ± tan .


± α2 − β2 α2 − β2
2 4 2 2 2
u u α −β u u

255. The sum S = sin θ + sin 2θ + ..... + sin nθ equals:


α +β αβ

Office.: 606 , 6 Floor, Hariom Tower, Circular Road, Ranchi-1, Ph.: 0651-2562523, 9835508812, 8507613968
th
20 ( BY R. K. MALIK’S NEWTON CLASSES
)
(n + 1)θ (n + 1)θ (n + 1)θ (n + 1)θ
sin sin sin cos cos sin cos cos
(a) 2 2 (b) 2 2 (c) 2 2 (d) 2 2 .
nθ nθ nθ nθ

sin sin sin sin


2 2 2 2
θ θ θ θ

1 1
256. If x + = 2 cos θ , then x3 + 3 is equal to:

(a) sin 3θ (b) cos 3θ (c) 2 sin 3θ (d) 2 cos 3θ .


x x

257. If α , β , α − β ≠ 2nπ satisfy the equation a cos θ + b sin θ = c , then the value of tan   is:
 2 
α + β 

(a) (b) (c) (d) .


a c b c

258. Since tan A and tan B are the roots of x 2 − ax + b = 0 . The value of sin 2 ( A + B ) is :
b b a a

(a) (b) (c) (d) .


a2 b2 a2 b2
a 2 + (1 − b) 2 a 2 + (1 − b) 2 b 2 + (1 − a) 2 a 2 + (1 − a) 2
259. In a triangle ABC, sin A − cos B = cos C , then the value of angle B is: (given that B > C )

(a) (b) (c) (d) .


6 4 3 2
π π π π

260. Maximum and minimum values of sin 2 (120° + θ ) + sin 2 (120° − θ ) are:
1 3 1 3 1 3
(a) min = 0, max = (b) min = 0, max = (c) min = , max = (d) min = , max = .
2 4 2 2 4 2
1 3
261. if sin 2θ + sin 2φ = and cos 2θ + cos 2φ = , then the value of cos 2 (θ − φ ) is:
2 2
3 5 3 5
(a) (b) (c) (d) .
8 8 4 4
sin A cos A
262. If = p, = q, then:
sin B cos B
− p q2 −1 p q2 −1 q2 −1
(a) tan A = (b) tan (c) tan B = (d) All are correct.
q 1 − p2 q 1 − p2 1 − p2
A =

263. If cos x = 3cos y, then 2 tan   is equal to:


 2 
 y−x

(a) cot  (b) cot  (c) cot  (d) cot  .


 4   4   2   2 
 y−x  x+ y  y−x  x+ y
  

264. The value of sin 2 5° + sin 2 10° + sin 2 15° + ...... + sin 2 90° is:
(a) 8 (b) 9 (c) 9.5 (d) 10.
265. If f (θ ) = 3sin θ − 6sin θ cos θ + cos θ + 2, the the range of g (θ ) = [ f (θ )] where [ ] denotes the
2 2

greatest integer function, is:


(a) {1, 2,3, 4, 5, 6, 7} (b) {+2, ± 3, ± 4} (c) {3, 4, 5} (d) {−7, −5, −3, −1, 0, 2, 4, 6} .
1
266. Suppose 0 < t < π and sin t + cos t = . Then, tan   is equal to:
5 2
t

1 1
(a) (b) 2 (c) (d) none of these
3 5
267. cos(α + β + γ ) + cos(α − β − γ ) + cos( β − γ − α ) + cos(γ − α − β ) is equal to:

Office.: 606 , 6 Floor, Hariom Tower, Circular Road, Ranchi-1, Ph.: 0651-2562523, 9835508812, 8507613968
th
( BY R. K. MALIK’S NEWTON CLASSES
) 21
(a) 2 cos α cos β cos γ (b) 3cos α cos β cos γ (c) 4 cos α cos β cos γ (d) 6 cos α cos β cos γ .
3π 5π 7π
268. cos 2 + cos 2
+ cos 2 + cos 2 is equal to:
16 16 16 16
π

(a) 0 (b) 1 (c) 2 (d) 3.


sin( x + y ) (a + b) tan x
269. If then is equal to:
sin( x − y ) (a − b) tan y
=

(a) (b) (c) (d) .


a a2 a 2 − b2 a2 + b2
b2 a2 + b2 a2 − b2
4 − p2 − q2
b

270. If cos α + cos β = p and sin α + sin β = q, then the value of equals:
p2 + q2

(a) sin 2  (b) cos 2  (c) tan 2  (d) cot 2  .


 2   2   2   2
α + β  α − β  α − β  α + β 
  

3π   5π  7π 

271. The value of 1 + cos  1 + cos  1 + cos 1 + cos  is equal to


8  8  8  8 
 π 

1 1 1 1

(a) (b) (c) (d)


4 6 8 2
272. sin 2 > sin 3 T/F
273. The value of tan 81° − tan 63° − tan 27° + tan 9° is
(a) 1 (b) 2 (c) 3 (d) 4
274. If sin θ1 − sin θ 2 = a and cos θ1 + cos θ 2 = b, then

(a) a 2 + b 2 ≥ 4 (b) a 2 + b 2 ≤ 4 (c) a 2 + b 2 ≥ 3 (d) a 2 + b 2 ≤ 2

275. If tan   , tan   are roots of 8 x 2 − 26 x + 15 = 0, then cos (α + β ) =


2 2
α  β 

627 627
(a) − (b) (c) −1 (d) None of these
725 675
276. If a cos θ + b sin θ = c then ( a sin θ − b cos θ ) is
2

(a) a 2 + b 2 + c 2 (b) a 2 + b 2 − c 2 (c) a 2 − b 2 − c 2 (d) b 2 − c 2 − a1


277. A man, jogging along a circular track at the speed of 11 km/hr, traverses in 36 seconds an arc of 56°,
The diameter of the track in metres [Given: π = 22 / 7 ] is ….
(a) 228 m (b) 225 m (c) 220 m (d) none of these
278. ∆ABC is equilateral with each side 18 cm. A circle is drawn on side BC as a diameter. The length of
the arc of this circle intercepted within the triangle is ….
(a) π (b) 2π (c) 3π (d) none of these
279. An arc of 30° in one circle is double an arc in a second circle. If the radius of the second circle is three
times the radius of the first circle, The measure of the angle subtended by the second arc at the centre
in degrees is …..
(a) 5° (b) 10° (c) 15° (d) None of these
280. An arc of 45° in one circle equals an arc of 60° in another. The radian measure of an angle subtended
at the centre of the first circle by an arc having length equal to the radius of the second circle is …
3 4
(a) (b) (c) 1 (d) none of these
4 3

Office.: 606 , 6 Floor, Hariom Tower, Circular Road, Ranchi-1, Ph.: 0651-2562523, 9835508812, 8507613968
th
22 BY R. K. MALIK’S NEWTON CLASSES
( )
281. OAB is a sector of a circle with centre O and radius 12 cm. If m∡AOB = 60°, The area enclosed
between arc AB and chord AB is …..
(a) 12 2π − 2 3
( ) (b) 12 2π − 3 3
( ) (c) 12 2π − 4 3
( ) (d) none of these

282. In a circle of radius 6 cm. chords AB and AC are respectively at a distance of 3 cm and 3 2 cm
from the centre. The perimeter and the area of the sector bounded by the minor arc BC
(a) 5π (b) 10π (c) 15π (d) none of these
283. Two circles, each of radius 5 cm, intersect each other. If the distance between their centers is 5 2
cm. The area of the region common to the circles
25 25 25
(a) (π − 2 ) (b) (π − 3 ) (c) (π − 1) (d) None of these
2 2 2
284. The minute-hand of a clock gains 5°30′ on the hour-hand in one minute T/F
1 1
285. = ....
cos 290° 3 ⋅ sin 250°
+

(a) 1/ 2 (b) 3 / 2 (c) 4 / 3 (d) None of these


286. sin A ⋅ tan A + cos A ⋅ cot A =
2 2 2 2

(a) 1 + tan 2 A + cot 2 A (b) tan 2 A + cot 2 A − 1 (c) 1 + sec 2 A + tan 2 A (d) 1 + csc2 A + cot 2 A
287. If sec θ ⋅ tan θ ( sec θ + tan θ ) + ( sec θ − tan θ ) = sec n θ + tan n θ , then : n =
(a) 1 (b) 2 (c) 3 (d) 4
288. If 4 ( sin 3θ ⋅ cos3 θ + cos 3θ ⋅ sin 3 θ ) = m ⋅ sin ( nθ ) , then : ( m, n ) =

(a) (1, 2 ) (b) ( 2, 3) (c) ( 3, 4 ) (d) ( 4, 5)


1 tan ( A + B )
289. If sin B = ⋅ sin ( 2 A + B ) , then
5 tan A
=

2 3 3
(a) (b) (c) (d) none of these
3 2 4
290. If sin A + cos B = a, sin B + cos A = b, then sin ( A + B ) =
a2 − b2 + 2 a2 + b2 − 2
(a) (b) (c) (d) None of these
a2 + b2
2 2 2

291. If cos θ1 = 2 ⋅ cos θ 2 then  tan 1 2   tan 1 2  =


2  2 
 θ −θ   θ +θ 

1 1

(a) (b) − (c) 1 (d) −1


3 3
cos x cos ( x + θ ) cos ( x + 2θ ) cos ( x + 3θ )
292. If then :
a+c
= = = =
b+d

(a) (b) (c) (d)


a b c d
a c b d

293. If a ⋅ sec 2 θ − b ⋅ tan 2 θ = c, then sin θ = ….. (given θ is acute angle)


d d c a

(a) (b) (c) (d)


a+c a−c a−c a+c

294. If a ⋅ cos A − b ⋅ sin A = c, then : a ⋅ sin A + b ⋅ cos A can be equal to


b+c b−c b+c b−c

(a) a2 + b2 − c2 (b) a 2 − b2 + c 2 (c) b2 + c2 − a2 (d) a2 + b2 + c2

Office.: 606 , 6 Floor, Hariom Tower, Circular Road, Ranchi-1, Ph.: 0651-2562523, 9835508812, 8507613968
th
( ) BY R. K. MALIK’S NEWTON CLASSES 23

295. If cos ec θ − sin θ = m and sec θ − cos θ = n, then : m 2 n 2 ( m 2 + n 2 + 3) =


(a) 1 (b) 2 (c) 3 (d) 5

 tan  2
 α + β 
296. If 3sin α = 5sin β , then : 

 

 tan  2
=
 α − β 


(a) 1 (b) 2 (c) 3 (d) 4


  

297. If the roots of x 2 + px + q = 0 are tan 30° and tan15°, then : − p + q + 2 =


(a) 1 (b) 2 (c) 3 (d) 0
298. If : 3sin β = sin ( 2α + β ) , then : tan (α + β ) − 2 tan α is independent of
(a) α (b) β (c) both α and β (d) none of these
299. If cos α + cos β = a, sin α + sin β = b and : α − β = 2θ , then : ( cos 3θ ) / ( cos θ ) =
(a) a 2 + b 2 − 2 (b) a 2 + b 2 − 3 (c) 3 − a 2 − b 2 (d) a 2 + b 2
300. If sec α and csc α are the roots of x 2 − px + q = 0, then :
(a) p 2 = q ( q − 2 ) (b) p 2 = q ( q + 2 ) (c) p 2 + q 2 = 2q (d) p = q
301. If sin 2 θ + 3cos θ − 2 = 0 then : cos3 θ + sec3 θ =
(a) 18 (b) 9 (c) 4 (d) ¼
302. If x = sin θ ⋅ cos θ and y = sin θ cos θ , then :
2 2

(a) ( x 2 y ) =1 (b) ( x 2 y ) + ( y2 / x) =1
2/3 2/3 2/3 2/3
= ( xy 2 )

(c) x 2 + y 2 = x 2 y 2 (d) None of these


303. If tan x + cot x = 4, then : tan x + cot x = ... 4 4

(a) 110 (b) 191 (c) 80 (d) 194


304. The number of integral triplets ( a, b, c ) such that a + b cos 2 x + c sin x = 0 for all x, is 2

(a) 0 (b) 1 (c) 3 (d) infinitely many

305. Statement-1: If tan A, tan B are the roots of the equation x 2 − ax − 1 = 0, then sin 2 ( A + B ) =
a2
1 + a2
tan 2 ( A + B )
Statement-2: sin ( A + B ) =
2

1 + tan 2 ( A + B )

(a) 1 (b) 2 (c) 3 (d) 4


306. If tan ( A + B ) = p and tan ( A − B ) = q, then the value of tan 2 A is

1 + pq
(a) (b) (c) (d)
1 + pq 1− p 1 − pq
p+q p−q p+q
p−q

sin 2 3 A cos 2 3 A
307.
sin 2 A cos 2 A
− =

(a) cos 2 A (b) 8 cos 2 A (c) 1/ 8 cos 2 A (d) none of these


3 5A
308. If cos A = , then the value of sin sin is
4 2 2
A

Office.: 606 , 6 Floor, Hariom Tower, Circular Road, Ranchi-1, Ph.: 0651-2562523, 9835508812, 8507613968
th
24 ( ) BY R. K. MALIK’S NEWTON CLASSES
1 11 11 11
(a) (b) (c) (d)
32 8 32 16
309. If sin x + sin 2 x = 1, then cos8 x + 2 cos 6 x + cos 4 x =
(a) 0 (b) −1 (c) 2 (d) 1
2π 4π
310. If x = y cos = z cos , then xy + yz + zx =
3 3
(a) −1 (b) 0 (c) 1 (d) 2
311. If sin α = sin β and cos α = cos β , then

(a) sin =0 (b) cos =0 (c) sin =0 (d) cos  =0


2 2 2  2
α +β α +β α −β α − β 

312. The maximum value of cos 2 A + cos 2 B − cos 2 C , is
(a) 0 (b) 1 (c) 3 (d) 2

Office.: 606 , 6 Floor, Hariom Tower, Circular Road, Ranchi-1, Ph.: 0651-2562523, 9835508812, 8507613968
th
JEE (MAIN & ADV.), MEDICAL + BOARD, NDA, X & IX
Enjoys unparalleled reputation for best results
in terms of percentage selection
www.newtonclasses.net

SOLUTION OF TRIGONOMETRIC FUNCTION ( )


Only one option is correct.
2 2
1. Ans. (c), tan 9° + cot 9° − ( tan 27° + cot 27° ) = [Using cot θ + tan θ = 2 cosec 2θ ]
sin18° sin 54°

 1 1   4 4  2
= 2 = 2 = 8× = 4
 sin18° cos 36°   5 −1 5 + 1 4
−  − 

tan 42° + tan 3°


2. Ans. (c), As we know that, 1 = tan 45° = tan ( 42° + 3° ) =
1 − tan 42° tan 3°
⇒ tan 42° + tan 3° + tan 42°.tan 3° = 1
1
3. Ans. (d), Given, 3 cos 23° − sin 23°
4
{ }
1 1 1
= {cos 30° cos 23° − sin 30° sin 23°} = cos ( 30° + 23° ) = cos 53°
2 2 2
4. Ans. (d), Given, sin 47° + sin 61° − ( sin11° + sin 25° ) = 2 sin 54° cos 7° − 2sin18° cos 7°

5 +1 5 −1
= 2 cos 7° ( sin 54° − sin18° ) = 2 cos 7°.2 cos 36°.sin18° = 4.cos 7°. . = cos 7°
4 4
5. Ans. (a), Given, cos 52° + cos 68° + cos172° = ( cos 52° + cos172° ) + cos 68°
= 2 cos112° cos 60° + cos 68° = cos112° + cos 68° = 2 cos ( 90° ) cos 22° = 0
cos 9° + sin 9° 1 + tan 9°
6. Ans. (a), Given, = tan ( 45° + 9° ) = tan 54°
cos 9° − sin 9° 1 − tan 9°
=

7. Ans. (a), The given equation is 3cos A + 2 = 0


2
∴ cos A = − and A is angle of a triangle. Hence A < π
3
5 sin A 5
⇒ sin A is positive. ∴ sin A = (1 − cos A) =
2
⇒ tan A =
3 cos A 2
=−

5 5
∴ sin A + tan A = − and sin A tan A = − .
6 6
Hence, the equation whose roots are sin A and tan A , is
5  5
x 2 − ( sin A + tan A) x + sin A tan A = 0 ⇒ x 2 + x+−  = 0 ⇒ 6 x2 + 5x − 5 = 0
6  6
1 − sin θ 1 + sin θ 1 − sin θ + 1 + sin θ 2 2
8. Ans. (b),
1 + sin θ 1 − sin θ 1 − sin θ
2
cos θ
2 cos θ
+ = = =

2
= −2sec θ
cos θ ∵ 2 < θ < π 
 π 
=−

9. Ans. (c)We have, tan A < 1 and A is acute.

⇒ cos A > sin A


4 4
π π
∴ − < A<

Office.: 606 , 6 Floor, Hariom Tower, Circular Road, Ranchi-1, Ph.: 0651-2562523, 9835508812, 8507613968
th
2 ( ) BY R. K. MALIK’S NEWTON CLASSES
1 + sin 2 A + 1 − sin 2 A ( cos A + sin A) + ( cos A − sin A)
2 2

Now,
1 + sin 2 A − 1 − sin 2 A ( cos A + sin A ) − ( cos A − sin A )
2 2
=

cos A + sin A + cos A − sin A ( cos A + sin A ) + ( cos A − sin A ) = cot A


cos A + sin A − cos A − sin A ( cos A + sin A) − ( cos A − sin A)
= =

10. Ans. (c), We have, cos 2 ( A − B ) + cos 2 B − 2 cos ( A − B ) cos A cos B

= cos 2 ( A − B ) + cos 2 B − cos ( A − B ) × cos ( A − B ) + cos ( A + B ) 

= cos 2 B − cos ( A − B ) cos ( A + B ) = cos 2 B − ( cos 2 A − sin 2 B )


= cos 2 B + sin 2 B − cos 2 A = 1 − cos 2 A = sin 2 A . Hence, it depends on A .
 cos A + cos B   sin A + sin B 
11. Ans. (b), 
 sin A − sin B   cos A − cos B 
n n

 + 

 2 cos  2  cos  2    2sin  2  cos  2  


n n
  A+ B   A− B    A+ B   A− B 

 2 cos  A + B  sin  A − B    2sin  A + B  sin  B − A  


=     +     

 2   2     2   2  
        

= cot n   + cot   = cot   + ( −1) cot 


 2   2   2   2 
 A− B  n  B− A n  A− B  n n  A− B 

= 2 cot n  [∵ n is even]
 2 
 A− B 

12. Ans. (d), Since sin θ , cos θ are the roots of ax 2 − bx + c = 0 .

∴ sin θ + cos θ = …(1) and sin θ cos θ = …(2)


b c
a a

Squaring (1) we get, sin 2 θ + cos 2 θ + 2sin θ cos θ = ⇒ 1+ 2  = 2


2
b2 c b
a2
b 2 2c b 2 − 2ac
a a

⇒ 1= ⇒ a 2 = b 2 − 2ac ⇒ b 2 − a 2 = 2ac ⇒ b 2 − a 2 = 2ac .


a2 a a2
− =

13. Ans. (a), 2 cos x − cos 3 x − cos 5 x

= 2 cos x − [ cos 3 x + cos 5 x ] = 2 cos x − ( 2 cos 4 x cos x ) ∵ cos C + cos D = 2 cos 2 cos 2 
 C+D C − D

= 2 cos x − 2 cos x cos 4 x = 2 cos x (1 − cos 4 x ) = 2 cos x 2sin 2 2 x ∵ 1 − cos x = 2sin
2

2 
 x

= 4 cos x ( 2sin x cos x ) = 16 sin 2 cos3 x


2

1
14. Ans. (c), Now, sin12° sin 48° sin 54° = ( 2sin12° sin 48° ) cos ( 90° − 54° )
2
1 1  5 + 1 1   5 + 1 5 + 1
( cos 36° − cos 60° ) cos 36° =  ∵ cos 36° =
2 2 4 2  4  4 

= −   

1  5 − 1  5 + 1 5 − 1 4 1

2  4   4  32 32 8
=   = = =

15. Ans. (b), Since, sec α , cosec α are the roots of the equation x 2 − px + q = 0

Office.: 606 , 6 Floor, Hariom Tower, Circular Road, Ranchi-1, Ph.: 0651-2562523, 9835508812, 8507613968
th
( BY R. K. MALIK’S NEWTON CLASSES
) 3
∴ sec α + cosec α = p, sec x .cosec α = q
sin α + cos α 1
= p , sin α cos α = ⇒ sin α + cos α =
sin α cos α
p

2 p2
q q

⇒ sin 2 α + cos 2 α + 2sin α cos α = ⇒ 1+ ⇒ q ( q + 2) = p2 .


p2
q2 q q2
=

16. Ans. (c), cos 2 A ( 3 − 4 cos 2 A ) + sin 2 A ( 3 − 4sin 2 A ) = ( 3cos A − 4 cos3 A ) + ( 3sin A − 4 sin 3 A )
2 2 2 2

= ( − cos 3 A ) + ( sin 3 A ) = 1 ∵ cos 3 A = 4 cos3 A − 3cos A and sin 3 A = 3sin A − 4sin 3 A
2 2

1 + tan θ  3 tan θ − tan 3 θ


17. Ans. (c), tan  θ +  = 3 tan 3θ = ⇒ 3 .
4 1 − tan θ  1 − 3 tan θ
 π 
2

⇒ 1 + tan θ − 3 tan 2 θ − 3 tan 3 θ = 9 tan θ − 3 tan 3 θ − 9 tan 2 θ + 3 tan 4 θ


 

⇒ 3 tan 4 θ − 6 tan 2 θ + 8 tan θ − 1 = 0 .


On simplification, we get 3 tan 4 θ − 6 tan 2 θ + 8 tan θ − 1 = 0 .
1
∴ Products of roots = tan α . tan β . tan γ .tan δ = −
3
26 13 15
18. Ans. (a), ∵ tan α / 2 + tan β / 2 = and tan α / 2 tan β / 2 = .
8 4 8
=

13
 α + β  tan α / 2 + tan β / 2 26
∴ tan  = 4 =− .
 2  1 − tan α / 2 tan β / 2 1 − 15 7
=
8
 26 
2

1 − tan   1−  − 
 =  7  = 49 − 676 = − 627 .
2

 2
α + β 
Now, cos (α + β ) =
 26  49 + 676 725
1 + tan 2 
2

 1+  − 
 2  7 
α + β 

19. Ans. (c), Given, 2 tan + 3sec − 4 cos = 2 tan18° + 3sec18° − 4 cos18°
10 10 10
π π π

2sin18° + 3 − 4 cos 2 18° 2sin18° + 3 − 2 (1 + cos 36° )


cos18° cos18°
= =

5 −1 5 +1 5 1 5 1
2 + 3− 2 − 2 − +1−
4 4 2 2 2 2 = 0 =0

cos18° cos18° cos18°
= =

tan 77° + cot 47° tan 77° + cot 47°


20. Ans. (b), = tan 77° cot 47°
1 1 1 1
=

cot13° tan 43° tan 77° cot 47°


+ +

21. Ans. (b), ( sin 2 θ + cos 2 θ ) = (1)


3 3

⇒ sin 6 θ + cos 6 θ + 3sin 2 θ cos 2 θ = 1 and sin 4 θ + cos 4 θ + 2 sin 2 θ cos 2 θ = 1


Both gives, 2 ( sin 6 θ + cos 6 θ ) − 3 ( sin 4 θ + cos 4 θ ) + 1 = 0
sin ( x + y )
22. Ans. (b), Given,
sin ( x − y )
a+b
=
a −b

Office.: 606 , 6th Floor, Hariom Tower, Circular Road, Ranchi-1, Ph.: 0651-2562523, 9835508812, 8507613968
4 BY R. K. MALIK’S NEWTON CLASSES
( )
sin ( x + y ) + sin ( x − y ) ( a + b ) + ( a − b )
Applying componendo and dividendo rule, we get,
sin ( x + y ) − sin ( x − y ) ( a + b ) − ( a − b )
=

2sin x cos y 2a tan x a


2 cos x sin y 2b tan y b
⇒ = ⇒ =

23. Ans. (b), Given, tan θ + sec θ = e x …(1) ∴ sec θ − tan θ = e− x …(2)
2
On adding equation (1) and (2), we get, ∴ 2 sec θ = e x + e− x ⇒ cos θ = x − x .

24. Ans. (a), ∵ ( tan α − cot α ) ≥0 ⇒ tan 2 α + cot 2 α − 2 ≥ 0 ⇒ tan 2 α + cot 2 α ≥ 2 .


e +e
2

3π 4π
25. Ans. (a), cos 2 + cos 2 = cos 2 108° + cos 2 144°
5 5
= cos 2 ( 90° + 18° ) + cos 2 (180° − 36° ) = sin 2 18° + cos 2 36°

 5 − 1   5 + 1  5 + 1 − 2 5 5 + 1 + 2 5 12 3
2 2

= sin 18° + cos 36° = 


2 2

 4   4  16 16 16 4
 +   = + = =

26. Ans. (b), cos 4 θ − sin 4 θ = ( cos 2 θ − sin 2 θ )( cos 2 θ + sin 2 θ ) = cos 2θ = 2 cos 2 θ − 1

1 − tan 2 θ 1 − tan 2 15° 3


27. Ans. (c), cos 2θ = = cos 30° = .
1 + tan 2 θ 1 + tan 15°
2
2

1
28. Ans. (c), The given relation sin θ + cosec θ − 2 = 0 ⇒ sin θ + −2=0
sin θ
⇒ sin 2 θ − 2 sin θ + 1 = 0 ⇒ ( sin θ − 1) = 0 ⇒ sin θ = 1 = cosec θ …(1)
2

∴ sin 2 θ + cosec 2 θ = 1 + 1 = 2 by using (1)


Alternatively :
sin 2 θ + cosec 2 θ = ( sin θ + cosec θ ) − 2sin θ cosec θ = (1 + 1) − 2 ×1 = 2 [By using (1)]
2 2

 π 9π   3π 7π 
29. Ans. (b),  , ,  ,  are complementary angles as their sum is π / 2 .
 20 20   20 20 
3π 5π 7π 9π  π 9π  5π 3π  π 7π 
tan tan tan tan tan = tan cot  −  tan tan cot  −
20 20 20 20 20 20  2 20  20 20  2 20 
π π

5π 3π 3π 5π
= tan cot tan tan cot = 1 × 1× 1 = 1 [∵ tan θ cot θ = 1 and tan = tan = 1 ]
20 20 20 20 20 20 4
π π π

30. Ans. (b), 2 sin 2 θ + 3cos 2 θ = 2 ( sin 2 θ + cos 2 θ ) + cos 2 θ = 2 + cos 2 θ ≥ 2 ∵ cos 2 θ > 0 

31. Ans. (b), sin 4 θ + cos 4 θ = ( sin 2 θ + cos 2 θ ) − 2sin 2 θ cos 2 θ = 1 − 2sin 2 θ cos 2 θ ≤ 1
2

[For greatest value of this, 2sin 2 θ cos 2 θ should have least value i.e., 0]
32. Ans. (c), We have sin (π /10 ) sin (13π / 10 ) = sin ( π / 10 ) sin ( π + 3π /10 )
1
= − sin (π /10 ) sin ( 3π /10 ) = − sin18° sin 54° = − sin18° cos 36° = −
4
( ) 14 (
5 −1 . 5 +1 = −
) 1
4

33. Ans. (b), The given expression


(1 − sin α ) + (1 + sin α )
(1 + sin α )(1 − sin α ) 

Office.: 606 , 6 Floor, Hariom Tower, Circular Road, Ranchi-1, Ph.: 0651-2562523, 9835508812, 8507613968
th
( BY R. K. MALIK’S NEWTON CLASSES
) 5
2 2 2
[∵ π / 2 < α < π ⇒ cos α < 0 ⇒ cos α = − cos α ]
( cos2 α ) cos α cos α
= = =−

7π 4π
34. Ans. (c), sin 2 + sin 2 + sin 2 + sin 2
18 9 18 9
π π

8π 2π 7π  4π 8π π 2π 
= sin 2 + sin 2 + sin 2 + sin 2 ∵ 9 = 18 , 9 = 18 
18 18 18 18
π

  π 8π   2 2π   π 7π  
2 2

= sin 2 + cos  −   + sin + cos  −


18   2 18   18   2 18  
π

 2π 2  2π
=  sin 2   + cos 2    +  sin 2   + cos    = 1+1 = 2
 18   18     18  18
 π   π    

35. Ans. (d), Since tan θ = 1/ 7 , we have sin θ = 1/ 5 2 and cos θ = 7 / 5 2 .


  

Also sin φ = 1/ 10 ⇒ tan φ = 1 / 3 .


∴ sin 2φ = ( 2 tan φ ) / (1 + tan 2 φ ) = ( 2 / 3) / 1 + (1 / 9 )  = 3 / 5
10 5 2
1 1 1
1−
1 − tan φ 9=4
and so cos 2φ =
2

1 + tan φ 1 + 1 5
2
3 7
= φ θ

9
1
4 7 3 25 1
∵ sin (θ + 2φ ) = sin θ cos 2φ + cos θ sin 2φ = . + . = ∴ θ + 2φ = 45°
5 2 5 5 2 5 25 2 2
=

36. Ans. (b), We have


 a+b  a −b  a +b+ a −b
[∵ a > b > 0]
( a 2 − b2 )
 +  =
 a −b   a+b

2a 2 2 2

{1 − (b / a ) } (1 − tan x ) sin 2 x
1−
2
− b2 ) 2 2
= = = =

cos 2 x
(a
2 cos x 2 cos x
∵ cos 2 x − sin 2 x = cos 2 x 
( cos 2
x − sin x )
2
( cos 2 x )
= =

cos 76° cos16°


3+ .
37. Ans. (c),
3 + cot 76° cot16° sin 76 sin16° = 3sin 76° sin16° + cos 76° cos16°
cot 76° + cot16° cos 76° cos16° sin ( 76° + 16° )
= °

sin 76° sin16°


+

( 2sin 76° sin16° ) + ( sin 76° sin16° + cos 76° cos16° ) = 2sin 76° sin16° + cos ( 76° − 16° )
sin 92° sin 92°
=

cos ( 76° − 16° ) − cos ( 76° + 16° ) + cos 60° cos 60° − cos 92° + cos 60°
sin 92° sin 92°
= =

1 1  92 
− cos 92° + 2sin 2  
= 2 2 = 1 − cos 92° =  2  = tan 46°
sin 92° sin 92°  92   92 
2sin   cos  
 2   2 
38. Ans. (a), The given equation
α + β = 90° ...(1) α − β = 30° ...(2)

Office.: 606 , 6th Floor, Hariom Tower, Circular Road, Ranchi-1, Ph.: 0651-2562523, 9835508812, 8507613968
6 BY R. K. MALIK’S NEWTON CLASSES
( )
From (1) & (2), α = 60° & β = 30° ∴ α + 2 β = 120° and 2α + β = 150°
So, tan (α + 2β ) tan ( 2α + β ) = tan120° tan150°
= tan ( 90° + 30° ) tan (180° − 30° ) = − cot 30° ( − tan 30° ) = 1
39. Ans. (c), Multiply numerator and denominator by 1 − cos θ + sin θ
2 sin θ (1 − cos θ + sin θ ) 2sin θ (1 − cos θ + sin θ ) 2sin θ (1 − cos θ + sin θ )
(1 + sin θ ) − cos θ (1 + sin θ ) − cos θ 1 + sin 2 θ + 2 sin θ − (1 − sin 2 θ )
2 2 2 2
∴ x= = =

2 sin θ (1 − cos θ + sin θ ) 2 sin θ (1 − cos θ + sin θ ) 1 − cos θ + sin θ 1 − cos θ + sin θ
2sin 2 θ + 2 sin θ 2sin θ (1 + sin θ ) 1 + sin θ 1 + sin θ
= = = ⇒ x=

40. Ans. (a), Let y = 3cos x + 5sin  x − 


6
 π

3 1 1 5 3

= 3cos x + 5  sin x . − cos x .  = cos x + sin x


2 2 2 2

 1 75   1 75 

∵ a cos x + b sin x lie between − a 2 + b 2 & a 2 + b 2 


4 4  4 4 
∴ −  + ≤y≤  + 
 

⇒ − 19 ≤ y ≤ 19 then ( a, b ) is − 19, 19 ( )
41. Ans. (c), sin ( x + 3α ) = 3sin (α − x )
⇒ ( sin x cos 3α + cos x sin 3α ) = 3 ( sin α cos x − cos α sin x )
⇒ sin x {cos 3α + 3cos α } = cos x {3sin α − sin 3α }
4 sin 3 α
⇒ tan x = = tan 3 α [∵ sin 3 A = 3sin A − 4sin 3 A and cos 3 A = 4 cos 3 A − 3cos A ]
4 cos α
3

42. Ans. (c), ∵ a sec α = d + c tan α …(1) and b sec α = c − d tan α …(2)
Squaring and adding equation (1) and (2),
(a 2
+ b 2 ) sec2 α = d 2 + c 2 tan 2 α + 2dc tan α + c 2 + d 2 tan 2 α − 2dc tan α

⇒ (a 2
+ b 2 ) sec2 α = c 2 ( tan 2 α + 1) + d 2 (1 + tan 2 α ) = ( c 2 + d 2 ) sec 2 α . Hence, a 2 + b 2 = c 2 + d 2 .

43. Ans. (b), The given expression


(1 − sin α ) + (1 + sin α )
(1 + sin α )(1 − sin α ) 
2 2 2
[∵ < α < π ⇒ cos α < 0 ⇒ cos α = − cos α ]
( cos α ) cos α cos α 2
π
2
= = =−

44. Ans. (d), 5cos A + 3 = 0 .


3
So, cos A = − ⇒ A being the angle of ∆ ABC , sin A cannot be –ve.
5
3 4 4
So, cos A = − ⇒ sin A = . Then tan A = − .
5 5 3
8 16
∴ S = sin A + tan A = − and P = sin A tan A = − .
15 15
Hence, sin A and tan A are the roots of the equation x 2 − Sx + P = 0

Office.: 606 , 6 Floor, Hariom Tower, Circular Road, Ranchi-1, Ph.: 0651-2562523, 9835508812, 8507613968
th
( BY R. K. MALIK’S NEWTON CLASSES
) 7

 8   16 
x 2 +   x −   = 0 or 15 x 2 + 8 x − 16 = 0
 15   15 
45. Ans. (d), ∵ sin θ1 + sin θ 2 + sin θ3 = 3 ,
which is possible only when sin θ1 = sin θ 2 = sin θ3 = 1

∴ cos θ1 + cos θ 2 + cos θ3 = 0 + 0 + 0 = 0


2
π
θ1 = θ 2 = θ3 =

46. Ans. (c), ∑ cos ( 5r ) = cos 2 5° + cos 2 10° + cos 2 15° + ..... + cos 2 85° + cos 2 90°
18
2 °

r =1

= ( cos 2 5° + cos 2 85° ) + ( cos 2 10° + cos 2 80° ) + ( cos 2 15° + cos 2 75° ) + .....

+ ( cos 2 40° + cos 2 50° ) + cos 2 45°

= ( cos 2 5° + sin 2 5° ) + ( cos 2 10° + sin 2 10° ) + ( cos 2 15° + sin 2 15° ) + .....
1
+ ( cos 2 40° + sin 2 40° ) + cos 2 45° = 1 + 1 + 1 + .. + 1 +
2
1 17
= 8+
2 2
=

47. Ans. (a), tan 75° − cot 75° = cot15° − cot 75°
= 2+ 3 − 2− 3 = 2 3
( ) ( ) [∵ cot15° = 2 + 3 & cot 75° = tan15° = 2 − 3 ]
48. Ans. (b), sin163° cos 347° + sin 73° sin167°
= sin (180° − 17° ) cos ( 360° − 13° ) + cos ( 90° − 17° ) sin (180 − 13° )
1
= sin17° cos13° + cos17° sin13° = sin (17° + 13° ) = sin 30° =
.
2
2 sin β cos β
49. Ans. (a), The given equation may be written as
cos 2α cos β sin β
= +

2 sin 2 β + cos 2 β
⇒ cos 2α = 2sin β cos β ⇒ cos 2α = sin 2 β
cos 2α sin β cos β
⇒ =

⇒ cos 2α = cos  − 2 β  ⇒ 2α = − 2 β ⇒ 2α + 2 β =
2 2 2 4
π  π π π
⇒α +β =

50. Ans. (c), 6 ( sin 6 θ + cos 6 θ ) − 9 ( sin 4 θ + cos 4 θ ) + 4


= 6 ( sin 2 θ + cos 2 θ ) − 3sin 2 θ cos 2 θ ( sin 2 θ + cos 2 θ ) 


3

 
−9 ( sin 2 θ + cos 2 θ ) − 2sin 2 θ cos 2  + 4 [∵ a 3 + b3 = ( a + b ) − 3ab ( a + b ) ]
2 3

= 6 1 − 3sin 2 θ cos 2 θ  − 9 1 − 2 sin 2 θ cos 2 θ  + 4 = 6 − 9 + 4 = 1


 

51. Ans. (c), Given equation cos x + cos y + cos α = 0 and sin x + sin y + sin α = 0 .
The given equation may be written as cos x + cos y = − cos α and sin x + sin y = − sin α .

Therefore, 2 cos   cos   = − cos α …(1)


 2   2 
 x+ y  x− y

2 sin   cos   = − sin α …(2)


 2   2 
 x+ y  x− y

Office.: 606 , 6th Floor, Hariom Tower, Circular Road, Ranchi-1, Ph.: 0651-2562523, 9835508812, 8507613968
8 ( BY R. K. MALIK’S NEWTON CLASSES
)

2 cos   cos 
 2   2  = cos α ⇒ cot  x + y  = cot α
 x+ y  x− y
Dividing equation (1) by (2), we get,

 x − y  sin α  2 
2sin  cos
 2   2 
 
 x+ y
  

52. Ans. (b), tan θ + sec θ = e x …(1) ∴ sec θ − tan θ = e− x …(2)


2
From equation (1) and (2), 2 sec θ = e x + e− x ⇒ cos θ = x − x

53. Ans. (d), tan A + cot A = 4


e +e

Squaring both side, we get, tan 2 A + cot 2 A + 2 tan A cot A = 16 ⇒ tan 2 A + cot 2 A = 14
Again squaring both side, we get, tan 4 A + cot 4 A + 2 = 196 ⇒ tan 4 A + cot 4 A = 194
sin 70° sin 20°
tan 70° − tan 20° cos 70° cos 20°
54. Ans. (b),
sin 50°

tan 50°
=

cos 50°
sin 70° cos 20° − cos 70° sin 20°
cos 70° cos 20° sin ( 70° − 20° ) cos 50°
sin 50° cos 70° cos 20° sin 50°
= =

cos 50°
2 sin 50° cos 50° 2 cos 50°
[∵ 2 cos A cos B = cos ( A + B ) + cos ( A − B ) ]
2 cos 70° cos 20° sin 50° cos 90° + cos 50°
= =

2 cos 50°
=2
0 + cos 50°
=

55. Ans. (c), ( sin θ + cos 2 θ ) = sin 6 θ + cos 6 θ + 3sin 2 θ cos 2 θ ( sin 2 θ + cos 2 θ )
2 3

⇒ 1 = sin 6 θ + cos 6 θ + 3sin 2 θ cos 2 θ


5 −1 5 − 5
56. Ans. (b), Here, ( sin 9° − cos 9° ) = 1 − 2sin 9° cos 9° = 1 − sin18° = 1 −
2

4 4
=

But In first half of first quadrant sin 9° < cos 9° or sin 9° − cos 9° = − 2 sin ( 45° − 9° )

10 − 2 5 5− 5 5− 5
= − 2 sin 36° = − 2 . Thus, sin 9° − cos 9° = −
4 2 2
=−

cos 4 x sin 4 x
57. Ans. (a), cos x = sin x ⇒ tan x + tan x =
4 2 2
+ 2 = cos2 x + sin 2 x = 1
4

cos x sin x
2

1 − tan 2 θ 1 − 2 tan 2 φ − 1
58. Ans. (a), cos 2θ = = − sin 2 φ ⇒ cos 2θ + sin 2 φ = 0
1 + tan θ 1 + 2 tan φ + 1
2 2
=

sin α .cos α
tan α −
59. Ans. (b), tan (α − β ) =
tan α − tan β 2 + cos 2 α = tan α  2 + cos α − cos α  = 2 tan α
2 2

1 + tan α . tan β 1 + tan α sin α .cos α  2 + cos 2 α + sin 2 α  3


=

2 + cos 2 α
 

60. Ans. (d), we have log cosec 89° = log sec1° and similarly other terms are obtained so the required sum
cos1° cos 2° cos 44°
is equal to log + log + ...... + log = 44 log1 = 0 = log ( cosec 45°.cos 45° )
cos1° cos 2° cos 44°
61. Ans. (d), Given, sin 2 24° − sin 2 6° sin 2 42° − sin 2 12°
( )( )

Office.: 606 , 6 Floor, Hariom Tower, Circular Road, Ranchi-1, Ph.: 0651-2562523, 9835508812, 8507613968
th
( ) BY R. K. MALIK’S NEWTON CLASSES 9

1 5 −1 5 +1 1
= sin 30°.sin18°.sin 54°.sin 30° = . .
4 4 4 16
=

tan 8° + 1
62. Ans. (b), Here = tan ( 45° + 8° ) = tan 53° = cot α = tan ( 90° − α ) ⇒ 90° − α = 53°
1 − tan 8°
 5 + 1   5 −1  3
2 2

63. Ans. (d), Given, sin 54° + cos 72° = sin 54° + sin 18° = 
2 2 2 2
 4   4  4
+ =

3 1 2.2  3 1 4
   

64. Ans. (d), Given, cos 20° − sin 20° = sin 40° = 4
sin 20° cos 20° cos 40°  2 2  sin 40°

− = 

tan 3θ tan θ
65. Ans. (c), Given expression = =1
tan 3θ − tan θ tan 3θ − tan θ

1
1 − 2 ( 2sin10°.sin 70° ) 1 − 2. + 2.sin10°
1 1 − 2 cos 60° + 2 cos80° 2
66. Ans. (d), − 2sin 70° = =1
2 sin10° 2sin10° 2 sin10° 2sin10°
= =

67. Ans. (c), The given expression = 2 cos 270°.cos 36° + 2 cos 90° cos 72° = 0

68. Ans. (b), 4 cos θ cos π +  θ −   .cos π +  θ +   = 4 cos θ .cos  θ −  cos  θ + 


3  3  3 3
  π    π   π  π
     

= 4 cos θ  cos2 θ − sin 2  = 4 cos3 θ − 3cos θ = cos 3θ ∴ k = 3


3
 π

1

69. Ans. (d), Here k = [ cos 40° − cos 60° + cos 200° − cos 300° + cos 240° − cos 260°]
2
1
cos 40° − − cos 20° − sin 30° − cos 60° + cos80°
2 2
1 
= 

1 3  1 3 3 3

− − 2sin10°.sin 30° + cos ( 90° − 10° )  =  − − sin10° + sin10°  = − . So, − k =


2 2  2 2 4 4

= 

70. Ans. (c), Given expression = sin (180° − 17° ) cos ( 360° − 13° ) + sin ( 90° − 17° ) sin (180° − 13° )

1
= sin17°.cos13° + cos17°.sin13° = sin (17 + 13) ° =
2
96.cos 25°.sin 35°.cos10° 96 cos 25° sin 35° 48cos 25°.sin 35°
71. Ans. (c), Given, = 24
2sin10°.cos10° + 2 cos10°.sin 60° 2 ( sin10° + sin 60° ) 2sin 35° − cos 2 35°
= =

3π 4π 2π
72. Ans. (d), cos2 + cos2 = cos 2  π −  + cos  π − 
2

5 5 5 3
  π
  
2π π  5 −1   5 + 1  3
2 2

= cos 2
+ cos 2 = 
5 5  4   4  4
 +  =

73. Ans. (d), Let = θ . then 15θ = π .


15
π

1
Now, cos θ .cos 4θ .cos 8θ = [sin 2θ .cos 2θ .cos 4θ .cos 8θ ]
2 sin θ
1 1 sin16θ −1
[sin 4θ .cos 4θ .cos8θ ] = [sin 8θ .cos8θ ] = [∵16θ = π + θ ]
2 sin θ 8sin θ 16sin θ 16
= 2 =

74. Ans. (d), Given expression = sin 20°.sin ( 60° − 20° ) sin ( 60° + 20° )

Office.: 606 , 6th Floor, Hariom Tower, Circular Road, Ranchi-1, Ph.: 0651-2562523, 9835508812, 8507613968
10 ( ) BY R. K. MALIK’S NEWTON CLASSES
3sin 20° − 4 sin 3 20° 1 3
= sin 20°. sin 2 60° − sin 2 20° = = .sin 60° =
4 4 8
( )
75. Ans. (a)
We have, Required distance = LM
= OL − OM = 12 cos 36° − 12 cos 72°

 5 +1 5 −1  72° O
= 12 ( cos 36° − sin18° ) = 12   = 6 units, 12
 4 4 
36°

⇒ Either all tan A, tan B, tan C are positive or two out of C


them are negative but if two are negative, then two angles 12
M D

become obtuse, which is not possible for a triangle.


Hence, ∆ is acute angled triangle.
A L B

1 + tan θ  3 tan θ − tan 3 θ


77. Ans. (c), = 3 .
1 − tan θ  1 − 3 tan θ

2

After simplification, we get, 3tan 4 θ − 6 tan 2 θ + 8 tan θ − 1 = 0


1
∴ Product of roots = tan α . tan β . tan γ . tan δ = −
3
78. Ans. (d), cosec2 x + 25sec 2 x = 26 + cot 2 x + 25 tan 2 x = 26 + 10 + ( cot x − 5 tan x ) ≥ 36
2

79. Ans. (a), Given, sin  x +  + cos  x + 


6 6
 π  π

 1 π 1 5π 
 

= 2 sin  x +  + cos  x +   = 2 sin  + x +  = 2 sin  x +


 2 6 2 6  4 6 12 
  π  π π 

5π  5π π
  

Hence, maximum value is 2 when sin  x +  = 1 = sin


12  2 2 12 12
 π π
∴x= − =

80. Ans. (c), ∵ sin α = sin β ⇒ sin α − sin β = 0 ∴ 2 cos   sin  =0
 2   2 
α + β  α − β 

and cos α = cos β ⇒ cos β = cos α = 0 ∴ 2 sin   sin  =0


 2   2
α + β  α − β 

From equation (i) and (ii), we get sin   = 0.


 2
α − β 

∵ Both sin  2  and cos  2  can not be zero simultaneously 


 α + β  α + β  

2π 4π 5π 6π 7π
     

81. Ans. (b), Given, cos cos cos cos cos cos
15 15 15 15 15 15
π

2π 4π 8π  3π 5π 6π
= cos cos cos cos  π −  cos cos cos
15 15 15 15  15 15 15
π 

2π 4π 8π  2π

= −  cos cos cos cos  cos cos cos


15 15 15 15  5 3 5
 π π π

 5 + 1   1   5 −1    2π  4π  8π  

  cos   cos   cos   cos 


 4  2  4   15   15  15  15  
π   
= −     
  
 

Office.: 606 , 6 Floor, Hariom Tower, Circular Road, Ranchi-1, Ph.: 0651-2562523, 9835508812, 8507613968
th
( ) BY R. K. MALIK’S NEWTON CLASSES 11

sin  2 4.  sin  π + 
1  15  = − 1 .  15  1
 π   π 

8 4 2 2
2 .sin   sin  
7
=− = 7

 15   15 
π  π 

2 (1 − tan 2 θ / 2 ) ( 2 tan θ / 2 )
82. Ans. (d), Given, 2 cos θ + sin θ = 1 ∴ =1
(1 + tan 2
θ / 2) (1 + tan 2
θ / 2)
+

⇒ 2 − tan 2 (θ / 2 ) + 2 tan (θ / 2 ) = 1 + tan 2 (θ / 2 ) ⇒ 3 tan 2 (θ / 2 ) − tan (θ / 2 ) − 1 = 0


1 1
∴ tan (θ / 2 ) = 1, tan θ / 2 = − ⇒ θ = 90°, tan θ / 2 = −
3 3
Also, 7 cos θ + 6sin θ at θ = 90° is 0 + 6 = 6
 1
7 1 − tan 2 θ / 2 7 1 −  + 12  −  56 − 4 20
) + 12 tan θ / 2 9  3 = 9 = 9 =2
 1
and 7 cos θ + 6sin θ =
(
(1 + tan θ / 2 ) (1 + tan θ / 2 ) 1 10 10
1+
2 2
= 

9 9 9
 a cos φ + b 
1− 
 1 − cos θ 
Ans. (a), tan θ / 2 = 
a + b cos φ  ( a − b )(1 − cos φ ) =
83.  tan (φ / 2 )
 1 + cos θ   a cos φ + b  ( a + b )(1 + cos φ )
  a −b 
1+ 
= = 

 a + b cos φ 
 a+b 

84. Ans. (b)


It is given that α , β , γ , δ are the smallest positive angles in ascending order of magnitude such that
sin α = sin β = sin γ = sin δ = k (a positive quantity)

⇒ β = π − α , γ = 2π + α and δ = 3π − α ∴ 4 sin + 3sin + 2sin + sin


2 2 2 2
α β γ δ

= 4 sin + 3cos − 2sin − cos


= 2  cos + sin  = 2 1 + sin α = 2 1 + k
2 2 2 2 2 2
α α  α α α α

1° 1 1° 1

85. Ans. (a), We know, sin 22 = 2 − 2 and cos 22 = 2+ 2


2 2 2 2
1°  1°   1°   1° 
∴ 1 + cos 22 1 + cos 67  1 + cos112  1 + cos157 
2  2  2  2

 1  1  1  1

= 1 + 2 + 2 1 + 2 − 2  1 − 2 − 2  1 − 2+ 2 
 2  2  2  2

4−2− 2 4−2+ 2 ) = ( 2 − 2 )( 2 + 2 ) = 4 − 2 = 1

 1  1
= 1 − 2 + 2  1 − 2 − 2  =
( )(
 4  4 16 16 16 8
( 
) ( )
1 1

1 1
86. Ans. (a), Let α + β = θ , where tan α = , tan β = ∴ tan θ = tan (α + β ) = 2 3 = 1 ⇒ θ =
1 1
+
2 3 4
π
1− .
2 3
87. Ans. (c), ( sec A + tan A − 1)( sec A − tan A + 1) − 2 tan A
= sec 2 A − tan 2 A + sec A + tan A − sec A + tan A − 1 − 2 tan A = 0
( ) ∵ sec 2 A − tan 2 A = 1

88. Ans. (d), Given, 3sin A + 5cos A = 5 ∴ ( 3cos A − 5sin A)


2

Office.: 606 , 6th Floor, Hariom Tower, Circular Road, Ranchi-1, Ph.: 0651-2562523, 9835508812, 8507613968
12 ( BY R. K. MALIK’S NEWTON CLASSES
)
= 9 cos A + 25sin A − 30sin A cos A = 9 − 9sin 2 A + 25 − 25cos2 A − 30sin A cos A
2 2

= 34 − 9sin 2 A + 25 cos 2 A + 30 sin A cos A = 34 − ( 3sin A + 5cos A) = 34 − 25 = 9


2
( )
tan A − tan B
89. Ans. (c), Given, A − B = ⇒ tan ( A − B ) = tan =1
4 4 1 + tan A tan B
π π

⇒ tan A − tan B − tan A tan B = 1 ⇒ tan A − tan B − tan A tan B + 1 = 2


⇒ (1 + tan A)(1 − tan B ) = 2 ⇒ y = 2 . Hence, ( y + 1) = ( 2 + 1) = ( 3) = 27
y +1 2 +1 3

cot A cot B 1 1
90. Ans. (d), .
1 + cot A 1 + cot B (1 + tan A)(1 + tan B ) tan A + tan B + 1 + tan A tan B
= =

Since, tan ( A + B ) = tan 225° ⇒ tan A + tan B = 1 − tan A tan B


1 1 1
Hence,
tan A + tan B + 1 + tan A tan B 1 − tan A tan B + 1 + tan A tan B 2
= =

91. Ans. (c), Given, cos ( A + B ) = α cos A cos B + β sin A sin B


But cos ( A + B ) = cos A cos B − sin A sin B ⇒ α = 1, β = −1
92. Ans. (c), Given, cos2 ( A − B ) + cos2 B − 2 cos ( A − B ) cos A cos B

= cos 2 ( A − B ) + cos 2 B − cos ( A − B ) {cos ( A − B ) + cos ( A + B )}

= cos2 B − cos ( A − B ) cos ( A + B ) = cos2 B − ( cos 2 A − sin 2 B ) = 1 − cos2 A = sin 2 A


Hence, it depends on A .
93. Ans. (c), cos12° + cos84° + cos156° + cos132°
1
= ( cos12° + cos132° ) + ( cos84° + cos156° ) = 2 cos 72° cos 60° − cos 36°
2
1 1  5 −1 5 + 1 1
= 2 cos 72°× − × cos 36°  = [ cos 72° − cos 36°] = 
2 2  4 4  2
 
− =−

3
 

94. Ans. (a), Given, cos ( A − B ) = ⇒ 5cos A cos B + 5sin A sin B = 3 …(i)
5
and sin A sin B = 2 cos A cos B …(ii)
1 1  2
∴ cos A cos B = and 5  + 1 sin A sin B = 3 ⇒ sin A sin B =
5 2  5
95. Ans. (c), Given equation cos x + cos y + cos α = 0 and sin x + sin y + sin α = 0.
The given equation may be written as cos x + cos y = − cos α and sin x + sin y = − sin α .

Therefore, 2 cos   cos   = − cot α …(i)


 2  2
x+ y x− y
 

and 2 sin   cos   = − sin α …(ii)


 2  2 
x+ y x− y

2 cos   cos 
2  2  cos α
 x+ y  x− y
Divide (i) by (ii), we get, ⇒ cot   = cot α

 x − y  sin α  2 
x+ y
2sin   cos 
  =

 2   2 
 x+ y

5 3
96. Ans. (b), tan A = = − tan C , cos B = − = − cos D
12 5

Office.: 606 , 6 Floor, Hariom Tower, Circular Road, Ranchi-1, Ph.: 0651-2562523, 9835508812, 8507613968
th
( BY R. K. MALIK’S NEWTON CLASSES
) 13
∴ In a cyclic quadrilateral A + C = π , B + D = π
5 12 3 4
∴ tan C = − ⇒ cos C = − ⇒ cos D = ⇒ tan D =
12 13 5 3
=α =β

Equation whose roots are cos C and tan D is x 2 − ( cos C + tan D ) x + ( cos C.tan D ) = 0
 12 4   12 4 
⇒ x2 −  − +  x +  − ×  = 0 ⇒ 39 x 2 − ( −36 + 52 ) x − 48 = 0 ⇒ 39 x 2 − 16 x − 48 = 0
 13 3   13 3 
tan A + tan B
97. Ans. (a), ∵ tan A + tan B = p, tan A tan B = q ; ∴ tan ( A + B ) =
1 − tan A tan B 1 − q
p
=

1 1 1
∴ sin .
2 p2
cosec2 ( A + B ) 1 + cot 2 ( A + B )  1− q 
2
p 2 + (1 − q )
2
( A + B) =
1+ 
= = =

98. Ans. (b), a sin 2 x + b cos 2 x = c


 p 

Dividing both side by cos 2 x , we get, a tan 2 x + b = c sec 2 x

⇒ a tan 2 x + b = c (1 + tan 2 x ) ⇒ tan 2 x ( a − c ) = c − b ⇒ tan 2 x =


c−b
a−c

tan y  b − d  ( d − a )( a − c ) ( a − d )( c − a )
d −a
∴ Similarly, tan 2 y = 
2 2

tan 2 x  c − b  ( b − d )( c − b ) ( b − c )( d − b )
d −a a
 ∴ 2 = = = =
b−d  b
 

1 3
 a−c 

99. Ans. (b), Given, sin 2θ + sin 2φ …(1) and cos 2θ + cos 2φ = …(2)
2 2
=

Square and adding,


1 9
∴ ( sin 2
2θ + cos 2 2θ ) + ( sin 2 2φ + cos 2 2φ ) + 2 [ sin 2θ sin 2φ + cos 2θ cos 2φ ] =
4 4
+

10 1 1
⇒ 2 [ sin 2θ sin 2φ + cos 2θ cos 2φ ] = − 2 = ⇒ cos 2θ cos 2φ + sin 2θ sin 2φ =
4 2 4
1 1 5
⇒ cos ( 2θ − 2φ ) = ⇒ cos 2 (θ − φ ) = ⇒ cos 2 (θ − φ ) = ∵ cos 2θ = 2 cos 2 θ − 1
4 4 8
1
100. Ans. (b), We have, tan α = and tan β = .
m +1 2m + 1
m

1
tan α + tan β
We know that tan (α + β ) = = m + 1 2m + 1
m

1
+
1 − tan α tan β 1 − m
( m + 1) ( 2m + 1)
2m 2 + m + m + 1 2m 2 + 2m + 1
=1 ⇒ tan (α + β ) = tan . Hence, α + β = .
2m 2 + m + 2m + 1 − m 2m 2 + 2m + 1 4 4
π π
= =

101. Ans. (b), cos   = 2 cos 


 2  2 
α − β  α + β 

⇒ cos cos + sin sin = 2 cos cos − 2sin sin


2 2 2 2 2 2 2 2
α β α β α β α β

1
⇒ 3sin sin = cos cos ⇒ tan tan
2 2 2 2 2 2 3
α β α β α β
=

Office.: 606 , 6th Floor, Hariom Tower, Circular Road, Ranchi-1, Ph.: 0651-2562523, 9835508812, 8507613968
14 ( BY R. K. MALIK’S NEWTON CLASSES
)
102. Ans. (b), ∵ A + B + C = π ⇒ A = π − ( B + C )

∴ cos π − ( B + C )  = cos B cos C ⇒ − cos ( B + C ) = cos B cos C

⇒ − [ cos B cos C − sin B sin C ] = cos B cos C ⇒ sin B sin C = 2 cos B cos C ⇒ tan B tan C = 2

103. Ans. (b), Given, ( cos x + sin x ) + k sin x cos x − 1 = 0, ∀x


2

⇒ cos 2 x + sin 2 x + 2 cos x sin x + k sin x cos x − 1 = 0, ∀x


⇒ ( k + 2 ) cos x sin x = 0, ∀x ⇒ k +2=0 ⇒ k = −2
1 13
104. Ans. (b), Given, cos P = , cos Q = ∴ cos ( P − Q ) = cos P cos Q + sin P sin Q
7 14
1 13 48 27 13 + 36 1
= . + . = = cos 60° ⇒ P − Q = 60° .
7 14 7 14 98 2
=

105. Ans. (d), We have, cos A = cos B cos C .


By triangle property, A + B + C = π ⇒ B+C =π − A ∴ cos ( B + C ) = cos (π − A)
⇒ cos B cos C − sin B sin C = − cos A = − cos B cos C [∵ Given cos A = cos B cos C ]
cos B cos C 1 1
⇒ 2 cos B cos C = sin B sin C ⇒ cot B cot C =
sin B sin C 2 2
⇒ =

1
106. Ans. (b), Given, x+ = 2 cos θ …(1)

1 1
x

On squaring both sides, we get, x + + 2 = 4 cos 2 θ ⇒ x + = 4 cos 2 θ − 2

1
= 2 ( 2 cos 2 θ − 1) = 2 cos 2θ …(2)
x x

⇒ x+

1
Again squaring both sides, x 2 + 2 + 2 = 4 cos 2 2θ
x

1 1
⇒ x 2 + 2 = 4 cos 2 2θ − 2 = 2 ( 2 cos 2 2θ − 1) ⇒ x 2 + 2 = 2 cos 4θ …(3)
x

1 
x x
3

Now taking cube of both sides,  x 2 + 2  = 8cos3 4θ


1 1  1  1
 x 

⇒ x6 + 6
+ 3 x 2 . 2  x 2 + 2  = 8cos3 4θ ⇒ x 6 + 6 + 3 ( 2 cos 4θ ) = 8cos3 4θ

1
x x  x  x

⇒ x 6 + 6 = 8cos3 4θ − 6 cos 4θ = 2 ( 4 cos 3 4θ − 3cos 4θ ) = 2 cos 3 ( 4θ ) ∵ cos 3θ = 4 cos3 θ − 3cos θ 

= 2 cos12θ
x

107. Ans. (a), A.M. ≥ G.M


1 tan 2 α   2 tan 2 α  tan 2 α
1/ 2

∴  x2 + x + . x2 + x + ≥ 2 tan α
2 x2 + x   x2 + x  x2 + x

  x + x  ⇒

108. Ans. (c), cosec2 α + 2 cot α = 1 + cot 2 α + 2 cot α = 1 + cot α .



But ⇒ cot α < −1 . ⇒ 1 + cot α < 0 . Hence, 1 + cot α = − (1 + cot α )
4
<α <π

1
109. Ans. (b), We have, x + = 2 cos θ
x

Office.: 606 , 6 Floor, Hariom Tower, Circular Road, Ranchi-1, Ph.: 0651-2562523, 9835508812, 8507613968
th
( ) BY R. K. MALIK’S NEWTON CLASSES 15

1  1 1 1
3

Now, x + 3 =  x +  − 3 x  x +  = ( 2 cos θ ) − 3 ( 2 cos θ ) = 8cos3 θ − 6 cos θ


3 3

= 2 ( 4 cos 3 θ − 3cos θ ) ∵ cos 3θ = 4 cos3 θ − 3cos θ 


x  x x x

= 2 cos 3θ
110. Ans. (a), We have cos 2 (θ + φ ) + 4 cos (θ + φ ) sin θ sin φ + 2sin 2 φ

= 2 cos 2 (θ + φ ) − 1 + 4 cos (θ + φ ) sin θ sin φ + 1 − cos 2φ ∵ cos 2θ = 2 cos 2 θ − 1 = 1 − 2sin 2 θ 

= 2 cos (θ + φ ) ( cos (θ + φ ) + 2sin θ sin φ ) − cos 2φ


= 2 cos (θ + φ )( cos θ cos φ − sin θ sin φ + 2sin θ sin φ ) − cos 2φ
= 2 cos (θ + φ ) cos (θ − φ ) − cos 2φ = cos 2θ + cos 2φ − cos 2φ = cos 2θ
21 27
111. Ans. (d), sin α + sin β = − , cos α + cos β = −
65 65
 21   27 
2 2

Now, ( sin α + sin β ) + ( cos α + cos β ) =  −  +  − 


2 2

 65   65 
441 729
⇒ 2 + 2 sin α sin β + 2 cos α cos β = 2 + 2
65 65
1170  α − β  1170
⇒ 2 + 2 cos (α − β )  = ⇒ 2.2 cos 2  ∵ 1 + cos θ = 2 cos 2 
2θ 

( 65 )  2  ( 65 )

2 = 2

 3 130 3 3  π α − β 3π 
⇒ cos  . Therefore, cos 
 2 130 130  2 130 ∵ 2 < 2 < 2 
α − β α − β 
= = =−
 

2sin cos 2 tan


sin θ 2 2 = 2
112. Ans. (b), tan θ = ⇒ tan θ =
θ θ θ

cos θ 1 − 2sin 2
1 − tan 2

2 2
θ θ

x −1 x −1 x +1
Using sin ; ∴ cos = 1 − sin 2 ∴ tan θ = x 2 − 1
and tan =
2 2x 2 2 x +1 2x 2
θ θ θ θ
= =

113. Ans. (c), Here cos α + cos β = − p, cos α cos β = q , sin α + sin β = − r , sin α sin β = s
Then, p 2 + r 2 = 2 + 2 ( cos α cos β + sin α sin β ) = 2 + 2 cos (α − β )
Also, q + s = cos (α − β ) ∴ p 2 + r 2 = 2 ( q + s ) + 2
1 1
114. Ans. (c), Given, a = cos 2 θ + sin 4 θ = (1 + cos 2θ ) + (1 − cos 2θ )
2

2 4
1 1 1 1 1 1 1
= (1 + cos 2θ ) + (1 − 2 cos 2θ + cos 2 2θ ) = + cos 2θ + − cos 2θ + (1 + cos 4θ )
2 4 2 2 4 2 8
3 1 1 7 cos 4θ 7
+ + cos 4θ = + . So, cos 4θ = 8  a − 
4 8 8 8 8 8

=

1 7 1 6 3

Now, −1 ≤ cos 4θ ≤ 1 ⇒ − ≤ a − ≤ ⇒ ≤ a ≤1 ⇒ ≤ a ≤1
8 8 8 8 4
115. Ans. (c), Here x 2 + y 2 = x ' 2 + y ' 2 and xy = ( x '2 − y '2 ) sin θ cos θ − x ' y ' ( cos 2 θ − cos2 θ )

Hence, x 2 + 4 xy + y 2 = x '2 + y '2 + 2 ( x '2 − y '2 ) sin 2θ − 4 x ' y 'cos 2θ

Office.: 606 , 6th Floor, Hariom Tower, Circular Road, Ranchi-1, Ph.: 0651-2562523, 9835508812, 8507613968
16 ( BY R. K. MALIK’S NEWTON CLASSES
)
= (1 + 2sin 2θ ) x ' + (1 − 2sin 2θ ) y '2 − 4 cos 2θ .x '. y ' , which is identical with ax ' 2 + by '2
2

Thus, a = 1 + 2 sin 2θ , b = 1 − 2 sin 2θ , cos 2θ = 0 . Now cos 2θ = 0 ⇒ θ = and so a = 3, b = −1


4
π

+ tan tan
116. Ans. (d), tan = tan  −  ⇒ 2 2 = cot C
A B

2 2 2 2
A+ B π C 
1 − tan tan
2 2
A B

⇒ tan tan + tan tan + tan tan = 1 [Remember this step]


2 2 2 2 2 2
A C B C A B

sin 2α 4 sin 2α + sin 2 β 5


117. Ans. (a),
sin 2 β 1 sin 2α − sin 2 β 3
= ⇒ =

2 sin (α + β ) cos (α − β ) 5 tan (α + β ) 5


2 sin (α − β ) cos (α + β ) 3 tan (α − β ) 3
⇒ = ⇒ =

1 3π 1 5π 1  21π 3π 
118. Ans. (d), The given expression = cos + cos −  cos + cos 
2 13 2 13 2  13 13 
1 5π 1 5π  1 5π 1 5π 1
= .cos − cos  2π −  = .cos − .cos = 0 = k . So, is undefined.
2 13 2 13  2 13 2 13

cos 4 x − sin 4 x 2 cos x − sin x


 k
2 2

119. Ans. (b), Here, cot x cos x − tan x sin x = = 2 cot 2 x ⇒k =2


2 2 ( )
sin x cos x sin 2 x
=

120. Ans. (c), Given, k cos 4 θ = 3 + 8cos 2 θ − 4 + ( 2 cos 2 2θ − 1) = −2 + 8cos 2 θ + 2 ( 2 cos 2 θ − 1)


2

= −2 + 8 cos 2 θ + 8cos 4 θ − 8cos 2 θ + 2 = 8cos 4 θ ⇒ k =8

121. Ans. (d), sin 4 + sin 4  −  + sin 4  +  + sin 4  π − 


8 2 8 2 8 8
π π π  π π   π

2

=  sin 4 + cos 4  +  cos 4 + sin 4  = 2  sin 2 + cos 2  − 2 sin 2 .cos 2 


8 8  8 8 8 8 8 8 
 π π  π π  π π π π 
 
 1 3
= 2 1 − .sin 2  = 2 1 −  =
 2 4  4 2
 1 π

122. Ans. (b), Here ( cos α + cos β ) + ( sin α + sin β ) = 1 ⇒ 2 + 2 cos (α − β ) = 1


2 2

1 1 1
⇒ cos (α − β ) = − ⇒ 2 cos 2 −1 = − ⇒ cos2
2 2 2 2 4
α −β α −β
=

123. Ans. (c), Here m + n = k ( cos θ + sin θ ) and ( m − n ) = k ( cos θ − sin θ )


3 3

( m + n ) 3 + ( m − n ) 3 = k 3 2 ( sin 2 θ + cos 2 θ ) = 2k 3
2 2 2 2

1
1−
tan 45° − tan 30° 3 = 3 −1
124. Ans. (b), We have, x =
1 + tan 45°. tan 30° 1 + 1 3 +1
=

3
1 3 −1 3 +1 4 3
So, x − = −2 3 or, x 2 + 2 3 x − 1 = 0
3 +1 3 −1 2
= − =−
x

Office.: 606 , 6 Floor, Hariom Tower, Circular Road, Ranchi-1, Ph.: 0651-2562523, 9835508812, 8507613968
th
( BY R. K. MALIK’S NEWTON CLASSES
) 17
99π 3π
125. Ans. (d), Given, ≤ θ ≤ 50π is equivalent to ≤ θ ≤ 2π
2 2
∴ x = − sin 2 θ and y = cos2 θ ⇒ y − x =1
cos x + cos ( x + 2θ ) 2 cos ( x + θ ) cos θ
126. Ans. (c),
b + d cos ( x + θ ) + cos ( x + 3θ ) 2 cos ( x + 2θ ) cos θ c
a+c b
= = =

2π 4π
cos cos
2π 4π 1 1 1 3 , 1 3
127. Ans. (b), Given, x = y cos = z cos = k (say) ⇒ = , =
3 3
=

1 1 1 1 2π 4π
x k y k z k

+ + = 1 + cos + cos =0 ⇒ xy + yz + zx = 0


3 3


x y z k 

128. Ans. (a), ∵ 0 < α < ⇒ 0< (in first quadrant|), then tan α / 2 is always positive
6 2 12
π α π
<

2 tan 1 − tan 2
∵ sin α + cos α =
7 2 + 2 = 7
α α

2 1 + tan 2 1 + tan 2 2

2 2
α α

⇒ 2  2 tan + 1 − tan 2  = 7 1 + tan 2  7 + 2 tan 2 − 4 tan 7 −2 =0


2 2 2 2 2
 α α  α α α
 
⇒ ( ) + ( )
4 ± 16 − 4 7 +2 7 −2 ) = 4± 16 − 12
∴ tan
( )(
2 2( 7 + 2) 2( 7 +2
α
=
)
4±2 2 ±1 1 3 7 −2
or or 7 −2
2 7 +2 7 +2 7 +2 7 +2 3
= = = =
( ) ( ) ( )
7 −2
Hence, tan (α / 2 ) = ∵ 0 <  2  < 12 
3
 α  π 
   

129. Ans. (b), ∵ = k (say)


cos α 2π  2π 
cos  α − cos  α +
x y z
= =

3  3 
 
 

2π 2π 2π 
 

∴ x + y + z = k cos α + cos  α −  + cos  α +   = k cos α + 2 cos α cos


3 3 3 
     

1
     

= k cos α + 2 cos α × −  = 0
2

130. Ans. (b), cos α + cos β = sin α + sin β


On squaring we get, cos 2 α + cos 2 β + 2 cos α cos β = sin 2 α + sin 2 β + 2sin α sin β
⇒ cos 2α + cos 2 β + 2 cos (α + β ) = 0 ⇒ cos 2α + cos 2 β = −2 cos (α + β )

131. Ans. (c), ∵ 4nα = π and tan ( 2n − 1) α = tan ( 2nα − α ) = tan  − α  = cot α
2
π 

and, tan ( 2n − 2 ) α = tan ( 2nα − 2α ) = tan  − 2α  = cot 2α ….


2
π 

∴ tan α tan 2α tan 3α ......tan ( 2n − 1) α ( tan α tan ( 2n − 1) α ) ( tan 2α tan ( 2n − 2 ) α ) ..tan nα


Office.: 606 , 6th Floor, Hariom Tower, Circular Road, Ranchi-1, Ph.: 0651-2562523, 9835508812, 8507613968
18 ( ) BY R. K. MALIK’S NEWTON CLASSES
= (1.1.1..1) tan nα = tan nα = tan =1
4
π

132. Ans. (d), ∵ −  −π < 2 x < − 2 i.e., in II quadrant 


2 4
π π  π 
<x<−

⇒ cos x + sin x = a ,
Squaring both sides, we get, cos2 x + sin 2 x + 2 cos x sin x = a 2

⇒ sin 2 x = ( a 2 − 1) ⇒ cos 2 x = 1 − a 2 − 1 = a2 (2 − a2 ) = a (2 − a )
2 2
( )
133. Ans. (b), Given, tan x tan y = a and tan ( x + y ) = tan  
6
π 

tan x + tan y 1 1
⇒ tan x + tan y = (1 − a )
1 − tan x tan y 3 3
⇒ =

Equation whose roots are tan x and tan y is, x 2 −


(1 − a ) x + a = 0 ⇒ 3 x 2 − (1 − a ) x + a 3 = 0
3
134. Ans. (d), Given that, tan θ − cot θ = a …(i) and sin θ + cos θ = b …(ii)

Now, b 2 − 1 + 4 = ( sin θ + cos θ ) − 1 } {( tan θ − cot θ ) + 4}


2 2 2 2
2
( ) (a ) {
= [1 + sin 2θ − 1]  tan 2 θ + cot 2 θ − 2 + 4  = [sin 2θ ]  tan 2 θ + 1 + cot 2 θ + 1
2 2

 1 1 
= sin 2 2θ cosec2θ + sec 2 θ = 4 sin 2 θ cos 2 θ  2 + =4
 sin θ cos θ 
( ) 2

2sin .cos
sin A + sin B C 2 2 =C
135. Ans. (d), As given, ⇒ tan
A+ B A− B

cos A + cos B D 2
A+ B C
2 cos .cos
= ⇒ =

2 2
A+ B A− B D D

2 tan 2
Thus, sin ( A + B ) = 2 D = 2CD
A+ B C

1 + tan 2 1+ 2
C 2 C 2 + D2
=

2
A+ B

136. Ans. (a), Given, sin θ + sin 3θ + sin 2θ = sin α


D

⇒ 2 sin 2θ cos θ + sin 2θ = sin α ⇒ sin 2θ ( 2 cos θ + 1) = sin α …(i)


and cos θ + cos 3θ + cos 2θ = cos α
⇒ 2 cos 2θ cos θ + cos 2θ = cos α ⇒ cos 2θ ( 2 cos θ + 1) = cos α …(ii)

From (i) and (ii) tan 2θ = tan α ⇒ 2θ = α ⇒ θ =


2
α

137. Ans. (b) sin θ + cos θ = p ⇒ (sin θ + cos θ )2 = p 2 ⇒ 1 + 2 sin θ cos θ = p 2


⇒ 1 + sin 2θ = p 2 ⇒ sin 2θ = p 2 − 1
1 1
∴ sin 4 θ + cos 4 θ = (sin 2 θ + cos 2 θ )2 − 2sin 2 cos 2 θ = 1 − (sin 2θ ) = 1 − ( p 2 − 1) 2 .
2 2
138. Ans. (a) sin θ + cosec θ = 2
1
⇒ sin θ + = 2 ⇒ sin 2 θ − 2sin θ + 1 = 0 ⇒ (sin θ − 1) 2 = 0 ⇒ sin θ = 1 .
sin θ

Office.: 606 , 6 Floor, Hariom Tower, Circular Road, Ranchi-1, Ph.: 0651-2562523, 9835508812, 8507613968
th
( BY R. K. MALIK’S NEWTON CLASSES
) 19
1
∴ sin n θ + cosec n θ = (sin θ ) n + = 1n + 1n = 1 + 1 = 2
(sin θ ) n

1 + tan cos + sin


139. Ans. (d) tan  +  = 2 = 2 2
A A A

 4 2  1 − tan
π
cos − sin
A

2 2 2
A A A

 cos + sin 1 + 2sin − sin


2 2 2 2 = 1 + sin A .
 A A A A

1 − 2sin cos 1 − sin A


=  2

 cos − sin
=

2 2 2 2
 A A A A

140. Ans. (c) 3 sin 75° − cos 75°


( )
 3 1
= 2  sin 75° − cos 75°  = 2 ( sin 75° cos 30° − cos 75° sin 30° ) = 2sin ( 75° − 30° )
 2 2

1

= 2 sin 45° = 2. = 2.
2
1 − tan α
141. Ans. (c) α + β = − α ⇒ tan β =
4 4 1 + tan α
π π
⇒β=

1 − tan α 2
⇒ 1 + tan β = + 1 ⇒ 1 + tan β = .
1 + tan α 1 + tan α

142. Ans. (b) tan  + θ  + tan  − θ  = p sec 2θ


4 4
π  π 

1 + tan θ 1 − tan θ (1 + tan θ )2 + (1 − tan θ )2


 

= p sec 2θ ⇒ = p sec 2θ
1 − tan θ 1 + tan θ 1 − tan 2 θ
⇒ +

2(1 + tan 2 θ ) 2
= p sec 2θ ⇒ = p sec θ ⇒ 2 sec 2θ = p sec 2θ ⇒ p = 2.
1 − tan θ
2
cos 2θ
 1 θ 1
143. Ans. (b) sin + cos = 2 sin + cos 
2 2  2 2 2 2
θ θ θ

= 2  sin cos 45° + cos sin 45°  = 2 sin  + 45° 


2 2 2
 θ θ  θ 
  

Now, 270° < θ < 360° ⇒ 135° < < 180° ⇒ 180° < + 45° < 225°
2 2
θ θ

+ 45° lies in IIIrd quadrant and so sin  + 45°  is – ve


2 2
θ θ 

Hence, sin + cos


has a –ve sign.
2 2
θ θ

144. Ans. (b) (1 + sin α ) (1 + sin β ) (1 + sin γ ) = (1 − sin α ) (1 − sin β ) (1 − sin γ ) = k


⇒ k 2 = k .k = (1 + sin α ) (1 + sin β ) (1 + sin γ ) (1 − sin α ) (1 − sin β ) (1 − sin γ )
⇒ k 2 = (1 − sin 2 α )(1 − sin 2 β )(1 − sin 2 γ ) ⇒ k 2 = cos 2 α .cos 2 β .cos 2 γ ⇒ k = ± cos α cos β cos γ .
145. Ans. (a) cos α sin( β − γ ) + cos β sin(γ − α ) + cos γ sin(α − β )
= cos α {sin β cos γ − cos β sin γ } + cos β {sin γ cos α − cos γ sin α } + cos γ {sin α cos β − cos α sin β }
= cos α sin β cos γ − cos α cos β sin γ + cos α cos β sin γ

Office.: 606 , 6th Floor, Hariom Tower, Circular Road, Ranchi-1, Ph.: 0651-2562523, 9835508812, 8507613968
20 ( ) BY R. K. MALIK’S NEWTON CLASSES
− sin α cos β cos γ + sin α cos β cos γ − cos α sin β cos γ = 0.
146. Ans. (b) To find the correct choice, it is celar that we have to climinate θ

Hence adding and subtracty the given equations we get tan θ = , sin θ =
2 2
α +β α −β

Also we know that cosec 2 θ − cot 2 θ = 1


 2   2 
2 2

 = 1 ⇒ 4 (α + β ) − (α − β )  = α − β ⇒ α 2 − β 2 = ±4 αβ
2 2 2
2 2
⇒  − ( )
147. Ans. (b) sin θ , cos θ and tan θ are in G.P.
α − β  α + β 

⇒ cos 2 θ = sin θ . tan θ ⇒ cos3 θ = sin 2 θ …(i)


cos 6 θ cos 2 θ sin 4 θ cos 2 θ
Now, cot 6 θ − cot 2 θ = [Using (i)]
sin 6 θ sin 2 θ sin 6 θ sin 2 θ
− = −

1 cos 2 θ 1 − cos 2 θ sin 2 θ


= 1.
sin 2 θ sin 2 θ sin 2 θ sin 2 θ
= − = =

148. Ans. (b) cos(θ + φ ) = m cos(θ − φ )


cos(θ − φ ) 1 cos(θ − φ ) + cos(θ + φ ) 1 + m
[By Compenendo-dividendo]
cos(θ + φ ) m cos(θ − φ ) − cos(θ + φ ) 1 − m
⇒ = ⇒ =

2 cos θ cos φ 1 + m  1+ m   1− m 
⇒ cot θ cot φ =   ⇒ tan θ =   cot φ .
2sin θ sin φ 1 − m  1− m   1+ m 
⇒ =

149. Ans. (a) cos A + cos B = 1


 A− B  1
⇒ 2 cos   cos   = 1 ⇒ 2 cos   cos   = 1 ⇒ cos 
 2   2  6  2   2  3
 A+ B   A− B  π   A− B 
=

Now, cos( A − B ) = 2 cos 2   − 1 [∵cos 2 A = 2 cos A − 1]


2

 2 
 A− B 

1 1
= 2. − 1 = − .
3 3

150. Ans. (a) cot(α + β ) = 0 = cot


2 2 2
π π π
⇒α +β = ⇒β= −α

∴ sin(α + 2 β ) = sin(α + π − 2α ) = sin(π − α ) = sin α .


151. Ans. (b) Let α = 12 sin θ − 9 sin 2 θ = 4 − (4 − 12sin θ + 9sin 2 θ ) = 4 − (2 − 3sin θ ) 2 .
Clearly, (2 − 3sin θ )2 ≥ 0
∴ α has the maximum value when (2 − 3sin θ ) 2 has the minimum value i.e., when (2 − 3sin θ )2 = 0
and so the maximum value of α is 4.
152. Ans. (c) A + B + C = 180°

tan + tan
⇒ A + B = 180° − C ⇒ = 90° − ⇒ tan  +  = tan  90° −  ⇒ 2 2 = cot C
A B

2 2 2 2 2 2
A+ B
1 − tan tan
C  A B   C 

2 2
 A B

1 2
C 9 C 7
⇒ 3 3 = cot ⇒ cot = ⇒ tan = .
1 2
+

1− . 2 2 7 2 9
C

3 3

Office.: 606 , 6 Floor, Hariom Tower, Circular Road, Ranchi-1, Ph.: 0651-2562523, 9835508812, 8507613968
th
( BY R. K. MALIK’S NEWTON CLASSES
) 21
153. Ans. (c) We have: 2 x + 3 x = 5 x
tan 2 x + tan 3 x
⇒ tan(2 x + 3 x) = 5 x ⇒ = tan 5 x ⇒ tan 2 x + tan 3 x = tan 5 x − tan 2 x tan 3 x tan 5 x
1 − tan 2 x tan 3 x
⇒ tan 5 x − tan 3 x − tan 2 x = tan 2 x tan 3 x tan 5 x .
cos 2 θ cos θ .sin θ − sin θ
154. Ans. (b) f (θ ) = cos θ .sin θ sin θ
2
cos θ
sin θ − cos θ 0
[C1 → C1 − (sin θ ). C3 , C2 → C2 + (cos θ ).C3 ]
1 0 − sin θ
0 1 cos θ = 1.(0 + cos 2 θ ) + sin θ .(0 + sin θ ) = cos 2 θ + sin 2 θ = 1 .
sin θ − cos θ 0
=

 π − 2α 
155. Ans. (a) 4 cos 2   + 4 sin α + sin 2α
4 2

 4 
 π − 2α  
= 2 1 + cos    + 4 sin α + 4sin α cos α = 2 1 + cos  − α   + 4sin α (sin α + cos α )
4 2 2 2 2 2

 2  2
  π 

= 2(1 + sin α )+ | 2 sin α | = 2 + 2 sin α + (−2 sin α )


  

∵ When α lies in 3rd quadrant, then 


 sin α is − ve and so |sinα | = − sin α  = 2

156. Ans. (b) cos 2(θ + φ ) + 4 cos(θ + φ ) sin θ sin φ + 2sin 2 φ


 

= {cos 2θ cos 2φ − sin 2θ sin 2φ} +4{(cos θ cos φ − sin θ sin φ ) sin θ sin φ} + 2sin 2 φ
= {(1 − 2sin 2 θ )(1 − 2sin 2 φ ) − 4 sin θ cos θ sin φ cos φ } +4 cos θ cos φ sin θ sin φ − 4sin 2 θ sin 2 φ + 2sin 2 φ
= 1 − 2sin 2 θ − 2sin 2 φ + 4sin 2 θ sin 2 φ − 4 sin 2 θ sin 2 φ + 2sin 2 φ = 1 − 2sin 2 θ = cos 2θ .
cos θ cos u − e
157. Ans. (c)
1 1 − e cos u
=

2 sin 2
1 − cos θ 1 − e cos u − cos u + e 2 = (1 + e) (1 − cos u )
θ

1 + cos θ 1 − e cos u + cos u − e 2 cos 2


θ (1 − e) (1 + cos u )
⇒ = ⇒

2
1+ e 1+ e
⇒ tan 2 . tan 2   ⇒ tan = ± . tan .
2 1− e 2 2 1− e 2
θ u θ u
=

1 + sin x − cos x
158. Ans. (d)
1 + sin x
(1 + sin x − cos x) (1 + sin x + cos x) 1 + sin x ) − cos 2 x
2

(1 + sin x) (1 + sin x + cos x) (1 + sin x )(1 + sin x + cos x )


= =
(

(1 + sin x) 2 − (1 − sin 2 x) (1 + sin x){(1 + sin x) − (1 − sin x)} 2sin x


= θ.
(1 + sin x) (1 + sin x + cos x) (1 + sin x) (1 + sin x + cos x) (1 + sin x + cos x)
= = =

159. Ans. (c) tn − tn − 2 = (sin n θ + cos n θ ) − (sin n − 2 θ + cos n − 2 θ )


= sin n − 2 θ (sin 2 θ − 1) + cos n − 2 θ (cos 2 θ − 1) = sin n − 2 θ (− cos 2 θ ) + cos n − 2 θ (− sin 2 θ )
= − sin 2 θ cos 2 θ (sin n − 4 θ + cos n − 4 θ ) = − sin 2 θ cos 2 θ . tn − 4 = ktn − 4 where k = − sin 2 θ cos 2 θ .

Office.: 606 , 6th Floor, Hariom Tower, Circular Road, Ranchi-1, Ph.: 0651-2562523, 9835508812, 8507613968 21
22 ( BY R. K. MALIK’S NEWTON CLASSES
)
160. Ans. (c) 4(cos θ − sin θ )
8 8

= 4(cos 4 θ − sin 4 θ ) (cos 4 θ + sin 4 θ )


= 4(cos 2 θ + sin 2 θ ) (cos 2 θ − sin 2 θ ) {(cos 2 θ + sin 2 θ )2 − 2 sin 2 θ cos 2 θ }
 1  (1 − cos 4θ )   3 cos 4θ 
= 4.1cos 2θ . 1 − sin 2 2θ  = 4 cos 2θ 1 −  = 4 cos 2θ  +  = cos 2θ (cos 4θ + 3)
 2 4 4 4 

161. Ans. (c) (1 + cos α ) (1 + cos β ) (1 + cos γ ) = λ


  

⇒ 8cos 2 cos 2 cos 2 …(i)


2 2 2
α β γ

(1 − cos α ) (1 − cos β ) (1 − cos γ ) = λ

⇒ 8sin 2 sin 2 sin 2 …(ii)


2 2 2
α β γ

2 2 2

Multiplying (i) and (ii) we get ; λ 2 =  2 sin cos  .  2sin cos  .  2sin cos 
2 2 2 2 2 2
 α α  β β  γ γ

⇒ λ 2 = sin 2 α .sin 2 β .sin 2 γ ⇒ | λ | = | sin α .sin β .sin γ |


  

Another Method:
(1 + cos α ) (1 + cos β ) (1 + cos γ ) = λ = (1 − cos α ) (1 − cos β ) (1 − cos γ )
⇒ λ 2 = {(1 + cos α ) (1 + cos β ) (1 + cos γ )} {(1 − cos α ) (1 − cos β ) (1 − cos γ )}
= {(1 + cos α ) (1 − cos α )}{(1 + cos β ) (1 − cos β )} {(1 + cos γ ) (1 − cos γ )}
= (1 − cos 2 α ) (1 − cos 2 β ) (1 − cos 2 γ ) = (sin 2 α ) (sin 2 β ) (sin 2 γ ) ⇒ λ = | sin α sin β sin γ | .
162. Ans. (c) (1 − tan 2 θ ) (1 − 3 tan 2 θ ) (1 + tan 2θ tan 3θ )
2 tan θ 3 tan θ − tan 3 θ 
= (1 − tan 2 θ ) (1 − 3 tan 2 θ ) 1 + .
 1 − tan θ 1 − 3 tan θ 

2 2 

2 x 3x − x3 
= (1 − x 2 ) (1 − 3x 2 ) 1 + . where x = tan θ
1 1 3

2 2 
− −
= (1 − x 2 ) (1 − 3 x 2 ) + 2 x.(3 x − x3 ) = 1 − 4 x 2 + 3 x 4 + 6 x 2 − 2 x 4
 x x 

= x 4 + 2 x 2 + 1 = ( x 2 + 1) 2 = (tan 2 θ + 1) 2 = (sec2 θ ) 2 = sec4 θ .


163. Ans. (b) sin θ + sin φ = a …(i)
cos θ + cos φ = b …(i)
Squaring both sides and adding (i) and (ii), we get
(sin 2 θ + sin 2 φ + 2sin θ sin φ ) + (cos 2 θ + cos 2 φ + 2 cos θ cos φ ) = a 2 + b 2
⇒ (sin 2 θ + cos 2 θ ) + (sin 2 φ + cos 2 φ ) + 2(cos θ cos φ + sin θ sin φ ) = a 2 + b 2
⇒ 1 + 1 + 2 cos(θ − φ ) = a 2 + b 2 ⇒ 2{1 + cos(θ − φ )} = a 2 + b 2
4 4
⇒ 4 cos 2   = a + b ⇒ sec 
2 2 2  θ −φ 
⇒ tan 2  −1
 2   2  a +b  2  a +b
 θ −φ   θ −φ 
= 2 2 = 2 2

4 − a 2 − b2
⇒ tan  .
 2 
θ −φ 
a2 + b2
 =

164. Ans. (d) sin α + sin β = a

22 Office.: 606 , 6th Floor, Hariom Tower, Circular Road, Ranchi-1, Ph.: 0651-2562523, 9835508812, 8507613968
( BY R. K. MALIK’S NEWTON CLASSES
) 23

⇒ 2 sin   cos  …(i)


 2   2 
α + β  α − β 
=a

cos α + cos β = b

⇒ 2 cos   cos  …(ii)


 2  2
α + β  α − β 
=b
 

Dividing (i) by (ii) we get: tan 


 2
α + β  a
=

2a
 b

2 tan 
 2  = b = 2ab .
α + β 
Now, sin(α + β ) =

1 + tan 2   1+ 2
a 2 a 2 + b2
 2 
α + β 

1 − cos 2α
b

165. Ans. (d) tan 2 α =


1 + cos 2α
3cos 2 β − 1
1−
3 − cos 2 β − 1 4 − 4 cos 2 β  1 − cos 2 β   2sin 2 β
= 2 2.  = 2 tan β ⇒ tan α = ± 2 tan β .
3cos 2 β − 1
2

2 + 2 cos 2 β  1 + cos 2 β   2 cos β



1+
2
= =

3 − cos 2 β
 

166. Ans. (b) cos 20° + cos100° + cos140°


= 2 cos 60° cos 40° + cos(180° − 40°) = cos 40° − cos 40° = 0 .

 1  1 − cos15° 1 − cos A 
167. Ans. (d) tan  7 °  = ∵tan =
 2  1 + cos15° 2 1 + cos A 
 A

3 +1 
1− 
2 2  2 2 − (1 + 3) 2 2 − (1 + 3)  Multiplying Nr. & Dr. by 

3 +1  2 2 + (1 + 3) 4−2 3  2 2 − (1 + 3)

1+ 
= = =  

2 2 
 

2 2 − (1 + 3) 2 2 − (1 + 3)

.
( 3 − 1) 2 ( 3 − 1)
= =

2π 3π 4π  2π 4π  3π
168. Ans. (c) cos cos cos cos =  cos cos cos  .cos
9 9 9 9  9 9 9  9
π π

sin  23.  8π
sin
9 9 = 1
 π
.cos =
3 8sin π 16
π
23.sin
= 

9 9
π

169. Ans. (c) 6(sin 6 θ + cos 6 θ ) − 9(sin 4 θ + cos 4 θ ) + 4


= 6{(sin 2 θ + cos 2 θ )3 − 3sin 2 θ cos 2 θ (sin 2 θ cos 2 θ )} −9{(sin 2 θ + cos 2 θ ) 2 − 2sin 2 θ cos 2 θ } + 4
= 6{1 − 3sin 2 θ cos 2 θ } − 9{1 − 2sin 2 θ cos 2 θ } + 4 = 1.
170. Ans. (a) 2 P6 − 3P4 + 1
= 2(cos 6 θ + sin 6 θ ) − 3(cos 4 θ + sin 4 θ ) + 1
= 2[(cos 2 θ + sin 2 θ ) (cos 4 θ + sin 4 θ − cos 2 θ sin 2 θ )] −3[(cos 2 θ + sin 2 θ ) 2 − 2sin 2 θ cos 2 θ ] + 1
= 2[1.{(cos 2 θ + sin 2 θ )2 − 3cos 2 θ sin 2 θ ] − 3[1 − 2sin 2 θ cos 2 θ ] + 1

Office.: 606 , 6th Floor, Hariom Tower, Circular Road, Ranchi-1, Ph.: 0651-2562523, 9835508812, 8507613968 23
24 ( BY R. K. MALIK’S NEWTON CLASSES
)
= 2[1 − 3cos θ sin θ ] − 3 + 6 sin 2 θ cos 2 θ + 1
2 2

= 2 − 6 cos 2 θ sin 2 θ − 3 + 6sin 2 θ cos 2 θ + 1 = 0.


sin( A − B ) sin( B − C ) sin(C − A)
171. Ans. (c)
sin A sin B sin B sin C sin C sin A
+ +

sin( A − B )  sin A cos B cos A sin B 


sin A sin B  sin A sin B sin A sin B 
=∑ = ∑ − 

∑ (cot B − cot A) = 0 .
1  a −b  1
172. Ans. (c) sin a + sin b = ⇒ 2sin   cos  …(i)
2  2   2  2
 a+b
=

3 3
cos a + cos b = ⇒ 2 cos   cos  …(ii)
2  2   2  2
 a+b   a −b 
=

1
2 tan  2×
 a+b  1  2  = 3 = 3
 a+b 
Dividing (i) by (ii) we get: tan  ⇒ sin ( a + b ) =
1

 2  3  a + b  1+ 2
1 + tan 2 
=
 2 
 3
173. Ans. (c) sin A + sin B = l

⇒ 2 sin   cos  …(i)


 2   2 
 A+ B   A− B 
=l

cos A − cos B = m

⇒ 2 sin   sin   = m ⇒ − 2 sin   sin  …(ii)


 2   2   2   2 
 A+ B   B − A  A+ B   A− B 
=m

1 − tan 2  1
m2
 2 =
 A− B 
Dividing (ii) by (i) we get: tan  cos( ) l2 = l − m .
 − 2 2

 2 
 A − B  −m
1 + tan 2  1
m2 l 2 + m2
= ∴ A − B =

 2 

 A− B 
l2
l
 +

174. Ans. (a) sin θ = sin15° + sin 45° = 2 sin 30° cos15°
 1
=  2 ×  sin 75° = sin 75° .
 2
3 + 2 cos A ( 3 + 2 cos A) ( 3 − 2 cos A)
175. Ans. (c)
1 − 2sin A (1 − 2sin A) ( 3 − 2cos A)
=

3 − 4 cos 2 A 3 − 4(1 − sin 2 A) 4sin 2 A − 1 (1 + 2sin A)


(1 − 2sin A) ( 3 − 2 cos A) (1 − 2sin A) ( 3 − 2 cos A) (2 sin A − 1) ( 3 − 2 cos A) ( 3 − 2 cos A)
= =− =−

 3 + 2 cos A   1 + 2 sin A   1 + 2sin A   1 + 2sin A 


−3 −3 −3 −3

 = 0.
 1 − 2 sin A   3 − 2 cos A   3 − 2cos A   3 − 2 cos A 
∴   +
  = −  +

176. Ans. (d), x cos θ + y sin θ A = A …(i)


x sin θ + y cos θ = B …(i)
Squaring (i) and (ii) on both sides and adding, we get:
( x 2 cos 2 θ + y 2 sin 2 θ − 2 xy sin θ cos θ ) + ( x 2 sin 2 θ + y 2 cos 2 θ +2 xy sin θ cos θ ) = A2 + B 2

24 Office.: 606 , 6th Floor, Hariom Tower, Circular Road, Ranchi-1, Ph.: 0651-2562523, 9835508812, 8507613968
( ) BY R. K. MALIK’S NEWTON CLASSES 25

⇒ x 2 (cos 2 θ + sin 2 θ ) + y 2 (sin 2 θ + cos 2 θ ) = A2 + B 2


⇒ x 2 + y 2 = A2 + B 2 .

2π 3π 4π 5π 6π 7π
177. Ans. (b) Let A = cos .cos .cos .cos .cos .cos .cos
15 15 15 15 15 15 15
π

2π 4π
⇒  2 sin  A =  2sin .cos  cos cos cos
15  15 15  15 15 15
 π   π π  3π

5π 6π 7π
 

cos cos cos


15 15 15
2π 2π  3π 4π
⇒  2 2 sin  A =  2sin cos  cos cos
15  15 15  15 15
 π  

5π 6π 7π
 

cos cos cos


15 15 15
4π 3π 5π 6π 7π
⇒  23 sin  A =  2 sin cos  cos cos cos cos
15  15 15 15 15 15 15
 π   4π 

1  8π  3π 6π 7π
  

⇒  23 sin  A =  sin  cos cos cos


15  2 15  15 15 15
 π 

7π 7π 3π 6π  8π 7π 7π
⇒  25 sin  A =  2 sin cos  cos cos ∵ sin = sin  π −   = sin
15  15 15 15 15  15 15 15
 π     

14π 2π 2π
    

⇒  25 sin  A = sin s cos cos ⇒ 25. A = cos cos


15  15 5 5 5 5
 π  π π

2π 2π 2π

⇒  26.sin  A =  2sin  cos ⇒  27 sin  A = 2 sin cos


5 5 5 5 5 5
 π  π  π

4π 1
  

⇒  27 sin  A = sin ⇒  27 sin  A = sin .


5 5 5 5 27
 π  π π
⇒ A=

178. Ans. (d), sin 2 α + sin 2 β + sin 2 γ − 2 cos α cos β cos γ


 

 1 − cos 2α   1 − cos 2 β   1 − cos 2γ 


 −2 cos α cos β cos γ
2 2 2
− + +

3 1
     

= − (cos 2α + cos 2 β + cos 2γ ) − 2 cos α cos β cos γ


2 2
3 1
= − [(2 cos 2 α − 1) + 2 cos( β + γ ) cos( β − γ )] −2 cos α cos β cos γ
2 2
3 1
= − [(2 cos 2 α − 1) + 2 cos(π − α ) cos( β − γ )] −2 cos α cos cos γ
2 2
3 1
= − [(2 cos 2 α − 1) − 2 cos α cos( β − γ )] −2 cos α cos β cos γ
2 2
3 1
= − [−1 + 2 cos α {− cos( β + γ ) − cos( β − γ )}] −2 cos α cos β cos γ
2 2
3 1
= − [−1 − 2 cos α {2 cos β cos γ }] − 2 cos α cos β cos γ
2 2
3 1
= + + 2 cos α cos β cos γ − 2 cos α cos β cos γ = 2 .
2 2

Office.: 606 , 6th Floor, Hariom Tower, Circular Road, Ranchi-1, Ph.: 0651-2562523, 9835508812, 8507613968 25
26 ( ) BY R. K. MALIK’S NEWTON CLASSES
Another Method:
We can directly use the formula: sin 2 α + sin 2 β + sin 2 γ = 2(1 + cos α cos β cos γ )
When α + β + γ = π or cos 2α + cos 2 β + cos 2γ = −1 − 4 cos α cos α cos β cos γ
when α + β + γ = π .
179. Ans. (b) Since sin θ and cosθ are the roots of the equation ax 2 − bx + c = 0, so we have:

sin θ + cos θ = …(i) and sin θ .cos θ = …(ii)


b c
a a

Squaring (i) on both sides, we get: sin 2 θ + cos 2 θ + 2sin θ cos θ =


b2
a2
2c b 2
⇒ 1+ [Using (ii)]
a a2
=

⇒ a 2 − b 2 + 2ac = 0.
180. Ans. (c) cos α + cos β + cos γ + cos(α + β + γ )

= 2 cos   cos   + 2 cos  + γ  cos 


 2  2  2  2 
α + β  α − β  α + β  α + β 

  

= 2 cos   cos   + cos 


 2   2  2
α + β  α − β  α + β 
+ γ 
 

= 2 cos   2 cos   cos    = 4 cos   cos   cos  .


 2   2  2   2  2  2 
α + β  α +γ   β + γ  α + β   β +γ   γ +α 

1 3
  

181. Ans. (c) cosec 10° − 3 sec10° =


sin10° cos10°

1 3
4  cos10° sin10°
cos10° − 3 sin10° 2 2  = 4[cos 60° cos10° − sin 60° sin10°]

sin10° cos10° 2sin10° cos10° sin 20°


= = 

4 cos 70° 4 sin 20°


= 4.
sin 20° sin 20°
= =

182. Ans. (b) cos 2 A + cos 2 B + cos 2 C


= cos 2 A + (1 − sin 2 B ) + cos 2 C = 1 + cos( A + B ) cos( A − B ) + cos 2 C
= 1 + cos C[cos( A − B ) + cos C ] = 1 + cos C[cos( A − B ) + cos( A + B )] = 1 + 2 cos A cos B cos C.
∴ cos 2 A + cos 2 B + cos 2 C − 2 cos A cos B cos C = 1 .
x tan 6° tan 42° sin 6° sin 42° sin 66° sin 78°
183. Ans. (a)
y cot 66° cot 78° cos 6° cos 42° cos 66° cos 78°
= =

(2sin 6° sin 66°).(2sin 42° sin 78°) (cos 60° − cos 72°).(cos 36° − cos120°)
(2 cos 6° cos 66°).(2 cos 42° cos 78°) (cos 72° + cos 60°).(cos120° + cos 36°)
= =

1 1 1 5 −1   5 + 1 1 
 − cos 72°  .  cos 36° +   .
2 2 2 4  4 2 (3 − 5).(3 + 5) 4

= = 1.
 − + 

1  1  5 −1 1   1 5 1  (1 + 5).(−1 + 5) 4
 cos 72° +  .  − + cos 36° 
   

+  . − +
= = =

2  2 4 2  2 4 
  +
   

x : y = 1:1 .

26 Office.: 606 , 6th Floor, Hariom Tower, Circular Road, Ranchi-1, Ph.: 0651-2562523, 9835508812, 8507613968
( BY R. K. MALIK’S NEWTON CLASSES
) 27
184. Ans. (c) ∵ a = b + c 2 . ∴ A = 90°
2 2

1 1
∴ ∆ = bc = (12 )( 5 ) = 30 .
2 2
B

1
Area of ∆ = × h × BC .
2
2∆ 2 × 30 60
Hence, h = .
13 13
h
= =

185. Ans. (d) m sin θ + n cos θ = p …(i)


BC A C

⇒ m sin θ + n cos θ + 2mn sin θ cos θ = p


2 2 2 2 2
⇒ m 2 (1 − cos 2 θ ) + n 2 (1 − sin 2 θ ) + 2mn sin θ cos θ = p 2
⇒ m 2 cos 2 θ + n 2 sin 2 θ − 2mn sin θ cos θ = m 2 + n 2 − p 2
⇒ (m cos θ − n sin θ ) 2 = m 2 + n 2 − p 2

⇒ m cos θ − n sin θ = ± m 2 + n 2 − p 2 …(ii)


Dividing (ii) by (i) we get:
m cos θ − n sin θ m2 + n 2 − p 2 m − n tan θ m2 + n2 − p 2
m sin θ + n cos θ n + m tan θ
=± ⇒ =±

186. Ans. (b), we know that cos a + cos 2a + .... + cos18a


p p

= cos a + cos(a + a ) + cos(a + 2a ) + .... + cos(a + 17 a )


∵ cos α + cos(α + β ) + cos(α + 2 β ) + .... + cos{α + (n − 1) β }
 17   18a 
cos a +  sin 
2  2   cos α + (n − 1) β  sin  nβ 
 
2  2 
 

sin
= 

2
 
sin  

a =
2
β  
 

 19π   18a   19π   π   19α   π 19π 


 

cos   sin   cos   sin   cos   sin  −  sin  −  − sin  


 36   2  =  36   2  =  2 =  2 36  =  36  =  36  = −1
 π  π 

sin   sin   sin   sin   sin   sin  


=

2  36   36   36   36   36 
 
a  π π
  π
  π
  π 

187. Ans. (c) ∠R = .


2 2 2
π π π
⇒ ∠P + ∠Q = π − ∠R = π − =

Now, since tan   and tan   are the roots of the equation
2 2
P Q

ax 2 + bx + c = 0, therefore tan   + tan   = …(i)


2 2 a
P  Q  −b

and tan   .tan   = …(ii)


2 2 a
P Q c

tan + tan
Now, tan  +  = 2 2 ⇒ tan = a
P Q b

2 2 4 1− c ∵ P + Q = 2 
π π
 1 − tan tan Q
P Q 

2 2
 P

⇒ 1= ⇒ b = c − a ⇒ a + b = c.
a
−b

188. Ans. (d) sin 2 α + sin 2 β − sin 2 γ


a−c

Office.: 606 , 6th Floor, Hariom Tower, Circular Road, Ranchi-1, Ph.: 0651-2562523, 9835508812, 8507613968 27
28 ( BY R. K. MALIK’S NEWTON CLASSES
)
= 1 − cos α + sin β − sin 2 γ = (1 − sin 2 γ ) + sin 2 β − cos 2 α
2 2

= cos 2 γ − cos(α + β ) cos(α − β ) = cos 2 γ + cos γ cos(α − β ) = cos γ {cos γ + cos(α − β )}


= cos γ {cos(α − β ) − cos(α + β )} = 2 sin α sin β cos γ
189. Ans. (c) tan A. tan B = 2
sin A.sin B 2 cos A cos B − sin A sin B −1 cos( A + B ) 1
cos A.cos B 1 cos A cos B + sin A sin B 3 cos( A − B ) 3
⇒ = ⇒ = ⇒ =−

1 1 3 1
⇒ cos( A + B ) = − .{cos( A − B )} = − .   = − .
3 3 5 5
1 − cos 2 B 1 1
190. Ans. (b) cos 2 A + sin 2 B = cos 2 A + = cos 2 A + − cos 2 B
2 2 2
 1 − tan 2 A  1 1  1 − tan 2 B  −2 tan 2 B 1 − tan 2 B 1
[∵ tan 2 A = 2 tan 2 B + 1]
 1 + tan A  2 2  1 + tan B  2 + 2 tan B 2 + 2 tan B 2
2 2 2 2
=  + −   = − +

−(1 + tan 2 B) 1 −1 1
+ = 0.
2(1 + tan 2 B) 2 2 2
= + =

191. Ans. (d) sin16° + cos16° = sin(15° + 1°) + cos(15° + 1°)


= sin15° cos1° + cos15° sin1° + cos15° cos1° − sin15° sin1°
= sin1°(cos15° − sin15°) + cos1°(cos15° + sin15°)
1 3 1
sin1° + cos1° = ( 3 cos1° + sin1°) .
2 2 2
=

tan x + tan y 1 (1 − a )
192. Ans. (c) x + y = ⇒ tan( x + y ) = tan   ⇒ ⇒ tan x + tan y =
6 6 1 − tan x tan y 3 3
π π 
=

1− a 
Now, tan x and tan y are the roots of the equation x2 −  x+a =0
 3 
⇒ 3 x 2 − (1 − a ) x + a 3 = 0 .
193. Ans. (c) sin α + sin β + sin γ − sin(α + β + γ )

= 2 sin   cos   − 2 cos  + γ  sin 


 2  2  2   2 
α + β  α − β  α + β  α + β 

 

= 2 sin   cos   − cos  + γ   = 2 sin   2 sin   sin 


 2   2  2  2  2   2
α + β  α − β  α + β  α + β  α − β   β +γ 

   

= 4 sin   sin   sin  .


 2   2   2 
α + β   β +γ   γ +α 

194. Ans. (a) α = sin 2 θ + cos 4 θ = sin 2 θ + cos 2 θ (1 − sin 2 θ )


1
= sin 2 θ + cos 2 θ − sin 2 θ cos 2 θ = 1 − (sin 2θ )2
4
3 3
Now when sin 2 2θ = 1 then θ = and when sin 2 2θ = 0 then θ = 1 ≤θ ≤1
4 4

cos θ cos α − cos β


195. Ans. (b)
1 1 − cos α cos β
=

1 − cos θ 1 − cos α cos β − cos α + cos β 1 − cos θ (1 − cos α ) (1 + cos β )


1 + cos θ 1 − cos α cos β + cos α − cos β 1 + cos θ (1 + cos α )(1 − cos β )
⇒ = ⇒ =

28 Office.: 606 , 6th Floor, Hariom Tower, Circular Road, Ranchi-1, Ph.: 0651-2562523, 9835508812, 8507613968
( ) BY R. K. MALIK’S NEWTON CLASSES 29

2sin 2 2sin 2 .2 cos 2


2 = 2 2 ⇒ tan 2  θ  = tan 2  α  cot 2  β  ⇒ tan  θ  = ± tan  α  cot  β  .
θ α β

2 cos 2 2 cos 2 .2 sin 2 2 2 2 2 2 2


2 2 2
θ α β            

 2π  4π  8π   14π 
196. Ans. (d) cos   cos   cos   cos 
 15  15  15   15 
 

 2π  4π  8π     2π  4π  8π 
 

= cos   cos   cos   cos  π −   = − cos   cos   cos   cos  


 15  15  15    15    15   15  15  15 
  π  π   

1  24 π   1
   

.sin  ∵ cos A.cos 2 A.cos 4 A....cos n −1 A = n .sin ( 2n A ) 


 15   2 sin A

24 sin  
=− 
 15 
π
   

1  16π 1 1  π  1
.sin  .sin  π +  = − . − sin    = .
 15 15   15   16
  π  
16sin   16 sin   16sin   
=− =−
 15   15   15 
π   π   π 

197. Ans. (c) α cos 2 3θ + β cos 4 θ = 16 cos 6 θ + 9 cos 2 θ


⇒ α (4 cos3 θ − 3cos θ )2 + β cos 4 θ = 16 cos 6 θ + 9 cos 2 θ
⇒ α (16 cos 6 θ + 9 cos 2 θ − 24 cos 4 θ ) + β cos 4 θ = 16 cos 6 θ + 9 cos 2 θ
⇒ 16α cos 6 θ + ( β − 24α ) cos 4 θ + 9α cos 2 θ = 16 cos 6 θ + 9 cos 2 θ
Comparing coefficients of cos 6 θ , cos 6 θ , and cos 2 θ on both sides, we get:
16α = 16, β − 24α = 0, 9α = 9
Solving these equations, we get: α = 1, β = 24.
1 1
cos 7 ° 2 cos 2 7 °
1 1 2 = 2
198. Ans. (d) tan 82 ° = cot 7 ° =
2 2 1 1 1
sin 7 ° 2sin 7 ° cos 7 °
2 2 2
1  3 1
1+
1 + cos15° 1 + cos(45° − 30°) 2  2 2  2 2 + ( 3 + 1) (2 2 + 3 + 1)( 3 + 1)
 + 

sin15° sin(45° − 30°) 1  3 1 ( 3 − 1) ( 3 − 1) ( 3 + 1)


= = = = =

2  2 2
 − 

2 2 3+2 2 +2 3+4
= 2 3+ 2 + 3+2 = 2 3+ 3+ 2 +2 = 3 2 +1 + 2 2 +1
2
= ( ) ( )
= ( 3 + 2) ( 2 + 1) .
3 +1 2 3 3 +1 2 2
199. Ans. (b) sin15° = ⇒ cosec15° = and cos15° = ⇒ sec15° =
2 2 3 −1 2 2 3 +1
2 2 2 2 (2 6 + 2 2) + (2 6 − 2 2) 4 6
∴ cosec15° + sec15° = =2 6.
3 −1 3 +1 (3 − 1) 2
+ = =

cos θ sin θ sin θ β


200. Ans. (a) = ⇒ tan θ =
cos θ α
β
= ⇒
α β α
 1 − tan 2 θ  2 tan θ 
Now, = α cos 2θ + β sin 2θ
sec 2θ cosec 2θ  1 + tan θ  1 + tan θ 
α β 
2 2
+ =α  +β  

Office.: 606 , 6th Floor, Hariom Tower, Circular Road, Ranchi-1, Ph.: 0651-2562523, 9835508812, 8507613968 29
30 (BY R. K. MALIK’S NEWTON CLASSES
)
 2β 
 1− 2 
2

 2αβ  α 3 − αβ 2 + 2αβ 2 α 3 + αβ 2
= α.
 β 
  α2 − β2 
 1+ β   1+ β 
α α
2  2 
=α  2 2 
+β 2 2 
α2 + β2
= 2
α +β2
=α  +β =
α + β  α + β 
 α2   α2 
   

21 −27
211. Ans. (b) sin α + sin β = − …(i) cos α + cos β = …(ii)
65 65
Squaring both side and adding (i) and (ii), we get:
(sin 2 α + sin 2 β + 2sin α sin β ) + (cos 2 α + cos 2 β + 2 cos α cos β )
441 729 1170
⇒ 2 + 2(sin α sin β + cos α cos β ) =
4225 4225 4225
= +

18  α − β  18 9 −3
⇒ 2{1 + cos(α − β )} = ⇒ 4 cos 2  ⇒ cos 
65  2  65  2  130 130
α − β 
= =− =

 Note: π < α − β < 3π (given)


.
 ⇒ π < α − β < 3π ⇒ cos  α − β  is − ve 


 2 2 2  2 
212. Ans. (b) Let P = (4 cos x + 3sin x).(3cos x − 4sin x)


Put 4 = r sin θ and 3 = r cos θ so that r = 42 + 32 r = 42 + 32 = 5


Then, P = (r sin θ cos x + r cos θ sin x). ( r cos θ cos x − r sin θ sin x )
25 25 25
= r 2 sin(θ + x).cos(θ + x) = sin ( 2 (θ + x ) ) As −1 ≤ sin 2(θ − x) ≤ 1 ⇒ − .
2 2 2
≤P≤

25
Hence, the maximum value of the given expression is .
2
3π 5π 7π 9π
213. Ans. (c) cos + cos + cos + cos + cos
11 11 11 11 11
π


cos .sin
11 11
π

sin
11
π

 ∵ cos α + cos(α + β ) + cos(α + 2 β ) + .... + cos{α + (n − 1) β } 

cos α + (n − 1)  sin 
 
2   2 
  β   nβ  

sin
 
=

2
 β 
 

 Here α = 11 , β = 11 , n = 5
 
 π 

 

10π 1 sin  π − π  1
 
1
sin sin
11 = 2  11  2 11 1
=2 = .
π
 

sin sin sin 2


=

11 11 11
π π π

 15π  7π   3π 
214. Ans. (a) sin   .sin   sin  
 32  16   8 

30 Office.: 606 , 6th Floor, Hariom Tower, Circular Road, Ranchi-1, Ph.: 0651-2562523, 9835508812, 8507613968
( BY R. K. MALIK’S NEWTON CLASSES
) 31

 π 15π   π 7π   π 3π 
= cos  −  .cos  −  .cos  −  = cos .cos .cos = cos .cos .cos
 2 32   2 16  2 8  32 16 8 32 16 8
π π π π π π

sin
4 = 1 sin(2n A) 
. ∵ cos A.cos 2 A.cos 4 A.....cos 2n −1 A = n
π

2 sin A 

8sin 8 2 sin
=

32 32
π π 

215. Ans. (b) cot( A + 15°) − tan( A − 15°)


cos( A + 15°) sin( A − 15°) cos( A + 15°) cos( A − 15°) − sin( A + 15°) sin( A − 15°)
sin( A + 15°) cos( A − 15°) sin( A + 15°) cos( A − 15°)
= − =

cos 2 A 2 cos 2 A
sin( A + 15°) cos( A − 15°) sin{( A + 15°) + ( A − 15°)} + sin{( A + 15°) − ( A − 15°)}
= =

2 cos 2 A 2 cos 2 A 4 cos 2 A


.
sin 2 A + sin 30° sin 2 A + 1 2 sin 2 A + 1
= = =

2
 π   5π   7π   2π  4π 
216. Ans. (b) sin   sin   sin   = cos   cos   cos 
 18   18   18  9  9  9 
π  

 8π   4π  

 ∵sin 18 = cos  2 − 18  = cos  18  = cos  9  


 π π π 

 5π  π 5π   4π   2π 
      

sin = cos  −  = cos   = cos   and 


 18  2 18   18   9 

7 2
 sin = cos  −  = cos   = cos

 18  2 18   18  9
 π  π π   π  π 

sin  23.  sin 8π


sin
 9= 9 = 9 =1.
 π π

π 8
23.sin 8sin 8sin
=

9 9 9
π π

217. Ans. (c) x 2 + 4 xy + y 2 = aX 2 + bY 2


⇒ ( X cos θ + Y sin θ ) 2 + 4( X cos θ + Y sin θ )
( X sin θ − Y cos θ ) + ( X sin θ − Y cos θ ) 2 = aX 2 + bY 2
⇒ X 2 (1 + 4sin θ cos θ ) + Y 2 (1 − 4 sin θ cos θ ) +4 XY (sin 2 θ − cos 2 θ ) = aX 2 + bY 2
Comparing coefficients of X 2 , Y 2 and XY on both sides, we get:
1 + 4sin θ cos θ = a …(i), 1 − 4sin θ cos θ = b …(ii), sin 2 θ − cos 2 θ = 0 …(iii)

Solving these equations, we get: θ = , a = 3, b = −1 .


4
π

5π 7π
218. Ans. (c) sin .sin .sin
18 18 18
π

π π  1  1
= sin .sin  −  .sin  +  = sin  3.  ∵sin θ .sin  3 − θ  .sin  3 + θ  = 4 sin 3θ 
18  3 18   3 18  4  18 
π π π   π   π  π 

1 π 1 1 1
     

= sin = . = .
4 6 4 2 8

Office.: 606 , 6th Floor, Hariom Tower, Circular Road, Ranchi-1, Ph.: 0651-2562523, 9835508812, 8507613968 31
32 ( )BY R. K. MALIK’S NEWTON CLASSES
1 − 4sin10° sin 70° 1 − 2(cos 60° − cos 80°)
219. Ans. (b) [∵ 2 sin A sin B = cos( A − B ) − cos( A + B )]
2sin10° 2 sin10°
=

1
1 − 2  − cos80° 
2  = 2 cos 80° = sin10° = 1 .

2sin10° 2sin10° sin10°


=

sin 2 3θ cos 2 3θ sin 2 3θ cos 2 θ − cos 2 3θ sin 2 θ


220. Ans. (b)
sin 2 θ cos 2 θ sin 2 θ cos 2 θ
− =

(sin 3θ cos θ − cos θ sin θ )


= ( sin 3θ cos θ + cos 3θ sin θ )
sin 2 θ cos 2 θ
sin(3θ + θ ) sin(3θ − θ ) sin 4θ sin 2θ 4 sin θ cos θ cos 2θ .2sin θ cos θ
= 8 cos 2θ .
sin 2 θ cos 2 θ sin 2 cos 2 θ sin 2 θ cos 2 θ
= = =

221. Ans. (b) sin 47° − sin 25° + sin 61° − sin11°
= (sin 47° + sin 61) − (sin 25° + sin11°) = (2sin 54° cos 7°) − (2sin18° cos 7°) = 2 cos 7°(sin 54° − sin18°)
 5 +1 5 −1  1
= 2 cos 7°.   = 2(cos 7°) × = cos 7° .
 4 4  2

222. Ans. (c) Let a be the first term and d be the common difference of the A.P.
10
Then, S10 = 11 ⇒ (2a + 9d ) = 11 ⇒ 10a + 45d = 11 …(i)
2
and t55 = 11 ⇒ a + 54d = 11 …(ii)
1 1
Solving (i) and (ii) we get: a = and d =
5 5
1 4
∴ t5 = tan A ⇒ a + 4d = tan A ⇒ + = tan A ⇒ tan A = 1 …(iii)
5 5
1 9
t10 = tan ⇒ a + 9d = tan B ⇒ + ⇒ tan B = 2 …(iv)
5 5
1 14
t15 = tan C ⇒ a + 14d = tan C ⇒ + = tan C ⇒ tan C = 3 …(v)
5 5
tan A + tan B + tan C − tan A tan B tan C
tan ( A + B + C ) =
1 − tan A tan B − tan B tan C − tan C tan A
⇒ tan ( A + B + C ) = 0 ⇒ A + B + C = π
5 1 4
223. Ans. (d), cos(α + β ) = ⇒ cos(α + β ) =
25 13 5
16 3
⇒ sin(α + β ) = 1 − = . ∵ sin(α + β ) is + ve since 0 < α + β < 2 
25 5
 π

1 1 5
And sin(α − β ) = ⇒ sin(α − β ) =
5 13 13
25 12
⇒ cos(α − β ) = 1 − cos (α − β ) + ve since − 4 < α − β < 4 
169 13
 π π
=

sin(α + β ) 3 / 5 3 sin(α − β ) 5 / 13 5
Now, tan(α + β ) = and tan(α − β ) = = .
(α + β ) 4/5 4 cos(α − β ) 12 / 13 12
= = =

∴ tan 2α = tan[(α + β ) + (α − β )]

32 Office.: 606 , 6th Floor, Hariom Tower, Circular Road, Ranchi-1, Ph.: 0651-2562523, 9835508812, 8507613968
( ) BY R. K. MALIK’S NEWTON CLASSES 33
3 5
tan(α + β ) + tan(α − β ) 56
= 4 12 = .
+
1 − tan(α + β ) tan(α − β )  3 5  33
1−  × 
=

 4 12 

− sin
2π 4π 1 7 =−1
224. Ans. (a) cos cos
cos .sin  23.  =
π

7 7 7  7  8sin π 8
π  π
23 sin
=

7 7
π

225. Ans. (c) sin 6θ = 2sin 3θ cos 3θ


= 2(3sin θ − 4 sin 3 θ ) (4 cos3 θ − 3cos θ ) = 2.{3sin θ − 4 sin θ (1 − cos 2 θ )}(4 cos3 − 3cos θ )
= 2.{4 cos 2 θ sin θ − sin θ }.(4 cos3 θ − 3cos θ ) = 32 cos5 θ sin θ − 32 cos3 θ sin θ + 6sin θ cos θ
= 32 cos5 θ sin θ − 32 cos3 θ sin θ + 3sin 2θ …(i)
Comparing the given equation with equation (i) we get x = sin 2θ .

226. Ans. (b) cos α + cos β = 0 ⇒ 2 cos   cos  =0 …(i)


 2  2
α + β  α − β 
 

sin α + sin β = 0 ⇒ sin   cos  =0 …(ii)


 2  2
α + β  α − β 
 

Solving (i) and (ii) we get: cos  =0.


 2 
α − β 

Now, cos 2α + cos 2 β = 2 cos(α + β ) cos(α − β )

= 2 cos(α + β ) 2 cos 2   − 1 = −2 cos(α + β ). ∵ cos  2  = 0 .


 2
 α − β    α − β  

sec 2 θ − tan θ 1 + tan 2 − tan θ


      

227. Ans. (c) Let x =


sec 2 θ + tan θ 1 + tan 2 θ + tan θ
=

⇒ ( x − 1) tan 2 θ + ( x + 1) tan θ + ( x − 1) = 0
Since tan θ is real we have: D ≥ 0 i.e., ( x + 1) 2 − 4( x − 1)2 ≥ 0
1 1 
i.e., − 3 x 2 + 10 x − 3 ≥ 0 i.e.,3 x 2 − 10 x − 3 ≤ 0 i.e., (3 x − 1) ( x − 3) ≤ 0 i.e., ≤ x ≤ 3 i.e., x ∈  ,3 .
3 3 
sec 2 θ − tan θ 1 
The value of always lies in the interval  , 3
sec θ + tan θ
2
3 
3 −1 
sin15°  2 2  3 − 1 ( 3 − 1) 2

228. Ans. (a) x = tan15° = = 2 − 3 ≈ 2 − 1.73 = 0.27 .


 

cos15°  3 +1  3 +1 2
= = =

2 2 
 

1 1 2 2

y = cosec 75° =
sin 75° cos15° 3 +1
= =

(2 2) ( 3 − 1)
= 2( 3 − 1) ≈ 1.41(1.73 − 1) = 1.41× 0.73 = 1.02
2
=

 5 −1 
z = 4sin18° = 4   = 5 − 1 = 2.23 − 1 = 1.23 ∴x< y< z.
 4 

Office.: 606 , 6th Floor, Hariom Tower, Circular Road, Ranchi-1, Ph.: 0651-2562523, 9835508812, 8507613968 33
34 ( BY R. K. MALIK’S NEWTON CLASSES
)
2π 4π
229. Ans. (b) x = y cos = z cos
3 3
1 1
⇒ x = − y = − z ⇒ y = z = −2 x ∴ xyz + zx = −2 x 2 + 4 x 2 − 2 x 2 = 0 .
2 2
2π 4π 8π
230. Ans. (a) cos cos cos cos
15 15 15 15
π

 16π 
sin  sin  π +  − sin 
1  π  1 15  1 15  1  15  1
 π   π 
.sin  2 4.  = .  = .  = . =− .
 

24 sin    15  16 sin  π  16 sin  π  16 sin 16


=

 15   15   15  15
π  π
   

sin A 1 cos A 1
231. Ans. (b) = m ⇒ sin B = sin A …(i), = n ⇒ cos B = cos A …(ii)
sin B cos B
To find tan A we eliminate tan B Hence
m n

Squaring (i) and (ii) on both sides and adding, we get:


1 1
sin 2 B + cos 2 B = sin 2 A + 2 cos 2 A

1 1 1 1
⇒ 1 = 2 sin 2 A + 2 cos 2 A ⇒ sec 2 A = 2 tan 2 A + 2 [Dividing throughout by cos 2 A ]
m n

1 1 1  2 1 m 2 (1 − n 2 )
m n m n

⇒ tan 2 A + 1 = tan 2
1 tan 1 tan 2
.
n 2 (m 2 − 1)

m2 n2 2 
n2
A + ⇒  − A = − ⇒ A =

m 1 − n2
 m 

⇒ tan A = ± . .
n m2 − 1
232. Ans. (b) , Because we want answer in terms of φ

1 − tan 2 
 2θ 

1 + tan  1 − tan 2 − x 1 + tan 2 


cos θ − x
− x
2 2

2
 θ  θ
Hence we proceed as
1 − x cos θ
1 − tan 2  1 + tan 2 − x  1 − tan 2 
=  = 
2θ  2θ 2θ 

1− x 
 

1 + tan 2 
2
θ

 1− x  2 φ   1− x  2 φ   1− x  2 φ

1−   tan − x 1 +   tan  (1 − x) − (1 + x).   tan


 1+ x  2  1+ x  2 1+ x  2
 1− x  2 φ   1− x  2 φ   1− x  2 φ
 tan  (1 − x) + (1 + x).  1 + x  tan 2
 
1+   tan − x 1 − 
= =

 1+ x  2   1+ x  2  

(1 − x). 1 − tan 2  1 − tan 2 φ


2 2 = cos φ .
 φ

(1 − x). 1 + tan  1 + tan


=  =
2 φ

2
2 φ 

2

1

233. Ans. (d) cos 2 76° + cos 2 16° − cos 76° cos16° = {(1 + cos152°) + (1 + cos 32°) − (cos 92° + cos 60°)}
2
1  1  1 3
 2 −  + cos152° + cos 32° − cos 92°  =  + (2 cos 92° cos 60°) − cos 92°
2  2  2 2

=

34 Office.: 606 , 6th Floor, Hariom Tower, Circular Road, Ranchi-1, Ph.: 0651-2562523, 9835508812, 8507613968
( BY R. K. MALIK’S NEWTON CLASSES
) 35
1 3  3
=  + cos 92° − cos 92°  = .
2 2  4
2π 3π
234. Ans. (d) sin + sin + sin
7 7 7
π

 π 3π 
3 π 
sin  7 7  .sin  × 
 +
 2  2 7

1 π 
sin  × 
=  

2 7
∵ we knw that sin α + sin(α + β ) + sin(α + 2 β ) + ......... + sin{α + (n − 1) β }
1st angle + last angle 
sin  .sin diff. of two cons. angles
 
2 2
 n  
  × 
1
 
sin  × diff. of two consecutive angles 
=    

2
  
 

 2π  3π   2π   3π   7π 
  

sin   .sin   2sin   sin   cos   − cos  cos  


 7  14  =  7   14  =  14   14  =  14  = 1 cot  π 
 π  π 

π  2  14 
sin   2sin   2sin   2sin  

 14   14   14   14 
 
π  π  π 

235. Ans. (b) cos12° + cos84° + cos132° + cos156°


= (cos12° + cos132°) + (cos 84° + cos156°) = 2 cos 72° cos 60° + 2 cos120° cos 36°
 5 + 1   5 −1  4 1
= (cos 72° − cos 36°) = (−2 sin 54° sin18°) = −2.   = −2. = − .
 4  4  16 2
 

236. Ans. (d) Since tan 30° and tan15° are the roots of the quadratic equation x 2 + px + q = 0, we have:
tan 30° + tan15° = − p …(i) [Sum of the roots of a Q.E.]
tan 30°.tan15° = q …(ii) [Product of the roots of a Q.E.]
2 + q − p = 2 + (tan 30° tan15°) + tan 30° + tan15° …. (iii)
tan 30° + tan15°
We know that 1 = ⇒ tan 30° + tan15° = 1 − tan 30° tan15°
1 − tan 30°.tan15°
⇒ (tan 30°.tan15°) + (tan 30° + tan15°) = 1 …(iv)
From (iii) and (iv) we get: 2 + q − p = 2 + 1 = 3
237. Ans. (a) We have: A + B = α …(i), A − B = x …(ii)
tan A k
and = i.e., tan A = k tan B
cos B 1
Now, from (iii) we get: tan A = k tan B
sin A cos B sin A cos B sin A cos B − cos A sin B k − 1
cos A sin B cos A sin B sin A cos B + cos A sin B k + 1
⇒ =k ⇒ =k ⇒ =

sin( A − B ) k − 1 sin x k − 1 k −1
⇒ sin x = sin α .
sin( A + B ) k + 1 sin α k + 1 k +1
⇒ = ⇒ =

sin 3 A sin A
tan 3 A tan 3 A − tan A cos 3 A cos A
238. Ans. (d) α = ⇒ α −1 =
sin A

tan A tan A
=

cos A

Office.: 606 , 6th Floor, Hariom Tower, Circular Road, Ranchi-1, Ph.: 0651-2562523, 9835508812, 8507613968 35
36 BY R. K. MALIK’S NEWTON CLASSES
( )
sin 3 A cos A − cos 3 A sin A sin 2 A 2 cos A cos 3 A 2
sin A cos 3 A sin A cos 3 A cos 3 A cos A α − 1
= = = ⇒ =

sin 3 A sin 3 A cos 3 A cos A tan 3 A cos 3 A 2 2α


Now, . . . = α. .
sin A cos 3 A cos A sin A tan A cos A α −1 α −1
= = =

7
239. Ans. (c), sin α + cos α =
2

2 tan 1 − tan 2
2 + 2 = 7
α α

1 + tan 2 tan 2 2

2 2
α α

⇒ 2  − tan 2 + 2 tan + 1 = 7  1 + tan 2  ⇒ 7 + 2 tan 2 − 4 tan 7 −2 =0


2 2  2 2 2
α α  α α α




( ) + ( )
7 + 2 x2 − 4x + 7 − 2 = 0 where x = tan
2
α
⇒ ( ) ( )
4 ± 16 − 12 4+2 3 1 1
or 7 − 2 or 7 −2
2 7 +2 2 7 +2 7 +2 7 +2 3
⇒ x= = = ⇒ x= ( ) ( )
( ) ( ) ( ) ( )
1
⇒ tan 7 −2
2 3
α
= ( )
We neglect tan 7 − 2 since 0 < α < ⇒ 0< ⇒ 0 < tan < tan
2 6 2 12 2 12
α π α π α π
= ( ) <

⇒ 0 < tan < 2 − 3 ⇒ 0 < tan < 0.27


2 2
α α

1 0.64
Now, 7 − 2 = 2.64 − 2 = 0.64 and 7 −2 = = 0.213
3 3
( ) ( )
240. Ans. (b), y − z = ( a cos 2 x + 2b sin x cos x + c sin 2 x ) − ( a sin 2 x − 2b sin x cos x + c cos 2 x )

= a ( cos 2 x − sin 2 x ) + 4b sin x cos x − c ( cos 2 x − sin 2 x ) = ( a − c ) cos 2 x + 2b sin 2 x

4b 2  4b
1
 1 − tan 2 x   2 tan x  2b 
  
2

 + 2b 
 + 2b  ( a − c )
∵ tan x = a − c 
 −   

 1 + tan x   1 + tan x  4b 2  4b 2
( a − c) 
1
 
2 2
= (a − c)   = (a − c)   
 ( a − c )2 2
 +   
  
 (a − c)
( a − c ) {( a − c ) − 4b 2 } 8b2 ( a − c ) ( a − c ) {( a − c ) + 4b 2 }
  
2 2

( a − c ) + 4b 2 ( a − c ) + 4b 2 ( a − c ) + 4b 2
2 2 2
= + = = (a − c)

∴ option (a), (c) and (d) are incorrect


Now, y + z = ( a cos 2 x + 2b sin x cos x + c sin 2 x ) + ( a sin 2 x − 2b sin x cos x + cos 2 x )

= ( a cos 2 x + sin 2 x ) + c ( sin 2 x + cos 2 x ) = a + c


3 + cot 76° cot16°
241. Ans. (b),
cot 76° + cot16°

36 Office.: 606 , 6th Floor, Hariom Tower, Circular Road, Ranchi-1, Ph.: 0651-2562523, 9835508812, 8507613968
( BY R. K. MALIK’S NEWTON CLASSES
) 37
cos 76° cos16° cos 76° cos16°
3+ 2 +1+
sin 76° sin16° = sin 76° sin16° = {2 sin 76° sin16°} + {sin 76° sin16° + cos 76° cos16°}
cos 76° cos16° cos 76° cos16°
⋅ ⋅
cos 76° sin16° + sin 76° cos16°
=

sin 76° sin16° sin 76° sin16°


+ +

{cos ( 76° − 16° ) − cos ( 76° + 16°)} + cos ( 76° − 16°)


sin ( 76° + 16° )
=

cos 60° − cos 92° + cos 60° 1 − cos 92° 2sin 2 46° sin 46°
= tan 46° = cot 44°
sin 92° sin 92° 2sin 46° cos 46° cos 46°
= = = =

242. Ans. (b), 5cos x + 12 cos y = 13 … (i) given


Let 5sin x + 12sin y = p ….. (ii)
Squaring (i) and (ii) on both sides and adding, we get
25 + 144 + 120 ( cos x cos y + sin x sin y ) = 169 + p 2

⇒ cos ( x − y ) = ⇒ −1 ≤ ≤1 ⇒ 0 ≤ ≤1 is always +ve 


p2 p2 p2  p2
120 120 120  120

∵

⇒ 0 ≤ p 2 ≤ 120 ⇒ 0 ≤ p ≤ 120

∴maximum value of p is 120 i.e., the maximum value of 5sin x + 12sin y is 120
243. Ans. (d), we have α sin 3 α + 3α cos 2 θ sin θ = x ….. (i) α cos3 θ + 3α cos θ sin 2 θ = x … (ii)
Adding (i) and (ii) we get; α {sin 3 θ + cos3 θ + 3cos θ sin θ ( sin θ + cos θ )} = x + y

⇒ α 2/ 3 ( sin θ + cos θ ) = ( x + y ) ….. (iii)


3 2/ 3

Subtracting (ii) from (i) we get: α {sin 3 θ − cos3 θ − 3sin θ cos θ ( sin θ − cos θ )} = x − y

⇒ α ( sin θ − cos θ ) = ( x − y ) ⇒ α 2/ 3 ( sin θ − cos θ ) = ( x − y ) … (iv)


3 2 2/3

Adding (iii) and (iv) we get : α 2/ 3 ( sin θ + cos θ ) + ( sin θ − cos θ )


2 2 2/ 3 2/ 3
{ } = ( x + y) + ( x − y)

= 2α 2 / 3
2/3 2/ 3
⇒ ( x + y) + ( x − y)
244. Ans. (b), sin 2 α + sin 2 β − sin 2 γ
= sin 2 α + sin ( β + γ ) sin ( β − γ ) ∵ sin 2 A − sin 2 B = sin ( A + B ) sin ( A − B ) 

= sin 2 α + sin ( β + γ ) sin (π − α ) [∵ α + β − γ = π ⇒ β − γ = π − α ]


= sin 2 α + sin ( β + γ ) sin α = sin α sin α + sin ( β + γ ) 

= sin α ⋅ sin {π − ( β − γ )} + sin ( β + γ )  = sin α [ 2sin β cos γ ] = 2 sin α sin β cos γ

245. Ans. (b), Since tan A and tan B are the roots of x 2 + mx + n = 0, so we have
tan A + tan B = − m …. (i) , tan A tan B = n …. (ii)
tan A + tan B
∴ tan ( A + B ) = [using (i) and (ii)]
1 − tan A tan B 1 − n
−m
=

∴ statement (a) is correct


n −1
m
=

m 2 + ( n − 1)
2

Now, sec ( A + B ) = 1 + tan ( A + B ) = 1 +


2 2 m2
( n − 1) ( n − 1)
2 2
=

Office.: 606 , 6th Floor, Hariom Tower, Circular Road, Ranchi-1, Ph.: 0651-2562523, 9835508812, 8507613968 37
38 BY R. K. MALIK’S NEWTON CLASSES
( )
( n − 1) n −1
⇒ cos ( A + B ) = ± ∴statement (b) is incorrect
m + ( n − 1)
2 2 2
2 1

2 m + n − n +

Now, sin 2 ( A + B ) + m sin ( A + B ) cos ( A + B ) + n cos 2 ( A + B )

( n − 1)  m2 + m m + n 
2

= cos 2
( A + B )  tan 2
tan
m2 + ( n − 1)  ( n − 1)
2 2
( n − 1) 
( A + B ) + m ( A + B ) 
+ n  =  

( n − 1)  m2 + m2 ( n − 1) + n ( n − 1)  m2 n + n ( n − 1) 2
2 2

Statement (c) is correct


m + ( n − 1)  ( n − 1) m 2 + ( n − 1)
2 2 2
= 2   = =n

246. Ans. (c), sin A = 3sin ( A + 2 B )
sin ( A + 2 B ) 1 sin ( A + 2 B ) + sin A 4
[By componendo-dividendo]
sin A 3 sin ( A + 2 B ) − sin A −2
⇒ = ⇒ =

2sin ( A + B ) cos B tan ( A + B )


= −2 ⇒ = −2 ⇒ tan ( A + B ) = −2 tan B
2 cos ( A + B ) sin B tan B

247. Ans. (c), m sin θ + n cos θ = p


⇒ n cos θ = ( p − m sin θ ) ⇒ n 2 cos 2 θ = p 2 + m2 sin 2 θ − 2mp sin θ
⇒ n 2 cos 2 θ = p 2 + m2 sin 2 θ − 2mp sin θ ⇒ ( m 2 − n 2 ) sin 2 θ − 2mp sin θ + ( p 2 − n 2 ) = 0 …. (i)
Since α and β are the values of θ which satisfy the given equation, so sin α and sin β are the roots of
equation ( m 2 − n 2 ) x 2 − 2mpx + ( p 2 − n 2 ) = 0
Now, since sin α and sin β are the roots of the above equation we have:
2mp
sin α + sin β = (sum of the roots)
m2 + n2
248. Ans. (b), Given sin A + cos B = a …. (i) sin B + cos A = b …. (ii)
Squaring (i) and (ii) on both sides and adding, we get
( sin A + cos B + 2sin A cos B ) + ( sin B + cos A + 2sin B cos A) = a + b
2 2 2 2 2 2

⇒ ( sin A + cos A ) + ( cos B + sin B ) + 2 ( sin A cos B + cos A sin B ) = a + b


2 2 2 2 2 2

a2 + b2 − 2
⇒ 2 + 2sin ( A + B ) = a 2 + b 2 ⇒ sin ( A + B ) =
2
249. Ans. (c), m sin θ + n cos θ = p
⇒ m sin θ = p − n cos θ ⇒ m 2 sin 2 θ = p 2 + n 2 cos 2 θ − 2 pm cos θ
⇒ m 2 (1 − cos 2 θ ) = p 2 + n 2 cos 2 θ − 2 pn cos θ

⇒ ( m 2 + n 2 ) cos 2 θ − 2 pn cos θ + ( p 2 − m 2 ) = 0 … (i)


Since α and β are the values of θ which satisfy the given equation, so cos θ and cos β are the roots
of equation ( m 2 + n 2 ) x 2 − 2 pnx + ( p 2 − m2 ) = 0
Now, since cos α and cos β are the roots of above equation (i), so we have

(product of the roots) = cos α cos β =


p 2 − m2
m2 + n2

38 Office.: 606 , 6th Floor, Hariom Tower, Circular Road, Ranchi-1, Ph.: 0651-2562523, 9835508812, 8507613968
( BY R.
) K. MALIK’S NEWTON CLASSES 39
2π 4π 
250. Ans. (a), x cos θ = y cos  θ +  = z cos  θ +  = k (say)
3 3 
  

2π 4π
  

Then cos θ = , cos  θ +  = and cos  θ +


3 3
k   k   k
=

2π 4π 
x   y   z

+ + = cos θ + cos  θ +  + cos  θ +


3 3 
k k k   
∴ 

1 1 1 1 1 1
x y z   

⇒ k  + +  = cos θ + 2 cos (θ + π ) cos  = cos θ − cos θ = 0 + + =0


3
 π

251. Ans. (c), cos θ = cos α cos β


x y z  x y z

cos θ cos β cos α 1 cos α − cos θ 1 − cos β


[By componendo-dividendo]
cos α 1 cos θ cos β cos α + cos θ 1 + cos β
⇒ = ⇒ = ⇒ =

2sin   sin   2sin sin   sin 


 2   2  =  2   2  = tan 2 β
2 β

2 ⇒
 α +θ   θ −α  θ +α   θ −α 

 α − θ  2 cos 2 β 2
2 cos   cos  cos  cos 

 2  2  2  2   2 
 α +θ  θ +α   θ −α 
  

⇒ tan   tan   = tan


2 β

 2   2  2
θ +α   θ −α 

252. Ans. (c), tan θ + tan  + θ  − tan  − θ 


3 3
π  π 
 

sin  + θ  sin  − θ  sin  + θ  cos  − θ  − cos  + θ  sin  − θ 


sin θ 3 3  = sin θ + 3 3 3  3
π  π  π  π  π  π 

cos θ  cos θ
cos  + θ  cos  − θ  cos  + θ  cos  − θ 
= + −   

3 3 3 3
π  π π  π 

sin θ 2sin θ cos θ sin θ 8sin θ cos θ sin θ 1 − 4sin θ + 8sin θ cos θ
   
sin θ sin 2θ
2 2
( )
cos θ cos 2 π − sin 2 θ cos θ 1 cos θ 1 − 4sin 2 θ cos θ 1 − 4sin 2 θ
− sin 2 θ
= + = + = + =

3 4
( )
sin θ − 4sin 3 θ + 8sin θ 1 − sin 2 θ ) = 9sin θ − 12 sin θ 3 ( 3sin θ − 4sin 3 θ ) 3sin 3θ
= 3 tan 3θ
3
(
cos θ 1 − 4 1 − cos 2 θ 4 cos θ − 3cos θ
3
( 4 cos θ − 3cosθ )
3
cos 3θ
= = =
{ ( )}
253. Ans. (d), Given ( sin θ + cos θ ) = h …… (i)
squaring (i) on both sides, we get sin 2 θ + cos 2 θ + 2sin θ cos θ = h 2
1
⇒ 1 + 2sin θ cos θ = h 2 ⇒ sin θ cos θ = h 2 − 1 …. (ii)
2
( )
Now, sin θ and cos θ are the roots of a quadratic equation therefore,
sum of the roots = sin θ + cos θ = h [using (i)]
1
Product of the roots = sin θ cos θ = h 2 − 1 [using (ii)]
2
( )
1
∴Required quadratic equation is x 2 − hx + h 2 − 1 = 0 i.e. 2 x 2 − 2hx + ( h 2 − 1) = 0
2
( )
α cos u − β
254. Ans. (c), cos θ =
α + β cos u

Office.: 606 , 6th Floor, Hariom Tower, Circular Road, Ranchi-1, Ph.: 0651-2562523, 9835508812, 8507613968 39
40 ( BY R. K. MALIK’S NEWTON CLASSES
)

 1 − tan 2 
 2 u 

 1 + tan  θ α 1 − tan 2 u  + β 1 + tan 2 u 


1 − tan 2 1 − tan 2
α +β
2 2
2 u

2 2 2
θ
   

1 + tan  1 − tan 2  1 + tan α 1 + tan 2  + β 1 − tan 2 


⇒ =   ⇒ =  
2 2θ

2 2
2 u 

2 2
θ   u  u

 1 + tan 
α +β  
2 u

2

1 − tan 2 (α + β ) − (α − β ) tan 2
2 tan 2 2 (α − β ) tan 2
2 = 2 ⇒ 2 = 2 [By componendo -dividndo]
θ u θ u

1 + tan 2 2
(α + β ) + (α − β ) tan

2θ 2 u

2 2
( α + β )

⇒ tan 2 tan 2 ⇒ tan = ± tan


2 2 2 2
θ α −β u θ α −β u
=

255. Ans. (a), S = sin θ + sin 2θ + ...... + sin nθ


α +β α +β

sin θ + ( n − 1)  sin
2 2
 θ  nθ

sin
= 

2
θ

sin α + ( n − 1)  sin 
2  2
  β   nβ 
∵ we know that formula sin α + sin (α + β ) + sin (α + 2β ) + .... + sin {α + ( n − 1) β } =
 

sin  
 

2
  β  

( n + 1)θ sin nθ
 

sin
2 2
sin
=

2
θ

256. Ans. (d), Given  x +  = 2 cos θ


 1

1 1 1 1
 x
3

⇒  x +  = 8 cos3 θ ⇒ x3 + 3 + 3  x +  = 8cos3 θ ⇒ x3 + 3 + 3 ( 2 cos θ ) = 8cos3 θ


 

1 1
 x x  x x

⇒ x3 + 3 = 8cos3 θ − 6 cos θ ⇒ x3 + 3 = 2 4 cos3 θ − 3cos θ = 2 cos 3θ


( )
257. Ans. (c), since α and β satisfy the given equation, we have
x x

a cos α + b sin α = 6 …. (i) a cos β + b sin β = c …(ii)


Subtracting (ii) from (i) we get a ( cos α − cos β ) + b ( sin α − sin β ) = 0

⇒ − 2a sin   sin   + 2b cos   sin  =0


 2   2  2   2
α + β  α −β  α + β  α − β 
 

⇒ − a sin   + b cos  =0 ∵ sin  2  ≠ 0 since α ≠ β 


 2  2
α + β  α + β   α − β  
     

⇒ b cos   = a sin   ⇒ tan 


 2  2   2
α + β  α + β  α + β  b
=
  a

40 Office.: 606 , 6th Floor, Hariom Tower, Circular Road, Ranchi-1, Ph.: 0651-2562523, 9835508812, 8507613968
( BY R. K. MALIK’S NEWTON CLASSES
) 41
258. Ans. (a), Sicne tan A and tan B are the roots of x 2 − ax + b = 0, so we have
tan A + tan B = a …. (i) and tan A tan B = b …. (ii)
tan A + tan B a 2 + (1 − b )
2

∴ tan ( A + B ) = ⇒ sec ( A + B ) = tan ( A + B ) + 1 = +1 =


2 2 a2
1 − tan A tan B 1 − b (1 − b ) (1 − b )
a
2 2
=

(1 − b ) (1 − b )
2 2

⇒ cos ⇒ sin ( A + B ) = 1 − cos ( A + B) = 1− 2


2 2 2 a2
a + (1 − b ) a + (1 − b ) a 2 + (1 − b )
( A + B) = 2 2 2 2
=

259. Ans. (d), sin A − cos B = cos C

⇒ sin A = cos B + cos C ⇒ 2 sin cos = 2 cos   cos 


2 2  2   2 
A A  B+C   B−C 

⇒ sin  −    cos = cos   cos 


 2  2  2  2   2   2 2  2 
 π  B + C  A  B+C   B −C   A π  B + C 
 ∵ A + B + C = π ⇒ = − 

⇒ cos   cos = cos   cos   ⇒ cos = cos 


 2  2  2   2  2  2 
 B+C  A  B+C   B −C  A  B −C 

…. (i) …. (ii)
2 2
A B −C
⇒ = ⇒ A = B−C

Also, A + B + C = π Solving (i) and (ii) we get: B =


2
π

260. Ans. (c), sin (120° − θ ) + sin (120° − θ )


2 2

= sin 2 {180° − (120° + θ )} + sin 2 {180° − (120° − θ )} = sin 2 ( 60° − θ ) + sin 2 ( 60° + θ )
1 1 1
1 − cos (120° − 2θ )} + {1 − cos (120° + 2θ )} = 1 − {cos (120° − 2θ ) + cos (120° + 2θ )}
2 2 2
= {
1 1
= 1 − {2 cos120° cos 2θ } = 1 − cos (180° − 60° ) cos 2θ = 1 − cos 2θ
2 2
1
Clearly, max value of 1 − cos 2θ is obtained when cos 2θ = −1
2
1 1 3
i.e. max value of 1 − cos 2θ is 1 + =
2 2 2
1 1 1 1
min. value of 1 − cos 2θ is obtained when cos 2θ = 1 i.e. min value of 1 − cos 2θ is 1 − =
2 2 2 2
1 3
∴ min = and max =
2 2
1 3
261. Ans. (b), sin 2θ + sin 2φ = …. (i) cos 2θ + cos 2φ = …. (ii)
2 2
Squaring and adding (i) and (ii) we get:
1 9
sin 2 2θ + sin 2 2θ + 2sin 2θ sin 2φ + cos 2 2θ + cos 2 2φ + 2 cos 2θ cos 2φ = +
4 4
( ) ( )
5
⇒ sin 2 2θ + cos 2 2θ + sin 2 2φ + cos 2 2φ + 2 ( sin 2θ sin 2φ + cos 2θ cos 2φ ) =
2
( ) ( )
5 5 5
⇒ 2 + 2 cos ( 2θ − 2φ ) = ⇒ 1 + cos 2 (θ − φ ) = ⇒ cos 2 (θ − φ ) =
2 4 8

Office.: 606 , 6th Floor, Hariom Tower, Circular Road, Ranchi-1, Ph.: 0651-2562523, 9835508812, 8507613968 41
42 BY R. K. MALIK’S NEWTON CLASSES
( )
sin A cos A
262. Ans. (d), = p ⇒ sin A = p sin B …… (i) = q ⇒ cos A = q cos B …. (ii)
sin B cos B
sin A p sin B
Dividing (i) by (ii) we get: ⇒ tan A = tan B … (iii)
cos A q cos B
p
=

Squaring (i) and (ii) on both sides and adding we get sin 2 A + cos 2 A = p 2 sin 2 B + q 2 cos 2 B
q

⇒ 1 = p 2 sin 2 B + p 2 cos 2 B + q 2 − p 2 cos 2 B


( )
⇒ 1 − p 2 = q 2 − p 2 cos 2 B ⇒ sec 2 B = 1 tan
q2 − p2 2 q2 − p2
1 − p2 1 − p2
( ) ⇒ + B =

q2 −1 q2 −1
⇒ tan 2 B = 1 tan …. (iv)
q2 − p2
1 − p2 1 − p2 1 − p2
− = ⇒ B = ±

p q2 −1 − p q2 −1
From (iii) and (iv) we get tan A = or tan
q 1 − p2 q 1− p
A =

Hence, all the alternatives are correct


cos x 3
263. Ans. (d), cos x = 3cos y ⇒
cos y 1
=

cos x + cos y 4
[By componendo-Divideno]
cos x − cos y 2
⇒ =

2 cos   cos  cos   cos 


 2   2  =2 ⇒  2   2  = 2 ⇒ cot  x + y  = 2 tan  y − x 
 x+ y  x− y  x+ y  y−x
 

2sin  sin sin sin  2   2 


 2   2   2   2 
   
 x+ y  y−x  x+ y  y−x
      

264. Ans. (c), sin 2 5° + sin 2 10° + sin 2 15° + ..... + sin 2 90°
= sin 2 5° + sin 2 85° + sin 2 10° + sin 2 80° + sin 2 15° + sin 2 75° + ..... +
( ) ( ) ( )
( sin 40° + sin 50°) + sin 45° + sin 90°
2 2 2 2

= ( sin 5° + cos 5° ) + ( sin 10° + cos 10° ) + ( sin 15° + cos 15° )
2 2 2 2 2 2

+...... + ( sin 40° + cos 40° ) + sin 45° + sin 90°


2 2 2 2

 1  1
2

= 1 + 1 + 1........ + 1 +   + 1 = 8 + 2 + 1 = 9.5
2

8 terms  2
265. Ans. (a), f (θ ) = 3sin 2 θ − 6sin θ cos θ + cos 2 θ + 2
 1 − cos 2θ  1 + cos 2θ
= 3  − 3sin 2θ +   + 2 = 4 − cos 2θ − 3sin 2θ ∴ 4 − 10 ≤ f (θ ) ≤ 4 + 10
2 2
 

⇒ Range of g (θ ) = {1, 2,3, 4,5,6, 7} ∵ g (θ ) =  f (θ ) 


   

1
266. Ans. (b), sin t + cos t =
5

2 tan 1 − tan 2
2 2 = 1 ⇒ 5  2 tan t + 1 − tan 2 t  = 1 + tan 2 t ⇒ 6 tan 2 t − 10 tan t − 4 = 0
t t

t 5 2 2 2 2 2
1 + tan 2 1 + tan 2
⇒ +

2 2
t  

42 Office.: 606 , 6 Floor, Hariom Tower, Circular Road, Ranchi-1, Ph.: 0651-2562523, 9835508812, 8507613968
th
( ) BY R. K. MALIK’S NEWTON CLASSES 43

10 ± 100 + 96 t 10 ± 14
⇒ 6 x 2 − 10 x − 4 = 0 where x = tan ⇒ tan =
2 12 2 12
t
⇒ x=

⇒ tan =2 ∵ 0 < t < π ⇒ 0 < 2 < 2 ⇒ tan 2 is +ve 


2
t  t π t 

267. Ans. (c) {cos (α + β + γ ) + cos (α − β − γ )} + {cos ( β − γ − α ) + cos ( γ − α − β )}

= {2 cos α cos ( β + γ )} + {2 cos ( −α ) cos ( β − γ )} = 2 cos α cos ( β + γ ) + 2 cos α cos ( β − γ )

= 2 cos α {cos ( β + γ ) + cos ( β − γ )} = 2 cos α {2 cos β cos γ } = 4 cos α cos β cos γ


3π 5π 7π
268. Ans. (c), cos 2 + cos 2 + cos 2 + cos 2
16 16 16 16
π

 π 3π  2 5π 7π  7π 2  5π  2  5π  2  7π 
= sin 2  −  + sin 2  −  + cos + cos 2 = sin 2   + sin   + cos   + cos 
 2 16   2 16  16 16  16  16   16   16 
π π  

7π 7π   2 5π 5π 

=  sin 2 + cos 2  +  sin + cos 2  = 1+1 = 2


16 16   16 16 

sin ( x + y )

269. Ans. (a),


sin ( x − y ) ( a − b )
(a − b)
=

sin ( x + y ) + sin ( x − y ) ( a + b ) + ( a − b )
[By componendo-dividendo]
sin ( x + y ) − sin ( x − y ) ( a + b ) − ( a − b )
⇒ =

2sin x cos y 2a tan x a


2 cos x sin y 2b tan y b
⇒ = ⇒ =

4 − p 2 − q 2 4 −  p + q 
2 2

270. Ans. (c),


p2 + q2 p2 + q2
=

4 −  cos α + cos β 2 + 2 cos α cos β + sin 2 α + sin 2 β + 2sin α sin β 


( ) ( )
cos α + cos β + 2 cos α cos β + sin α + sin β + 2 sin α sin β
 
2 2 2 2
=
( ) ( )
2 − 2 {cos α cos β + sin α sin β }
∵ cos 2 A + sin 2 A = 1
2 + 2 {cos α cos β + sin α sin β }
=

2sin 2 
1 − cos (α − β ) 2 
α −β 
= tan 2 

1 + cos (α − β )  2 
α − β 
2 cos 2 
= = 

 2 

α −β 


271. Ans. (c)Using cos = cos  +  = − sin
8 2 8 8
π π  π

3π 7π
⇒ cos = cos  −  = sin ⇒ cos = cos  π −  =  − cos 
8 2 8 8 8 8  8
π π  π  π  π

⇒ 1 + cos 1 − cos 1 + sin 1 − sin  ⇒ 1 − cos 2  1 − sin 2  = sin 2 cos 2
8  8  8  8 8  8 8 8
 π  π  π  π  π  π π π
 

1 π  1 1 1
2

 sin  Hence, (c) is the correct answer.


4 4 4 2 8
⇒ ⇒ ⋅ =

Office.: 606 , 6th Floor, Hariom Tower, Circular Road, Ranchi-1, Ph.: 0651-2562523, 9835508812, 8507613968 43
44 ( ) BY R. K. MALIK’S NEWTON CLASSES
272. Note that x, y ∈  , π  then x < y ⇒ sin x > sin y y -axis
2 
π 

∵ sin θ is decreasing function in  , π 


2 
π 

sin 2
Hence, the given statement is true sin 3
x -axis
2 2 3
273. Ans. (d)
tan 81° − tan 63° − tan 27° + tan 9° = tan 9° + cot 9° − tan 27° − cot 27°
1 1 2 2 2 2
sin 9° cos 9° sin 27° cos 27° sin18° sin 54° sin18° cos 36°
= − = − = −

2× 4 2 × 4 8  5 + 1 − 5 + 1
=4
5 −1 5 +1 5 −1
= − =

274. Ans. (b) sin θ1 − sin θ 2 = a, cos θ1 + cos θ 2 = b ⇒ a 2 + b 2 = 2 + 2 cos (θ1 + θ 2 ) ⇒ 0 ≤ a 2 + b 2 ≤ 4.

26 13
275. Ans. (a), ∵ tan + tan …… (i)
2 2 8 4
α β
= =

15
tan ⋅ tan ∴ tan   = tan  + 
2 2 8  2 2 2
α β α + β  α β 
=

tan + tan 1 − tan 2 


2 2 = (13 / 4 ) = − 26 ; ∴ cos (α + β ) =  2  = 1 − ( −26 / 7 )
α + β
2
α β 

1 − tan ⋅ tan 1 − (15 / 8 ) 7  1 + ( −26 / 7 )2


1 + tan 2 
=

2 2  2
α β α + β

49 − 676 627

49 + 676 725
= =−

276. Ans. (b)Given a cos θ + b sin θ = c …(1)


Let a sin θ − b cos θ = k …(2)
To find k , squaring and adding (1) and (2) we get
a 2 cos 2 θ + b 2 sin 2 θ + 2ab sin θ cos θ + a 2 sin 2 θ + b 2 cos 2 θ − 2ab sin θ cos θ = c 2 + k 2
a 2 ( cos 2 θ + sin 2 θ ) + b 2 ( sin 2 θ + cos 2 θ ) = c 2 + k 2

( a sin θ − b cos θ ) = a 2 + b 2 − c 2 , (Using (2))


2
⇒ a 2 + b2 = c2 + k 2 ⇒ k 2 = a 2 + b2 − c 2 ∴

277. Ans. (b) We have 11 km = 11× 1000 m =11000 m and 1 hr = 60 minutes


= 60 × 60 seconds = 3600 seconds
The man travels 11 km in 1 hour. i.e. in 3600 seconds, he travels 11000 m.
11
36 m
∴ in 36 seconds, he travels × 11000 = 110 m =S
B

3600
56°
  14 
Also, θ = 56° =  ⋅ 56  =  π 
 180   45 
c c
 π S O r A
∵ S = rθ ∴ r =
θ

44 Office.: 606 , 6 Floor, Hariom Tower, Circular Road, Ranchi-1, Ph.: 0651-2562523, 9835508812, 8507613968
th
( BY R. K. MALIK’S NEWTON CLASSES
) 45
110 45 7
∴ from the figure diameter = 2r = 2 = 2 × = 220 × × = 225 m
14π / 45 14 22
S

278. Ans. (c), From the figure


θ

The arc of the circle intercepted within the triangle is EXF


Now ∵ OB = OE
A

∴ m∠OEB = m∠OBE = 60° ∴ m∠BOE is also = 60°


Similarly, m∠COF is also = 60°
X

∴ m∠EOF is also = 60°


E F

∴ for the arc EXF , r = 9 cm and θ = 60° =  × 60  =   9 9


180 3
c c
 π  π  B C
O

∴ l ( EXF ) = rθ = 9 × (π / 3) = 3π cm
   

279. Ans. (a), We have θ1 = 30° =  × 30  =  


 180  6
c c
 π  π 

Let AXB be the arc of the first circle which is double the arc CYD of the second circle, where r2 = 3r1
∵ l ( arc AXB ) = 2 × l ( arc CYD ) ∴ r1θ1 = 2 ⋅ r2θ 2

= 2 ⋅ 3r1 ⋅ θ 2
B

6
π /6 Y
π

r2 C
D
∴ r1 ⋅
X

 π   180 π 
O1 r1 O2 θ

×  = 5°
A

 36   π 36 
c o

∴ θ2 =   = 

280. Ans. (a), We have, θ1 = 45° =  × 45  =  


 180  4
c c
 π  π 
E

θ 2 = 60° =  × 60  =  
B

π /3 Y
Z

 180  3
π /4
c c
 π  π 
C
X D

O2 r
F θ

∵ l ( arc AXB ) = l ( arc CYD ) ∴ r1θ1 = r2θ 2


O1 r1
2
A

r1 4
……(I)
4 3 r2 3
π π
∴ r1 ⋅ = r2 ⋅ ∴ =

Now, let EZF be an arc of the first circle whose length is equal to the radius of the second circle
If m∠EO1 F = θ c , then : l ( arc EZF ) = r2 ∴ r1 θ = r2

1 1 3
..from (i)
r1 ( r1 / r2 ) ( 4 / 3)
θ= 2=
4
c
r
= ∴ θ = 

281. Ans. (b), From the figure m∠AOB = 60° ∴ m∠AOM = 30°
∴ ∆OAM is a 30° − 60° − 90° triangle ∴ AM = half of hypotenuse = 6
∴ AB = 2 ( AM ) = 2 ( 6 ) = 12 ∴ OA = OB = AB = 12
∴ ∆OAB is an equilateral triangle with each side a = OA = 12 12 12
O

a2 3 3 6 M 6
= (12 ) ⋅ = 36 3 …..(i)
2

4 4
A
∴ A ( ∆AOB ) = B

Also, for the sector OAB r = OA = 12 and θ = m∠AOB = 60°


1 1
= ⋅ 60  =   ∴ A ( sector OAB ) = r 2θ = × (12 ) × = 24π …(ii)
2

 180  3 2 2 3
c c
 π  π  π

Office.: 606 , 6th Floor, Hariom Tower, Circular Road, Ranchi-1, Ph.: 0651-2562523, 9835508812, 8507613968 45
46 ( BY R. K. MALIK’S NEWTON CLASSES
)
∴ from (i) and (ii) area between arc AB and chord AB
= A ( sector OAB ) − A ( triangle OAB ) = 24π − 36 3 = 12 2π − 3 3 sq. cm
( )
282. Ans. (c), Let OE ⊥ AB and OF ⊥ AC
Then. OE = 3 cm and OF = 3 2 cm Also, OA = r = 6 cm
OE 3 1
If, in ∆ΟΕΑ, m∠EAO = θ1 , then sin θ1 = = = ∴ θ1 = 30°
OA 6 2
A

OF 3 2 1
If, in ∆OAF , m∠FAO = θ 2 , then : sin θ 2 = ∴ θ 2 = 45°
θ 2θ1
6 2 3 2O 3
= = F E

∴ m∠EAF = θ1 + θ 2 = 30° + 45° = 75° 6 6


OA

  5π 
∴ m∠BOC = 2 ( 75° ) = 150° =  × 150  = 
C B

 180   6 
c c
 π


D

∴ l ( arc BDC ) = rθ = 6 × = 5π cm ∴ Perimeter of sector O − BDC is


6
= OB + OC + l ( arc BDC ) = 6 + 6 + 5π = (12 + 5π ) cm
1 2 1 5π
Also, area of sector O − BDC is = r θ = × 62 × = 15π cm 2
2 2 6
∴ Perimeter = (12 + 5π ) cm and area = 15π cm 2
283. Ans. (a) Let O1 and O2 be the centres of the given two circles
Also let A and B be their points of intersection
Then, O1 A = O1 B = O2 A = O2 B = 5 cm and O1O2 = 5 2 cm

But ( O1 A ) + ( O2 A ) = 25 + 25 = 50 = 5 2
2 2 2 2
( ) = ( O1O2 )

∴ by converse of pythagoras Theorem m∠O1 AO2 = 90°

∴ □ O1 AO2 B is a square . ∴∠AO1 B = 90° and ∠AO2 B = 90° =  × 90  =  


 180 2
c c
 π  π 

1 1 π 25π

∴ A ( sector O1 AB ) = A ( sector O2 AB ) = r 2θ = × 52 × = cm 2
2 2 2 4 A

1 25
Now area of ∆OAB = Area of ∆O2 AB = × 5 × 5 =
2 2
O1 O2

 25π 25  25
∴ Required area = 2  −  = × 2 (π − 2 )
 4 2  4
B

284. Ans. (T), The minute-hand of a clock goes from one mark to the next in 5 minutes. i.e., it moves
through an angle of ( 30 / 5) ° = 6° in 1 minute
On the other hand (1) , the hour-hand complete one rotaion is 12 hours, i.e., it moves through an angle
of 360° in 12 × 60 = 720 minutes. Hence, the hour-hand moves through an angle of
( 360 / 720°) = (1/ 2 ) ° in 1 minute
1  1
∴ in 1 minute, the minute-hand gains 6 −   =  5  = 5 30′ on the hour-hand
2  2
285. Ans. (c), Clearly cos 290° = cos ( 270° + 20° ) = sin 20° and sin 250° = sin ( 270° − 20° ) = − cos 20°

46 Office.: 606 , 6 Floor, Hariom Tower, Circular Road, Ranchi-1, Ph.: 0651-2562523, 9835508812, 8507613968
th
( ) BY R. K. MALIK’S NEWTON CLASSES 47

1 1 3 ⋅ cos 20° − sin 20°


∴ Given Expression =
sin 20° 3 ⋅ cos 20° 3 ⋅ sin 20° ⋅ cos 20°
− =

 3 1
2 ⋅ cos 20° − ⋅ sin 20° 
2 2  = 4 ⋅ ( sin 60° ⋅ cos 20° − cos 60° ⋅ sin 60° )

3 3 sin ( 2 × 20° )
= 
( 2 sin 20° ⋅ cos 20 )
2
4 sin ( 60° − 20° ) 4
3 sin 40° 3
= ⋅ =

286. Ans. (b), sin 2 A ⋅ tan 2 A + cos 2 A ⋅ cot 2 A

 For simplicity we have written 


2 3 2 3
s2 c2 s 4 c4 s6 + c6 ( s ) + ( c )
= s2 ⋅ 2 + c2 ⋅ 2 = 2 + 2 = 2 2 =
s 2c 2  s for sinA and c for cosA
 
c s c s sc 
= ( s 2 + c 2 ) − 3s 2 c 2 ( s 2 + c 2 ) 1 − 3s 2 c 2 1
3

= 2 2 −3
s 2c 2 2 2
=

= ( sec 2 A ⋅ cos ec 2 A ) − 3 = ( sec 2 A + cos ec 2 A ) − 3 = (1 + tan 2 A )(1 + cot 2 A ) − 3 = tan 2 A + cot 2 A − 1


sc sc

287. Ans. (c), sec θ ⋅ tan θ ( sec θ + tan θ ) + ( sec θ − tan θ ) = sec 2 θ ⋅ tan θ + sec θ ⋅ tan 2 θ + sec θ − tan θ
= ( sec 2 θ ⋅ tan θ − tan θ ) + ( sec θ + sec θ ⋅ tan 2 θ ) = tan θ ( sec 2 θ − 1) + sec θ (1 + tan 2 θ )
= tan θ ( tan 2 θ ) + sec θ ( sec 2 θ ) = tan 3 θ + sec3 θ = tan n θ + sec n θ ∴ n=3

288. Ans. (c), 4 ( sin 3θ ⋅ cos3 θ + cos 3θ ⋅ sin 3 θ )

= sin 3θ ⋅ ( 4 cos3 θ ) + cos 3θ ⋅ ( 4sin 3 θ ) = sin 3θ ⋅ ( 3cos θ + cos 3θ ) + cos 3θ ⋅ ( 3sin θ − sin 3θ )

= sin 3θ ⋅ ( 3cos θ ) + cos 3θ ⋅ ( 3sin θ ) = 3 ( sin 3θ ⋅ cos θ + cos 3θ .sin θ )


= 3sin ( 3θ + θ ) = 3sin ( 4θ ) = m ⋅ sin ( nθ ) ∴ m = 3, n = 4
sin ( 2 A + B ) 5
289. Ans. (b), ∵
sin B 1
=

sin ( 2 A + B ) + sin B 5 +1 2sin ( A + B ) ⋅ cos A 6 tan ( A + B ) 3


sin ( 2 A + B ) − sin B 5 −1 2 cos ( A + B ) ⋅ sin A 4 tan A 2
∴ = ∴ = ∴ =

290. Ans. (c), Squaring and adding we have ( sin A + cos B ) + ( sin B + cos A ) = a 2 + b 2
2 2

a2 + b2 − 2
⇒ 2 + 2sin ( A + B ) = a 2 + b 2 Hence sin ( A + B ) =
2
291. Ans. (a), ∵ cos θ1 = 2 ⋅ cos θ 2
cos θ1 1 cos θ1 − cos θ 2 1 − 2
cos θ 2 2 cos θ1 + cos θ 2 1 + 2
∴ = ∴ =

−2sin ⋅ sin
2 2 = − 1 ∴  tan θ1 − θ 2  tan θ1 + θ 2  = 1
θ1 + θ 2 θ1 − θ 2

2 cos 1 2 ⋅ cos 1 2 3 2  2  3

2 2
θ +θ θ −θ   

cos x + cos ( x + 2θ ) 2 cos ( x + θ ) ⋅ cos θ cos ( x + θ ) b


292. Ans. (c),
b + d cos ( x + θ ) + cos ( x + 3θ ) 2 cos ( x + 2θ ) ⋅ cos θ cos ( x + 2θ ) c
a+c
= = = =

Office.: 606 , 6th Floor, Hariom Tower, Circular Road, Ranchi-1, Ph.: 0651-2562523, 9835508812, 8507613968 47
48 (BY R. K. MALIK’S NEWTON CLASSES
)
293. Ans. (b), ∵ a ⋅ sec θ − b ⋅ tan 2 θ = c
2

∴ a ⋅ sec 2 θ − b ( sec 2 θ − 1) = c ∴ ( a − b ) sec 2 θ + b = c

∴ cos 2 θ = ∴ sin 2 θ = 1 − cos 2 θ = 1 −


cos θ
(a − b) = − b−a b−a
2 (b − c)

b−c b−c
b−c −b+a a −c
= =

294. Ans. (a), Given : a ⋅ cos A − b ⋅ sin A = c


b+a b−c

Let a ⋅ sin A + b ⋅ cos A = x


∴ x 2 + c 2 = ( a ⋅ cos A − b ⋅ sin A ) + ( a ⋅ sin A + b ⋅ cos A )
2 2

∴ Simplifying : x 2 + c 2 = a 2 + b 2 ⇒ x 2 = a 2 + b 2 − c 2 ∴ x = ± a 2 + b2 − c 2
1 1 − s 2 c2
295. Ans. (a), m = cos ec θ − sin θ = − s = = ( where c means cos θ and s means sin θ )
1 1 − c2 s 2
s s s

n = sec θ − cos θ = −c = =
c c c

⋅ = sc ∴ mn = sin θ ⋅ cos θ ……(i)


c2 s2
∴ mn =
s c

Also ; m 2 + n 2 + 3 = 3 +3
c4 s4 c6 + s6
s 2 c2 s 2c 2
+ + =

+ s 2 ) − 3s 2 c 2 ( c 2 + s 2 ) 1 − 3s 2 c 2 1 1  1  1
2 3
2 2

+3 = +3 = 2 2 −3+ 3 =   = 
(c
s2c2 2 2
=  = 2 2

∴ m 2 n 2 ( m 2 + n 2 + 3) = 1
sc sc  sc   mn  mn

sin  ⋅ cos  tan 


sin α 5 sin α + sin β 5 + 3  2   2
 8
 2
α + β  α − β  α + β 
296. Ans. (d), ∵  =4
 
sin β 3 sin α − sin β 5 − 3  2
cos   ⋅ sin  tan 
= ∴ = ⇒ = ⇒

 2   2  2
 α + β  α − β α − β 
 

297. Ans. (c), As tan 30° and tan15° are roots of x 2 + px + q = 0 . Hence, tan 30° + tan15° = − p,
 

tan 30° ⋅ tan15 = q


tan 30° + tan15°
Now = tan ( 30° + 15° )
1 − tan 30° ⋅ tan15°

= 1 ∴ − p = 1− q ∴ − p + q = 1 ∴ −p+q+2 = 3
1− q
−p

sin ( 2α + β ) 3
298. Ans. (c), ∵
sin β 1
=

sin ( 2α + β ) + sin β 3 +1 2sin (α + β ) ⋅ cos α


=2
sin ( 2α + β ) − sin β 3 −1 2 cos (α + β ) ⋅ sin α
∴ = ∴

∴ tan (α + β ) = 2 tan α ∴ tan (α + β ) − 2 tan α = 0, Hence, independent of both α and β

299. Ans. (b), ∵ a 2 + b 2 = ( cos α + cos β ) + ( sin α + sin β )


2 2

∴ simplifying r.h.s. we get a 2 + b2 = 2 + 2cos (α − β )


= 2 + 2 cos 2θ = 2 (1 + cos 2θ ) = 2 ( 2 cos 2 θ ) = 4 cos 2 θ

48 Office.: 606 , 6 Floor, Hariom Tower, Circular Road, Ranchi-1, Ph.: 0651-2562523, 9835508812, 8507613968
th
( ) BY R. K. MALIK’S NEWTON CLASSES 49

cos 3θ 4 cos3 θ − 3cos θ


= ( 4 cos 2 θ ) − 3 = a 2 + b 2 − 3
cos θ cos θ
∴ =

300. Ans. (b), ∵ Given sec α + cos ec α = p, sec α ⋅ cos ec α = q


1 1 1 sin α + cos α 1
= p, = p,
cos α sin α sin α ⋅ cos α cos α ⋅ cos α sin α ⋅ cos α
∴ + =q ∴ =q

∴ sin α + cos α = ∴ ( sin α + cos α ) = 2 ∴ ( sin α + cos α ) + 2sin α ⋅ cos α = 2


p 2 p2 2 2 p2

2 p2
q q q

∴ 1+ ∴ q 2 + 2q = p 2 ∴ p 2 = q ( q + 2 )
q q2
=

301. Ans. (a), sin 2 θ + 3cos θ − 2 = 0 ∴ (1 − cos 2 θ ) + 3cos θ − 2 = 0


1
∴ cos 2 θ + 1 = 3cos θ ∴ cos θ + = 3 ∴ cos θ + sec θ = 3 …. (i)
cos θ
( cos θ + secθ ) = 33 ∴ ( cos3 θ + sec3 θ ) + 3 ( cos θ ⋅ sec θ )( cos θ + sec θ ) = 27
3

∴ cos3 θ + sec3 θ + 3 (1)( 3) = 27 ∴ cos3 θ + sec3 θ = 18


302. Ans. (b), Squaring and dividing the first equation by second equation we get
x 2 / y = sin 3 θ and similarly y 2 / x = cos3 θ

/ y) + ( y2 / x ) = sin 2 θ + cos 2 θ = 1
2 2/3 2/3
∴ (x
303. Ans. (d), Given ⇒ ( tan x + cot x ) = 4 2 ∴ tan 2 x + cot 2 x + 2 = 16 ∴ tan 2 x + cot 2 x = 14
2

∴ ( tan 2 x + cot 2 x ) = 142 ∴ tan 4 x + cot 4 x + 2 = 196 ∴ tan 4 + cot 4 = 194

304. Ans. (d) We have, a + b cos 2 x + c sin 2 x = 0 for all x


⇒ a + b + ( c − 2b ) sin 2 x = 0 for all x ⇒ a + b = 0 and c − 2b = 0 ⇒ a = −b and c = 2b

Thus, the triplets are ( −b, b, 2b ) , where b ∈ R. Hence, there are infinitely many triplets.

tan 2 θ tan 2 θ
305. Ans. (d) We have, = sin 2 θ .
1 + tan θ sec θ
2 2
=

So, statement-2 is true.


tan A + tan B a
We have, tan A + tan B = a and tan A tan B = −1 ∴ tan ( A + B ) =
1 − tan A tan B 2
=

a2
tan ( A + B ) 4 = a
Hence, sin 2 ( A + B ) = So, statement-1 is not correct.
2 2

1 + tan 2 ( A + B ) a2 4 + a2
1+
=

4
306. Ans. (d) We have, tan ( A + B ) = p and tan ( A − B ) = q

tan ( A + B ) + tan ( A − B )
∴ tan 2 A = tan {( A + B ) + ( A − B )} ⇒ tan 2 A =
1 − tan ( A + B ) tan ( A − B ) 1 − pq
p+q
=

sin 2 3 A cos 2 3 A
307. Ans. (b) We have,
sin 2 A cos 2 A

Office.: 606 , 6th Floor, Hariom Tower, Circular Road, Ranchi-1, Ph.: 0651-2562523, 9835508812, 8507613968 49
50 ( )BY R. K. MALIK’S NEWTON CLASSES
sin 2 3 A cos 2 A − cos 2 3 A sin 2 A sin 3 A (1 − sin A ) − cos 3 A sin A
2 2 2 2

sin 2 A cos 2 A sin 2 A cos 2 A


= =

sin 2 3 A − sin 2 A ( cos 2 3 A + sin 2 3 A ) sin ( 3 A + A ) sin ( 3 A − A )


sin A cos A
2 2
sin 2 A cos 2 A
= =

( 4 sin A cos A cos 2 A )( 2sin A cos A ) = 8cos 2 A


sin 2 A cos 2 A
=

5A
308. Ans. (c) We have, sin sin
2 2
A

1 5A  1 1
=  2 sin sin  = ( cos 2 A − cos 3 A ) = {2 cos A − 1 − 4 cos A + 3cos A}
2 3

2 2 2  2 2
A

1 9 27 3  11
2 × − 1 − 4 × + 3 ×  =
2  11 64 4  32
=

309. Ans. (d) We have, sin x + sin 2 x = 1 ⇒ sin x = 1 − sin 2 x ⇒ sin x = cos 2 x.

∴ cos8 x + 2 cos 6 x + cos 4 x = sin 4 x + 2sin 3 x + sin 2 x = ( sin x + sin 2 x ) = 1


2

2π 4π
310. Ans. (b) We have, x = y cos = z cos
3 3

= λ (say)
2 2 1 −2 −2
y z x y z
⇒ x=− =− ⇒ = =

⇒ x = λ , y = −2λ , z = −2λ ⇒ xy + yz + zx = −2λ 2 + 4λ 2 − 2λ 2 = 0


311. Ans. (c) We have, sin α = sin β , cos α = cos β

⇒ sin α − sin β = cos α − cos β = 0 ⇒ 2 cos sin = −2 sin sin =0


2 2 2 2
α +β α −β α +β α −β

⇒ sin =0
2
α −β

312. Ans. (d), For varying values of A, B and C the expression will attain the maximum value when

cos 2 A, cos 2 B attain their maximum values each equal to 1 and cos 2 C is least i.e., 0.
Hence, required maximum value = 1 + 1 − 0 = 2.

50 Office.: 606 , 6 Floor, Hariom Tower, Circular Road, Ranchi-1, Ph.: 0651-2562523, 9835508812, 8507613968
th

You might also like